Vous êtes sur la page 1sur 146

A 65-year-old African-American man is brought to the emergency department (ED) with a two-

hour history of sudden onset of sharp, stabbing chest pain under his sternum. He was doing
some physical activity when the symptoms started. The pain reached its maximum intensity
within a few minutes, and is now radiating to his upper back and shoulder tips. His past
medical history is significant for hypertension for the past 10 years. He is a heavy smoker. His
blood pressure is 196/112 mmHg in the right arm and 120/62 mmHg in the left arm. His pulse
rate is 120/min and respiratory rate is 18/min. The lung examination is clear to auscultation
bilaterally. The cardiovascular examination reveals a regular heart rhythm with an early
diastolic murmur heard at the right sternal border. A stat transesophageal echocardiogram is
done in the ED, which confirms the diagnosis of acute aortic dissection with extension into the
left subclavian artery. What is the best next step in the management of this patient?
A. Aspirin
B. Heparin
C. Propranolol
D. Sodium nitroprusside
E. Hydralazine

Explanation:
Acute aortic dissection refers to a tear in the aortic intima with separation of the intima from
the media, thereby creating a false lumen within the aortic wall. The dissection may propagate
either distal or proximal to the intimal tear, causing the associated clinical manifestations. It is
usually seen in elderly males with a longstanding history of hypertension and atherosclerosis;
however, some conditions associated with or predisposing to aortic dissection in younger
patients include a history of connective tissue disorder (Marfan's syndrome, Ehler-Danlos
syndrome), inflammatory vasculitis (Takayasu's arteritis, giant cell arteritis, syphilis aortitis),
bicuspid aortic valve, coarctation of aorta, use of crack cocaine and trauma.
Most of the patients present with a sudden onset of sharp tearing chest or back pain. Patients
with ascending aortic dissection may also develop acute aortic insufficiency, thereby causing
acute heart failure (with early diastolic decrescendo murmur), acute myocardial infarction due
to the dissection extending into coronary vessels, cardiac tamponade or hemothorax or
neurological deficits due to the direct extension of dissection into the carotid arteries. Physical
examination may reveal a significant difference in the blood pressure between the two arms
due to the involvement of the subclavian vessels. The diagnosis is usually suspected based on
the history and physical examination, along with mediastinal widening on chest radiograph and
the absence of typical electrocardiographic findings of myocardial ischemia or infarction.
Transesophageal echocardiography is the modality of choice for rapid confirmation of the
diagnosis.
Acute aortic dissection involving the ascending aorta is a life-threatening emergency, and all
such patients should have emergent surgical intervention. The goal of early medical treatment
is to rapidly reduce the shearing stress on the aortic wall and prevent the further propagation of

3/19/2011 Item 1 of 38
usmleworld.com/Step3//qstShow.as 1/2
dissection. This is achieved by rapidly lowering the systolic blood pressure and left ventricular
contractility with the use of intravenous beta-blockers. Intravenous beta-blockers are the initial
drugs of choice in the management of patients with aortic dissection. The goal is to reduce the
systolic blood pressure to 100 to 120 mmHg, and to reduce the heart rate to less than 60/min.
An intravenous loading dose of propanolol or labetalol followed by an IV infusion can be used
to achieve the desired heart rate and blood pressure in most patients. If the blood pressure
continues to remain high (systolic blood pressure greater than 100 mmHg), direct vasodilator
sodium nitroprusside should be added to the beta-blockers to achieve the desired blood
pressure goal.
(Choice A) Aspirin has no role in the management of patients with aortic dissection.
(Choice B) Heparin is absolutely contraindicated in patients with aortic dissection.
(Choice D) Sodium nitroprusside should only be used in conjunction with beta-blockers. When
used alone, it can cause reflex activation of the sympathetic nervous system, thereby causing
an increase in the heart rate and ventricular contractility. This, in turn, leads to an increase in
the shearing stress on the aortic wall and causes worsening of the aortic dissection.
(Choice E) Hydralazine is another direct vasodilator and causes reflex activation of
sympathetic nervous system. It has a less predictable blood pressure response and should be
avoided in patients with acute aortic dissection.
Educational Objective:
Intravenous beta-blockers (propranolol or labetalol) should be used in the initial management
of patients with acute aortic dissection. Sodium nitroprusside should not be used alone for
blood pressure control without prior beta blockade.
41% of people answered this question correctly;
3/19/2011 Item 1 of 38
usmleworld.com/Step3//qstShow.as 2/2

A 53-year-old Caucasian sales clerk presents to the emergency department after spitting up
blood while at work. She is very concerned and says this has never happened before. She
reports a lengthy history of shortness of breath, fatigue, palpitations, and cough while lying
down, but has never sought medical attention for these symptoms because she has no health
insurance. Her medical history is significant for atrial fibrillation and iron-deficient anemia, and
she underwent a cholecystectomy ten years ago. Physical examination reveals a thin, fatigued,
anxious woman in no acute distress. She is mildly tachypneic. J ugular venous distention is
present, and there is a questionable apical low-pitched rumbling murmur. Bilateral rales are
noted. An electrocardiogram shows a broad notched P wave in lead II as well as some right
axis deviation. Chest radiograph shows Kerley B lines, a redistribution of blood flow toward
the apices, prominent pulmonary arteries at the hilum, elevation of the left mainstem bronchus,
and left atrial enlargement with a flattening of the left heart border. What is the most likely
diagnosis?
A. Mitral stenosis
B. Mitral regurgitation
C. Aortic stenosis
D. Aortic insufficiency
E. Pulmonic stenosis

Explanation:
Mitral stenosis (Choice A) is a narrowing of the outflow tract from the left atrium to the left
ventricle, resulting in increased pressure in the left atrium, pulmonary vasculature, and right
side of the heart. It is more common in women, and is most commonly linked to a remote
episode of rheumatic fever, though congenital mitral stenosis may be found as well. On
physical examination, mitral stenosis produces a diastolic thrill palpable over the apex and a
low-pitched, rumbling diastolic murmur best heard over the apex when the patient is lying in a
left lateral decubitus position.
Mitral regurgitation (Choice B) causes an apical holosystolic murmur, and is most often
secondary to myocardial infarction, mitral valve prolapse, rheumatic heart disease, or coronary
artery disease.
Aortic stenosis (Choice C) causes a crescendo-decrescendo systolic murmur with a normal
S1, a diminished A2, and a paradoxical splitting of S2. It is most often secondary to senile
degenerative calcification, or congenital malformation.
Aortic insufficiency (Choice D) causes a diastolic murmur best heard adjacent to the sternum
in the second to fourth intercostal space. It is most often secondary to infective endocarditis,
congenital malformation, connective tissue disorders, or rheumatic heart disease.
Pulmonic stenosis (Choice E) causes a systolic crescendo-decrescendo ejection murmur in
the left upper sternal border. It is most often secondary to congenital malformation, rheumatic

3/19/2011 Item 2 of 38
usmleworld.com/Step3//qstShow.as 1/2
heart disease, or carcinoid.
Educational Objective:
Mitral stenosis can result in hemoptysis. It causes a low-pitched, rumbling diastolic murmur
and is most commonly linked to a remote episode of rheumatic fever. Prominent pulmonary
arteries at the hilum, elevation of the left mainstem bronchus, and left atrial enlargement with a
flattening of the left heart border are the other clues for the diagnosis.
61% of people answered this question correctly;
3/19/2011 Item 2 of 38
usmleworld.com/Step3//qstShow.as 2/2

A 55-year-old male with a history of leukemia comes to the emergency department and
complains of shortness of breath and weakness. He is tachycardic, hypotensive, and ill-
looking. Auscultation reveals distant heart sounds. Which of the following statements regarding
his diagnosis and tests is true?
A. An inspiratory fall in blood pressure of 10 mmHg is diagnostic of cardiac
tamponade.
B. A diffuse ST segment elevation in the electrocardiogram is diagnostic for
constrictive pericarditis.
C. Echocardiogram will show pericardial effusion with evidence of right atrial and/or
ventricular wall collapse during diastole, thereby leading to a diagnosis of cardiac
tamponade.
D. A chest x-ray showing an enlarged heart with evidence of pulmonary edema will lead
to a diagnosis of acute pericarditis.
E. Soft heart sounds on auscultation are reflective of a stenotic valve and lead to a
diagnosis of mitral stenosis.

Explanation:
A pericardial effusion may occur in response to any cause of pericarditis or from
malignancies. The pericardial effusion may develop slowly or rapidly; however, sudden filling
of the pericardial space with fluid can have catastrophic consequences by limiting ventricular
filling.
Patients with pericardial tamponade often complain of shortness of breath. The typical
physical signs and symptoms arise from the limited filling of the ventricle. The classic Beck's
triad is hypotension, muffled or distant heart sounds, and elevated jugular venous pressure.
Cardiac tamponade is a surgical emergency, and an ECHO should be obtained urgently. The
ECHO will reveal the large pericardial effusion, with prominent collapse of the right atrium and
ventricle during diastole. Cardiac catheterization can confirm the diagnosis by showing
equalization of diastolic pressures in all chambers. However, if the patient is collapsing, one
should not wait for an ECHO. Rapid pericardiocentesis is life-saving.
(Choice A) Arterial systolic blood pressure normally drops 10 -12 mmHg with inspiration. A
marked inspiratory drop in systolic blood pressure (>20 mmHg) is an important physical
finding in cardiac tamponade; however, this can also be seen in severe obstructive pulmonary
disease and constrictive pericarditis.
(Choice B) Diffuse ST segment elevations present in all leads may indicate acute
pericarditis. In pericardial tamponade, low limb voltage with alternating size of QRS complex
due to swinging of the heart is seen.
(Choice D) The chest x-ray in cardiac tamponade will show an enlarged cardiac silhouette.
When a chest
x-ray shows an enlarged heart with pulmonary edema, congestive heart failure should be

3/19/2011 Item 3 of 38
usmleworld.com/Step3//qstShow.as 1/2
suspected. In cardiac tamponade, ther is usually no pulmonary edema.
(Choice E) The heart sounds are muffled or distant in cardiac tamponade because of the fluid
around the heart. In mitral stenosis, the first heart sound is loud. There is an opening snap and
a diastolic murmur. Mitral stenosis is usually caused by rheumatic fever.
Educational Objective:
Cardiac tamponade will cause a limitation of fluid entering the ventricles, and an ECHO will
reveal diastolic collapse of both the right atrium and ventricle.
65% of people answered this question correctly;
3/19/2011 Item 3 of 38
usmleworld.com/Step3//qstShow.as 2/2

The following vignette applies to the next 2 items
A 55-year-old Caucasian man comes to the emergency department with a sudden onset of
retrosternal chest pain that began eight hours ago. An electrocardiogram revealed the
presence of ST segment elevation in leads V2 to V5. He is taken emergently for cardiac
catheterization, where he undergoes percutaneous transluminal coronary angioplasty with
stent placement to the left anterior descending artery. After the procedure, the ST changes
resolve, and the patient has an uneventful hospital course. On the third night, just prior to his
discharge, he called for the nurse due to a sudden onset of sharp, retrosternal chest pain. He
first noticed the pain while he was turning around in the bed. The pain gets worse with deep
breathing. His vital signs are as follows: temperature 37.8C (100F), blood pressure 134/80
mmHg, heart rate 108/min, and respiratory rate 22/min. Physical examination reveals jugular
venous distention (3 cm), regular heart sounds without any murmurs, and clear lung fields.
There is a scratchy sound heard during ventricular systole over the left sternal border. An EKG
done during the episode reveals the presence of sinus tachycardia with Q waves in leads V2
to V5.
Item 1 of 2
Which of the following is the most likely cause of this patient's recurrent symptoms?
A. Acute pericarditis
B. Left ventricular aneurysm
C. Chordae tendineae rupture
D. Acute myocardial infarction
E. Dressler's syndrome

Explanation:
The patient has a clinical syndrome consistent with a diagnosis of acute pericarditis. Acute
pericarditis (infarct associated or infarction pericarditis) can occur within one to four days as a
direct complication of a transmural myocardial infarction. The recurrence of chest pain three
days after a myocardial infarction, which gets worse with position changes and deep
inspiration, is suggestive of infarction pericarditis. It is usually a clinical diagnosis, and is
supported by the presence of pericardial friction rub and EKG changes of pericarditis (sinus
tachycardia, diffuse ST segment elevations with PR segment depression). A pericardial rub is
usually heard over the left sternal border and can be present during any of the phases of the
cardiac cycle. It is heard as a superficial scratchy or grating sound, which gets more
pronounced when the patient leans forward. The EKG changes of pericarditis are not always
seen, but usually resemble that of an acute myocardial infarction.
Infarction pericarditis is seen less frequently in patients with early and complete reperfusion. It
is usually a transient episode and does not affect the management of acute myocardial
infarction, unless it is complicated by a large pericardial effusion or tamponade.

3/19/2011 Item 4 of 38
usmleworld.com/Step3//qstShow.as 1/2
(Choice B) A left ventricular aneurysm usually occurs as a late complication of a transmural
myocardial infarction (usually a large anterior wall MI). It may result in heart failure, ventricular
arrhythmias, or peripheral arterial embolization due to the formation of a left ventricular
thrombus. It develops over a longer period of time, and is usually not associated with chest
pain. Persistent ST elevation can be present in these patients.
(Choice C) Chordae tendineae or papillary muscle rupture is a life threatening mechanical
complication of an acute myocardial infarction. It usually occurs two to seven days after the
infarction and causes acute hemodynamic instability. Patients also develop acute pulmonary
edema.
(Choice D) Acute myocardial infarction is unlikely in the absence of any specific EKG
changes of ischemia or infarction. The presence of 'Q' waves indicates an old infarction in this
patient.
(Choice E) Dressler's syndrome, or post-cardiac injury syndrome, occurs in patients with
myocardial infarction and after cardiac surgery. It is an autoimmune mediated syndrome,
which usually develops weeks to months after an acute MI. It usually presents with fever,
leukocytosis, pleuritic chest pain, and a pericardial rub.
Educational Objective:
Acute pericarditis (infarction pericarditis) can occur within one to four days as a complication
of transmural myocardial infarction.
47% of people answered this question correctly;
3/19/2011 Item 4 of 38
usmleworld.com/Step3//qstShow.as 2/2

Item 2 of 2
Which of the following is the most appropriate next step in the management of this patient?
A. High-dose aspirin
B. Glycoprotein IIb/IIIa inhibitors
C. Oral corticosteroids
D. Urgent cardiac catheterization for in-stent restenosis
E. Immediate cardiothoracic surgical referral

Explanation:
Acute pericarditis occurring after a myocardial infarction is usually transient (lasts for only a
few days). It should be managed with close clinical observation and adequate pain control.
Aspirin (anti-inflammatory doses) is effective in alleviating the pain associated with acute
pericardial inflammation. There are some concerns that other NSAIDs may increase the risk of
myocardial rupture after a transmural MI.
(Choices B and D) There is no evidence of myocardial ischemia or infarction in this patient.
Glycoprotein IIb/IIIa inhibitors and emergent cardiac catheterization are therefore not indicated
in this patient.
(Choice C) Corticosteroids are useful in refractory cases of Dressler's syndrome, when
nonsteroidal antiinflammatory drugs fail to control the symptoms. Its use has been reported to
increase the incidence of ventricular aneurysm formation.
(Choice E) The patient in the above vignette does not exhibit any signs of acute mitral
regurgitation (MR) or chordae tendinae rupture. Urgent cardiothoracic surgical evaluation and
management are required for patients with chordae tendineae rupture since it leads to an
acute MR and hemodynamic instability.
Educational Objective:
Acute pericarditis following a transmural myocardial infarction is usually transient and should
be managed by pain control with aspirin.
66% of people answered this question correctly;

3/19/2011 Item 5 of 38
usmleworld.com/Step3//qstShow.as 1/1

A 44-year-old Caucasian woman presents to the ER after sudden onset of weakness and
dizziness. She denies any chest pain but feels her heart pounding in her chest. A portion of her
EKG is shown below.
Which of the following is the best initial treatment for this patient's condition?
A. Quinidine
B. Digoxin
C. Adenosine
D. Isoproterenol
E. Lidocaine

Explanation:
This patient has a narrow QRS complex tachycardia consistent with a supraventricular
tachycardia (SVT). Given this patient's age, gender, and the regularity of the QRS complexes,
she may have paroxysmal SVT (PSVT). However, it is often difficult to distinguish between
PSVT and the numerous other types of SVT (e.g., sinus tachycardia, AV nodal reentrant
tachycardia, AV reentrant tachycardia, atrial fibrillation, and atrial flutter). One diagnostic and
therapeutic maneuver to distinguish between these entities is to increase vagal tone to the
heart by Valsalva or adenosine administration. This will slow the heart rate until a rhythm can
be recognized. It can also potentially "break" the SVT and return the patient to normal sinus
rhythm.
(Choice A) Quinidine is a class IA antiarrhythmic used to treat a spectrum of rhythm
disturbances. Notably, quinidine has anti-vagal actions and often induces a tachycardia. This
drug may be associated with increased mortality in atrial fibrillation. It is not a first-line
treatment for SVT.
(Choice B) Digoxin is a cardiac glycoside used to treat symptomatic heart failure. It
decreases heart failure symptoms and hospitalizations but does not prolong life. Digoxin is
also an AV node blocker and can be useful in treating atrial fibrillation. It is not, however, a
first-line agent for SVT.
(Choice D) Isoproterenol is a -1 and -2 adrenergic agonist. As a result, administration of
isoproterenol will increase heart rate and contractility due to its -1 effects and decrease
mean arterial blood pressure due to -2-mediated vasodilation.

3/19/2011 Item 6 of 38
usmleworld.com/Step3//qstShow.as 1/2
(Choice E) Lidocaine is a class IB antiarrhythmic used predominantly to treat ventricular
arrhythmias. Lidocaine is not recommended for SVT.
Educational Objective:
Supraventricular tachycardias are characterized by narrow QRS complexes. The initial
diagnostic and therapeutic maneuver is to increase a patient's vagal tone via the Valsalva
maneuver or adenosine administration.
67% of people answered this question correctly;
3/19/2011 Item 6 of 38
usmleworld.com/Step3//qstShow.as 2/2

A 53-year-old Caucasian businessman is brought in to the emergency department by
coworkers. He was in a meeting at work when he suddenly experienced severe pain and
tightness in his chest. The pain radiates to his jaw. He reports feeling very anxious and
lightheaded, and is extremely sweaty upon arrival. He was given supplemental oxygen and
intravenous access was established. His vital signs include temperature of 36.9C (98F), blood
pressure of 132/90 mm Hg, pulse of 82/min, and respirations are 20/min. No
electrocardiogram results are yet available. What medication should be administered next?
A. Aspirin
B. Captopril
C. Heparin
D. Metoprolol
E. Streptokinase

Explanation:
Unless there is clear evidence to the contrary, clinicians managing chest pain should always
assume it is ischemic in origin. Supplemental oxygen should be administered, pulse oximetry
readings obtained, and intravenous access established. Ideally, 325 mg of aspirin (Choice A)
should be chewed en route to the hospital. Nitroglycerin is typically given next, unless
contraindications exist (e.g., hypotension or recent usage of sildenafil).
ACE inhibitors such as captopril (Choice B) are not as commonly given in the acute setting.
However, they are known to significantly improve cardiac hemodynamics and reduce mortality
when given for a several week period following myocardial infarction.
Heparin (Choice C) may be given intravenously after aspirin, nitroglycerin, and morphine have
been administered and the electrocardiogram performed.
Beta-blockers such as metoprolol (Choice D) are of help in controlling heart rate and
decreasing myocardial oxygen demand, and are typically given after aspirin, nitroglycerin, and
morphine have been administered.
Streptokinase (Choice E) is used for thrombolysis as an alternative to primary coronary
intervention (e.g., stent placement or balloon dilation), but is not one of the first medications
administered after myocardial infarction.
Educational Objective:
In patients whose chest pain may be ischemic in origin, supplemental oxygen and aspirin
should be administered immediately, followed by nitroglycerin, morphine, and beta-blockers
(presuming no contraindications exist).
90% of people answered this question correctly;

3/19/2011 Item 7 of 38
usmleworld.com/Step3//qstShow.as 1/1

The following vignette applies to the next 3 items
A 70-year-old African-American woman is brought to the emergency department because she
woke up this morning with a sudden onset of shortness of breath. She has never had angina or
shortness of breath. She denies wheezing, chest pain, leg swelling, palpitations, and loss of
consciousness. Her other medical problems include hypertension, diabetes mellitus-type 2,
and hypercholesterolemia. She had bronchial asthma when she was a child. Her only hospital
admission occurred two months ago, when she underwent a spinal fusion for lower back
problems. She denies the use of alcohol or tobacco. Her family history is not significant. Her
temperature is 36.1 C (97 F), blood pressure is 210/106 mmHg, pulse is 65/min, and
respirations are 34/min. Her pulse oximetry reading is 81% at room air. Physical examination
shows diffuse crackles all over the lung fields. Her labs reveal the following:
CBC
Hb 11.4 g/dL
Platelet count 180,000/cmm
Leukocyte count 8,000/cmm
Her chest x-ray is shown below. Oxygen is administered.
Item 1 of 3
Which of the following is the most appropriate next step in the management of this patient?
A. Give IV nitroglycerine
B. Give intravenous metoprolol
C. Administer furosemide
D. Start intravenous heparin
E. Albuterol nebulization

Explanation:

3/19/2011 Item 8 of 38
usmleworld.com/Step3//qstShow.as 1/2
The patient's chest x-ray is characteristic of flash pulmonary edema, which is most likely due to
hypertensive crisis. The initial steps in management of patients with acute pulmonary edema
includes the administration of oxygen, morphine, and loop diuretics (IV furosemide).
Educational objective:
Patients with acute pulmonary edema are initially managed with oxygen, morphine and loop
diuretics (IV furosemide).
61% of people answered this question correctly;
3/19/2011 Item 8 of 38
usmleworld.com/Step3//qstShow.as 2/2

Item 2 of 3
The patient is given the appropriate treatment. She also undergoes further evaluation. Her
EKG reveals left ventricular hypertrophy with repolarization changes. The first set of cardiac
enzymes is negative. She is still in moderate distress, and has similar vital signs. What is the
next best step in the management of this patient?
A. Add intravenous steroids
B. Start dopamine
C. Increase metoprolol
D. Start IV nitroglycerine
E. Continue heparin

Explanation:
This patient's flash pulmonary edema is most likely due to hypertensive crisis. The two most
commonly used antihypertensive agents in the management of hypertensive crisis
complicated by acute pulmonary edema are IV nitroglycerine and IV nitroprusside.
(Choice B) Dopamine is indicated in patients with cardiogenic shock complicated by
hypotension. This patient has hypertensive crisis; therefore, the use of dopamine in this case
is very inappropriate.
(Choice E) Heparin is indicated in either pulmonary embolism (PE) or acute coronary event;
however, there is no evidence of ischemia (normal EKG), no chest pain, and no elevated
cardiac enzymes, which could suggest the latter. PE is also unlikely in this case, given the
patient's findings of pulmonary edema in the chest x-ray, as well as hypertension.
(Choice C) Beta-blockers are not usually given in acute pulmonary edema.
(Choice A) Steroids are indicated in acute bronchial asthma or COPD exacerbation, which is
not the case here.
Educational objective:
The two most commonly used antihypertensive agents in the management of hypertensive
crisis complicated by acute pulmonary edema are IV nitroglycerine and IV nitroprusside.
59% of people answered this question correctly;

3/19/2011 Item 9 of 38
usmleworld.com/Step3//qstShow.as 1/1

Item 3 of 3
The patient is responding well to the treatment. The second set of cardiac enzymes is
negative. What other test is necessary for this patient?
A. Echocardiogram
B. Coronary angiogram
C. Pulmonary function tests
D. V/Q scan and D-dimer
E. Renal duplex scan

Explanation:
Mitral stenosis and acute aortic or mitral regurgiation can sometimes present with flash
pulmonary edema; therefore, an echocardiogram should be performed to evaluate the systolic
and diastolic function, and to check for any left ventricular hypertrophy or valvular disease.
(Choice E) A renal duplex scan is indicated in patients with severe, uncontrolled hypertension
and recurrent episodes of flash pulmonary edema.
Educational objective:
All patients with flash pulmonary edema of unknown etiology should be evaluated with an
echocardiogram.
68% of people answered this question correctly;

3/19/2011 Item 10 of 38
usmleworld.com/Step3//qstShow.as 1/1

A 62-year-old man comes to the emergency department because of shortness of breath, chest
pain, and palpitations. His other medical problems include hypertension, obesity, chronic
obstructive pulmonary disease, myocardial infarction requiring a coronary artery bypass graft,
and carotid artery stenosis. His medications include metoprolol, aspirin, tiotropium, and
lisinopril. Electrocardiogram shows atrial fibrillation with a rapid ventricular response
(140150/min). The patient is connected to a cardiac monitor. While the team is securing
intravenous access, the patient becomes unresponsive. He has no palpable pulses over the
major arteries. The cardiac monitor continues to show atrial fibrillation at a rate of 145/min.
Which of the following is the most appropriate next step in the management of this patient?
A. Arterial blood gas test
B. Chest compressions
C. Defibrillation
D. Intravenous bicarbonate and digoxin
E. Synchronized cardioversion

Explanation:
This patient's condition would be classified as pulseless electrical activity (PEA), which is
defined as the presence of an organized electrocardiographic rhythm (such as atrial fibrillation
with a rapid ventricular rate, as in this patient) without sufficient cardiac output to produce a
palpable pulse or measurable blood pressure. The absence of a palpable pulse also means
that the brain is not being appropriately perfused; therefore, cardiopulmonary resuscitation
(CPR) is necessary despite the presence of an organized rhythm. The most important initial
step in the management of PEA is chest compressions. The airway should also be secured,
but the 2010 Advanced Cardiac Life Support (ACLS) guidelines place a greater importance
on circulation than on airway and breathing.
It is important to distinguish this patient's condition from unstable tachycardia. In unstable
tachycardia, the patient has hypotension, ischemic chest pain, altered mental status, or heart
failure, but a pulse is present. Cardioversion would be the best choice if this patient had
unstable tachycardia. However, defibrillation and cardioversion are not a component of ACLS
guidelines for PEA management. The patient should be evaluated and treated for an
underlying metabolic cause of his atrial fibrillation as resuscitation is taking place. If the patient
regains a pulse, then the situation would change, and cardioversion would likely be warranted
given that the patient would then be in an unstable tachycardia, assuming his rhythm remains
the same. It is important to adjust to a different ACLS algorithm as the clinical situation
changes.
(Choice A) Obtaining an arterial blood gas test is a reasonable step while the patient is being
resuscitated. However, the most important first step is to begin chest compressions in an
attempt to adequately perfuse the brain.
(Choice C) Defibrillation is only effective if there is a shockable rhythm, such as ventricular

3/19/2011 Item 11 of 38
usmleworld.com/Step3//qstShow.as 1/2
fibrillation or pulseless ventricular tachycardia.
(Choice D) The most important initial step is to initiate chest compressions. Epinephrine is an
important component of resuscitation in PEA patients, but bicarbonate or digoxin would not be
indicated at this time.
(Choice E) Synchronized cardioversion would be indicated for unstable tachycardia, but given
that this patient has no pulse, the PEA algorithm as opposed to the tachycardia algorithm
should be followed.
Educational objective:
Pulseless electrical activity (PEA) is defined as the presence of an organized
electrocardiographic rhythm without sufficient cardiac output to produce a palpable pulse or
measurable blood pressure. Immediate cardiopulmonary resuscitation with chest
compressions should be started. Defibrillation or cardioversion are not a component of
resuscitation for PEA, although cardioversion is the best choice in patients who have unstable
tachycardia with a pulse.
43% of people answered this question correctly;
3/19/2011 Item 11 of 38
usmleworld.com/Step3//qstShow.as 2/2

A 66-year-old man presents to the emergency department for evaluation of left-sided chest
pain. He describes his pain as sharp, non-radiating, and lasting several minutes at a time. His
chest pain is present both at rest and with exertion. He also has intermittent claudication with
walking that manifests as a crampy, right-sided thigh pain. His past medical history otherwise
includes hypertension, type 2 diabetes mellitus, osteoarthritis of the right knee, and obesity
(BMI of 42 kg/m
2
). His daily medications are aspirin, insulin detemir, lisinopril, metoprolol,
and atorvastatin. He also takes omeprazole for occasional heartburn. He has a 50-pack year
smoking history. He has no known allergies. His physical examination and electrocardiogram
are unremarkable. Which of the following is the most appropriate test for further evaluation of
this patient?
A. Adenosine myocardial perfusion imaging
B. Ambulatory 24-hour EKG monitoring
C. Exercise echocardiography
D. Exercise electrocardiography
E. Radionuclide ventriculography

Explanation:
The first concern when evaluating a patient such as this in the emergency department is
assessment of risk for coronary artery disease and possible acute coronary syndrome. This
patient's history of smoking, hypertension, diabetes, and obesity all place him at risk for
coronary artery disease. His symptoms of intermittent extremity claudication are almost
certainly secondary to atherosclerosis in the lower extremity arteries, which makes the
presence of atherosclerosis in the coronary arteries highly likely. However, it is unclear at this
point if this patient's symptoms are secondary to coronary artery disease or an unrelated
problem as his presentation is not entirely specific. Further evaluation with measurement of
cardiac markers would be recommended at this time. If his cardiac markers are negative,
stress testing before discharge would be ideal to help risk-stratify the patient. Given his
intermittent claudication and osteoarthritis, however, he will likely be unable to effectively
exercise for the stress test. An adenosine myocardial perfusion test would therefore be the
best choice for this particular patient.
(Choice B) Ambulatory 24-hour EKG monitoring is most useful when there is suspicion for an
intermittent arrhythmia, for example, in patients being evaluated for syncope.
(Choice C) This patient is unlikely to be able to exercise effectively and therefore an exercise
echocardiogram would not be a good choice. A dobutamine echocardiogram could be
considered, but given the patient's obesity, finding an appropriate acoustic window can
sometimes be difficult.
(Choice D) Exercise electrocardiography is currently less frequently used as a stress testing
modality (compared to exercise echocardiography). This test's sensitivity is lower than if
imaging with echocardiography or radionuclides are used. Therefore in a patient like this one

3/19/2011 Item 12 of 38
usmleworld.com/Step3//qstShow.as 1/2
with a reasonably high pretest probability, coronary artery disease could not be reliably
excluded even in the presence of a negative test.
(Choice E) Radionuclide ventriculography is a highly accurate method for determining
ejection fraction, and is often used to follow patients receiving cardiotoxic chemotherapy.
Educational objective:
Stress testing is needed to risk-stratify patients suspected of having coronary artery disease.
Exercise echocardiography or nuclear perfusion studies are preferred for most patients,
although a pharmacologic stress test may be needed in patients unable to exercise. Exercise
electrocardiography is less sensitive than exercise echocardiography or nuclear perfusion
studies.
65% of people answered this question correctly;
3/19/2011 Item 12 of 38
usmleworld.com/Step3//qstShow.as 2/2

A 65-year-old female presents to the emergency room with a chief complaint of headache.
Upon further questioning, she states that the headache is localized to the right side of her
head. The patient has no other complaints. Her only regular medications are acetaminophen
for arthritis in her left shoulder and metformin for recently diagnosed DM type II. On physical
exam you notice that the patients right pupil is smaller than the left and her right eyelid is
slightly drooping. Initial labs reveal:
Hemoglobin 12.2g/L
Platelets 206,000/mm3
Leukocyte count 4,500/mm3
ESR 20 mm/hr
What is the next best step in the management of this patient?
A. CT head without contrast
B. MRA of the head and neck
C. Begin oral prednisone
D. Order temporal artery biopsy
E. Perform lumbar puncture

Explanation:
This patient has a unilateral headache with associated Horners syndrome, which consists of
miosis (small pupil), ptosis (eyelid drooping), and often anhidrosis (lack of sweating) on the
affected side. This patient should be considered to have a carotid dissection until proven
otherwise. The key to the presentation is the presence of Horners syndrome, which occurs
because the sympathetic chain providing innervation to the head travels on the carotid artery.
Carotid artery dissections can be due to trauma, connective tissue disease, smoking, neck
manipulation, hypertension, and three point restraint seatbelts in motor vehicle accidents.
Once carotid artery dissection is suspected, the preferred method of noninvasive imaging is
MRA. If the MRA results are unclear but carotid dissection is still suspected, catheter
angiography is the definitive test. These patients are at a high risk for developing cerebral
infarction. Treatment consists primarily of anticoagulation with platelet agents and/or heparin.
(Choice A) CT head would be useful if an intracranial process were suspected, especially a
subarachnoid hemorrhage or bleed. However, the presence of Horners syndrome points
more towards carotid dissection.
(Choice C) Oral prednisone would be indicated if temporal arteritis were suspected. Horner
syndrome is not seen typically in temporal arteritis. A highly elevated ESR would also be
helpful in distinguishing temporal arteritis from carotid dissection.
(Choice D) A temporal artery biopsy would be indicated if temporal arteritis was suspected,
but this is unlikely given the presence of Horners syndrome. Of note, treatment with

3/19/2011 Item 13 of 38
usmleworld.com/Step3//qstShow.as 1/2
prednisone should not be delayed while waiting for a temporal artery biopsy to be done.
(Choice E) A lumbar puncture is useful if one suspects meningitis or a subarachnoid
hemorrhage, but neither of these entities would normally present in this fashion.
Educational objective:
A unilateral headache with associated Horners syndrome (miosis, ptosis, and anhidrosis) is
generally a carotid dissection until proven otherwise, and MRA is the initial diagnostic modality
of choice.
17% of people answered this question correctly;
3/19/2011 Item 13 of 38
usmleworld.com/Step3//qstShow.as 2/2

The following vignette applies to the next 2 items
A 28-year-old Mexican-American woman comes to the emergency room because of
palpitations. She has no other medical problems. She does not use tobacco, alcohol, or
drugs. She takes no medications. Her temperature is 36.7C (98F), blood pressure is
100/60 mmHg, pulse is 172/min, and respirations are 20/min. A 12-lead EKG is taken, which
shows a heart rate of 172 beats/min with no P waves, and with regular and narrow- appearing
QRS complexes.
Item 1 of 2
Which of the following is the most likely diagnosis?
A. Atrial fibrillation
B. Ventricular tachycardia
C. Sinus tachycardia
D. Paroxysmal supraventricular tachycardia
E. Atrial flutter

Explanation:
The patient's EKG findings (narrow and regular QRS complexes, absent P wave) are
characteristic of a paroxysmal supraventricular tachycardia (SVT). SVT includes such
common entities as AV nodal reentrant tachycardia (AVNRT) and AV reentrant tachycardia.
The term 'paroxysmal SVT' excludes atrial fibrillation and atrial flutter, other common forms of
supraventricular arrhythmias.
(Choice C) P waves, along with narrow QRS complexes, are present in sinus tachycardia.
(Choice A) Atrial fibrillation is an "arrhythmic arrhythmia"; the rhythm is completely irregular,
and P waves are absent.
(Choice B) Ventricular tachycardia presents with wide QRS complexes (more than 0.12 sec).
(Choice E) Atrial flutter is characterized by the presence of "flutter waves" in a saw tooth
pattern. The heart rate for 'typical' atrial flutter with 2:1 conduction is usually around 150 bpm.
Educational Objective:
Paroxysmal SVT and sinus tachycardia are frequent types of tachyarrhythmias. Narrow and
regular QRS complexes, and absent P waves suggest paroxysmal SVT.
57% of people answered this question correctly;

3/19/2011 Item 14 of 38
usmleworld.com/Step3//qstShow.as 1/1

Item 2 of 2
The patient receives antiarrhythmic therapy, and her heart rate decreases from 172/min to
68/min. The palpitations resolve, and the patient begin to feel better. A new 12-lead EKG is
taken, which shows a heart rate of 70/min, regular rhythm, presence of P waves, a PR interval
of 0.10 sec, a slurred initial part of activation of the QRS complex, a QRS of 0.12 sec, and
nonspecific T wave abnormalities. Which of the following is the most likely diagnosis?
A. Mahaim tachycardia
B. Wolff-Parkinson White (WPW) syndrome
C. Dual AV nodal pathways
D. J ames fibers preexcitation
E. Right bundle branch block

Explanation:
The abovementioned features are characteristic of Wolff-Parkinson White (WPW) Syndrome.
The slurred upstroke of the QRS complex during the initial part or ventricular activation is
known as the delta wave. QRS duration is generally equal to 0.12 sec, but may have greater
values. Another feature of WPW syndrome is a PR interval less than 0.12 sec.
(Choices A and D) Mahaim and J ames fibers tachycardias are preexcitation forms that lack
delta waves.
(Choice E) A right bundle branch block (RBBB) has wide QRS complexes, but does not have
delta waves or short PR intervals. Furthermore, an RBBB will not explain the paroxysmal SVT
of the patient.
(Choice C) AV nodal reentrant tachycardias are the most common etiologies of paroxysmal
SVT. On resting EKG, there is no delta wave and the QRS complex is narrow.
Educational Objective:
WPW Syndrome can be identified in around 0.2% of the population. It is characterized by a
short PR interval (less than 0.12sec), a delta wave at the beginning of the QRS complex, QRS
duration of 0.12sec or wider, and non-specific ST segments or T wave abnormalities.
77% of people answered this question correctly;

3/19/2011 Item 15 of 38
usmleworld.com/Step3//qstShow.as 1/1

A 60-year-old man is brought to the emergency department due to acute-onset shortness of
breath. His past medical history is significant for hypertension, coronary artery disease, and
type 2 diabetes mellitus. His current medications are hydrochlorothiazide, insulin, simvastatin,
aspirin, and a daily multivitamin. His blood pressure is 128/72 mmHg, pulse is 116/min,
temperature is 36.7C (98F), and respirations are 28/min. Physical examination shows an
anxious-appearing, diaphoretic man. There is jugular venous distention, a regular heart rhythm,
and an ejection murmur over the cardiac base. Lung examination reveals crackles to the mid-
lung level bilaterally. He has pitting edema in both lower extremities. Which of the following is
the best initial approach to improving this patient's symptoms?
A. Blocking sympathetic activity
B. Decreasing cardiac preload
C. Decreasing heart rate
D. Improving myocardial contractility
E. Increasing coronary perfusion pressure

Explanation:
This patient's shortness of breath accompanied by jugular venous distention, crackles on
exam, and pitting edema are all consistent with congestive heart failure. The acuity of his
symptom onset would classify this as acute decompensated heart failure, or flash pulmonary
edema. The patient's tachycardia is likely secondary to poor myocardial contractility, although
there is no hypotension at this point to suggest cardiogenic shock. This patient will need
further evaluation to determine the underlying cause of his acute decompensated heart failure;
at this point the most emergent concern would be a myocardial infarction. A pulse oximetry
reading should be obtained, with administration of oxygen and/or noninvasive ventilation
administered as needed. Given that this patient is clearly volume overloaded, decreasing the
cardiac preload with a diuretic such as furosemide would be indicated at this time.
(Choice A) While morphine does not directly block sympathetic activation, it does relieve
patient anxiety which in turn may reduce sympathetic outflow. Data as to the safety and
efficacy of morphine in this setting is limited. If morphine is administered, it should not take the
place of diuretic therapy.
(Choice C) Decreasing the heart rate with medications such as beta-blockers does improve
mortality in the long-term in patients with chronic CHF. However, in this patient's acute
decompensated heart failure, his tachycardia is attempting to counteract the low cardiac
output. Heart rate-lowering medications would risk detrimental hypotension here.
(Choice D) Improving myocardial contractility with inotropic agents such as dobutamine or
milrinone should only be considered in patients with hypotension causing hypoperfusion.
(Choice E) Coronary artery vasodilators such as nitroglycerine may be indicated here if the
response to diuretic therapy is inadequate. It should not be given if the patient is hypotensive.

3/19/2011 Item 16 of 38
usmleworld.com/Step3//qstShow.as 1/2
Educational objective:
There are multiple possible causes of acute decompensated heart failure, or flash pulmonary
edema, with myocardial infarction being the most pressing entity on the differential diagnosis.
Initial management should involve ensuring adequate oxygenation and ventilation followed by
diuretic therapy.
50% of people answered this question correctly;
3/19/2011 Item 16 of 38
usmleworld.com/Step3//qstShow.as 2/2

A 64-year-old healthy male comes to your office after a local emergency department (ED) visit
for an episode of paroxysmal atrial fibrillation, which ceased spontaneously three weeks ago.
The ED doctor gave him a prescription for metoprolol, and advised a follow-up appointment
with you, the primary care physician. He currently has no complaints. He takes no other
medications. Physical examination revealed no abnormalities. EKG showed normal sinus
rhythm. A 2D-Echocardiogram is ordered, which revealed cardiac dilatation and an ejection
fraction of 45%. Which of the following is the most appropriate next step in diagnosis?
A. Refer the patient for ambulatory monitoring of blood pressure (AMBP).
B. Refer the patient for cardiac stress test.
C. Refer the patient for renal artery Doppler ultrasound.
D. Refer the patient for polysomnography.
E. Check BNP (B-Natriuretic peptide) and troponins.

Explanation:
Ischemic heart disease is the most common cause of congestive heart failure (CHF),
especially dilated cardiomyopathy, in the United States. Approximately 50 to 75% of the
patients with heart failure (HF) have coronary disease as the etiology. Other known etiologies
are: hypertension (13%), valvular disease (10-to12%), renovascular disease, and very rare
causes, such as obstructive sleep apnea, myocarditis, alcohol or cocaine abuse, etc.
Although this patient is asymptomatic, his echocardiogram results (cardiac dilatation and low
ejection fraction) are very suggestive of CHF. The next step in the management of this patient
is to identify the etiology of his CHF. Since ischemic heart disease is the most common
etiology, a cardiac stress test should be done first. The objective here is to quickly identify the
presence of ischemia to address the need for coronary angiography (for possible
revascularization).
(Choices A, C and D) Hypertension, renovascular disease, and sleep apnea are less
common causes of CHF.
(Choice E) Troponin and BNP level determination is not useful in this setting. Troponins can
be elevated in any type of heart failure or coronary ischemia. This is also true for BNP (-
natriuretic peptide), which is mainly used to distinguish cardiogenic pulmonary edema from
primary pulmonary conditions.
Educational Objective:
The most common cause of heart failure (CHF) is ischemic heart disease. In a new case of
CHF with unknown etiology, efforts must first be made to rule out the presence of coronary
lesions which may be corrected by an angioplasty. Other important causes of CHF are
hypertension, valvular and renovascular disease.
51% of people answered this question correctly;

3/19/2011 Item 17 of 38
usmleworld.com/Step3//qstShow.as 1/2
3/19/2011 Item 17 of 38
usmleworld.com/Step3//qstShow.as 2/2

A 42-year-old Caucasian woman is brought to the emergency room by paramedics with a
change in mental status. She has a past medical history of hypothyroidism, major depression,
and chronic back pain. She takes levothyroxine, amitriptyline, and oxycodone regularly. In the
emergency room, she is markedly confused, agitated, and keeps talking to herself. On
physical examination, her temperature is 38.9 C (102 F), respiratory rate is 22/minute, heart
rate is 110/minute, and blood pressure is 92/52 mmHg. There is marked flushing, mydriasis,
and hyperreflexia. Her EKG reveals the presence of sinus tachycardia, with widening of the
QRS interval. You start her on sodium bicarbonate infusion. While waiting for more data, she
has an episode of sustained ventricular tachycardia. Which of the following is the most
appropriate antiarrhythmic in this setting?
A. Lidocaine
B. Procainamide
C. Propranolol
D. Disopyramide
E. Quinidine

Explanation:
Tricyclic Antidepressants (TCA) are a frequently prescribed group of medications for
depression, despite having the highest risk for overdose and suicide attempts. TCA overdose
is the leading cause of hospitalization and death. The signs of TCA overdose include
hypotension, anticholinergic effects, CNS manifestations, and cardiac arrhythmias.
Cardiotoxic effects are responsible for most of the mortality in patients with TCA overdose.
TCAs inhibit fast sodium channels, which result to slowing of the phase 0 depolarization in His-
Purkinje tissue and the myocardium. This may lead to QRS prolongation and reentrant
arrhythmias, like ventricular tachycardia, ventricular fibrillation, and torsades de pointes.
Sodium bicarbonate is the most effective agent for the management of TCA-induced
cardiotoxic effects. Lidocaine is the antiarrhythmic drug of choice for TCA-induced ventricular
dysrhythmias.
(Choice C) Propranolol depresses myocardial contractility and conduction and, thus,
enhances the tricyclic toxicity.
(Choices B, D, and E) Antiarrhythmic drugs should be used with caution in patients with TCA
overdose. Procainamide, disopyramide, and quinidine have membrane-stabilizing effects and
enhance TCA toxicity.
Educational Objective:
Lidocaine is the drug of choice for patients with TCA-induced ventricular arrhythmias.
62% of people answered this question correctly;

3/19/2011 Item 18 of 38
usmleworld.com/Step3//qstShow.as 1/1

A 62-year-old Caucasian male is brought to the emergency department (ED) because of pain
and numbness of his right leg. He says, "I was watching TV two hours ago, when this pain
started, and then my leg turned numb." His past medical history is significant for recent acute
anterior wall myocardial infarction, hypertension, and hyperlipidemia. His current medications
are metoprolol, ramipril, simvastatin, and aspirin. He admits to not being regularly adherent
with his medications. He does not smoke, and he occasionally consumes alcohol. His
temperature is 36.7C(98F), blood pressure is 150/90 mm Hg, pulse is 80/min, and
respirations are 14/min. Physical examination reveals a cool, right lower extremity with
delayed capillary refill and significant pulse deficits over the popliteal and dorsalis pedis
arteries, compared to the left lower extremity. Which of the following is the best next step in the
management of this patient?
A. Order Doppler Echocardiography.
B. Proceed with angiography.
C. Refer the patient to the vascular surgeon.
D. Administer IV heparin.
E. Hydrate the patient and observe.

Explanation:
This patient's history and presentation suggest a diagnosis of acute arterial occlusion, which is
presumably caused by arterial thromboembolism. Recent myocardial infarction and atrial
fibrillation are two most common causes of arterial thromboembolism. The single most
important step in the early management of the patients with acute arterial occlusion diagnosed
by history and physical examination is immediate IV heparin therapy followed by continuous
heparin infusion. Heparin therapy will prevent further propagation of thrombus, and inhibit
thrombosis distally in the arterial and venous systems due to low flow and stasis.
(Choices A, B, and C) Immediate anticoagulation therapy should be started in patients with
acute arterial occlusion. It cannot wait until after diagnostic procedures are performed, or after
other therapeutic measures (i.e., surgery) are planned.
(Choice E) Delaying anticoagulation therapy may lead to loss of the limb if the ischemia is
severe.
Educational Objective:
The single most important step in the management of a clinical diagnosis of acute arterial
occlusion is immediate IV heparin therapy followed by continuous heparin infusion.
37% of people answered this question correctly;

3/19/2011 Item 19 of 38
usmleworld.com/Step3//qstShow.as 1/1

A 52-year-old male comes to the emergency department because of chest tightness and
shortness of breath. The symptoms started two hours ago and woke him up from the sleep. He
also complains of lightheadedness and dry cough. He has type 1 diabetes mellitus and mild
intermittent asthma controlled with an albuterol inhaler for symptomatic relief. He was started
on trimethoprim-sulfamethoxazole therapy three days ago for the right big toe infection. He
does not use tobacco, alcohol, or drugs. His daughter, who is seven years old, has had an
upper respiratory infection recently. His temperature is 37.2C (99.0F), blood pressure is
155/92 mmHg, pulse is 100/min and respirations are 22/min. His oxygen saturation is 92% on
2L of nasal canula oxygen. His BMI is 27 kg/m2. Scattered wheezes are heard bilaterally.
Crackles are heard at lung bases. His initial EKG is shown on the slide below.
Two months ago his EKG was reported to be 'normal.' Which of the following is the most
appropriate next step in management?
A. Anticoagulation, aspirin and close observation
B. Cardiac enzymes followed by coronary angiography, if positive
C. Coronary angiography
D. CT angiography of the chest
E. D-dimer followed by CT angiography, if positive

Explanation:
This patient's EKG shows new onset left bundle branch block (LBBB). Left bundle branch block
(LBBB) suggests underlying heart disease and is present in nearly 7 percent of patients with
acute myocardial infarction. The LBBB can mask Q waves and change the early and later
ventricular depolarization stages, leading to secondary ST segment and T wave changes in the

3/19/2011 Item 20 of 38
usmleworld.com/Step3//qstShow.as 1/2
same direction as the QRS complex. The left anterior descending artery (LAD) provides the
main blood supply to the left bundle branch and a complete blockage of the artery can cause a
new LBBB on EKG. There should be a high suspicion of a possible new anterior MI in patients
with a previously normal EKG who present with a new LBBB. The next step should be an
immediate coronary angiography in addition to medical therapy.
(Choice A) Aspirin and anticoagulation are indicated in patients with both ST elevation MI
(STEMI) and non-ST elevation MI (NSTEMI). However, STEMI patients must get immediate
coronary angiography without delay. New onset LBBB in an appropriate clinical setting is
synonymous with STEMI. NSTEMI patients who have evidence of hemodynamic instability,
heart failure, ongoing angina, or persistent arrhythmias must also get immediate coronary
intervention. NSTEMI patients who do not have high risk characteristics should undergo
coronary intervention as early as possible, preferably within 24 hours.
(Choice B) Cardiac enzymes are a sensitive marker for cardiac injury and also predict both
short-term and long-term prognosis in acute MI. They may take two to three hours to be
positive and decrease the therapeutic window for coronary angiography in the setting of an
acute MI.
(Choice D) CT angiography of the chest is currently most useful for diagnosing pulmonary
embolism. New onset right bundle branch block in an appropriate clinical setting should make
you think about large pulmonary embolism. CT angiogram is not indicated in acute coronary
syndrome.
(Choice E) D-dimer is elevated in a variety of conditions but is most useful for ruling out
pulmonary embolism in patients with low pretest probability.
Educational objective:
A new left bundle branch block on an EKG in patients with acute chest pain is suggestive of a
possible acute myocardial infarction and should be managed aggressively with coronary
angiography and medical therapy.
38% of people answered this question correctly;
3/19/2011 Item 20 of 38
usmleworld.com/Step3//qstShow.as 2/2
The following vignette applies to the next 2 items
A 36-year-old man comes to the emergency department because of loss of consciousness.
He was standing on the crowded subway station when he felt lightheaded, had a pounding
sensation in his chest, and then passed out. Upon awakening, he felt short of breath for a little
while and then was "completely fine." He denies any chest pain, confusion, tongue biting, or
urinary incontinence. He has never had similar episodes. The night before, he was at a party
with his friends and consumed some alcohol. Later he was nauseated and had three episodes
of vomiting and one episode of diarrhea. In the emergency department, his blood pressure is
124/68 mm Hg and pulse is 80/min. There is a small laceration above his right eyebrow. Heart
examination shows normal heart sounds with regular rhythm. Lungs are clear to auscultation.
Neurologic examination shows no abnormalities. Electrocardiogram is shown on the slide
below.
Item 1 of 2
Which of the following is the most likely cause of this patient's syncope?
A. Acute coronary syndrome
B. Bradyarrhythmia
C. Orthostatic hypotension
D. Tachyarrhythmia
E. Vasovagal syncope

Explanation:
3/19/2011 Item 21 of 38
usmleworld.com/Step3//qstShow.as 1/3

The electrocardiogram (EKG) presented on the slide above is consistent with Wolff-
Parkinson-White (WPW) pattern, a type of preexcitation syndrome caused by an accessory
pathway. Normal electric impulses pass from the atria to the ventricles via a specialized
conduction pathway, the atrioventricular (AV) node/His-Purkinje system. In patients with WPW
syndrome, there is an extra conduction pathway (also known as bypass tract or accessory
pathway) that directly connects the atria to the ventricles, bypassing the AV junction. The
accessory pathway conducts faster than the AV node and excites the ventricles prematurely,
manifesting as a short PR interval with characteristic delta wave on EKG. QRS complex is
prolonged (>0.12 sec). All these findings are seen on resting EKG.
Although patients with WPW pattern on EKG can be completely asymptomatic, some develop
tachyarrhythmias. Paroxysmal supraventricular tachycardia is the most common arrhythmia; it
is usually a regular, narrow complex tachycardia. If patients with ventricular preexcitation
develop atrial fibrillation, they can conduct down the accessory pathway from the atria to the
ventricles at a very fast rate; therefore, syncope and collapse are common. Progression to life-
threatening ventricular arrhythmias may sometimes lead to sudden death. This patient's
presentation, especially drinking the night before (a well-known precipitant of atrial fibrillation),
definitely raises that possibility.
(Choice A) Acute coronary syndrome (ACS) can present with symptoms, including nausea
and vomiting, palpitations, and/or syncope, without any chest pain. However, this patient has
no known cardiac risk factors for ACS and does not have the characteristic EKG findings,
such as ST segment elevation/depression or T-wave inversions.

3/19/2011 Item 21 of 38
usmleworld.com/Step3//qstShow.as 2/3
(Choice B) Bradyarrhythmias generally arise from either sinoatrial (SA) node dysfunction or
conduction blocks through the AV node and typically present with syncope or presyncope.
However, this patient has a WPW pattern on his EKG and felt palpitations before passing out,
which make a tachyarrhythmia a more likely possibility.
(Choice C) Orthostatic hypotension is defined as a change in blood pressure and heart rate
with positional change and can be due to an inadequate autonomic response, seen in many
autonomic nervous system disorders, or volume depletion. Although this patient had episodes
of vomiting, his presentation is more consistent with WPW syndrome given the palpitations
with the episode and classic EKG.
(Choice E) Vasovagal syncope, also known as neurocardiogenic syncope, can have many
triggering factors, such as stress and pain, and is caused by decreased sympathetic drive
(causing vasodilation) combined with increased parasympathetic response (causing
bradycardia). The EKG findings in this patient make this a less likely cause of the syncope.
Educational objective:
The classic resting electrocardiogram findings in a patient with Wolff-Parkinson-White
syndrome are short PR interval, delta wave, and wide QRS complex. Patients become
symptomatic with tachyarrhythmias. Development of atrial fibrillation can be dangerous as it
can result in ventricular fibrillation and cardiac arrest.
30% of people answered this question correctly;
3/19/2011 Item 21 of 38
usmleworld.com/Step3//qstShow.as 3/3

Item 2 of 2
The patient's serum potassium level is 3.4 mEq/L, blood urea nitrogen level is 12 mg/dL, and
creatinine level is 0.9 mg/dL. Chest x-ray shows no infiltrates or cardiomegaly.
Echocardiogram shows normal left ventricular size and function and no significant valvular
abnormalities. Which of the following is the most appropriate next step in management?
A. Order 24-hour Holter monitoring
B. Refer for catheter ablation
C. Refer for coronary angiography
D. Refer for nuclear stress testing
E. Refer for tilt-table testing

Explanation:
Catheter-ablation therapy is the recommended therapy in symptomatic patients with Wolff-
Parkinson-White (WPW) syndrome. Catheter-ablation therapy has a nearly 90% efficacy rate
and a <5% risk of complications, replacing surgical ablation as the preferred treatment for
accessory pathways.
The incidence of sudden cardiac death in patients with WPW pattern on electrocardiogram
(EKG) is generally low (<1%), but this risk is increased in patients with previous episodes of
tachyarrhythmia. Atrial fibrillation is especially dangerous because impulses can conduct from
the atria to the ventricles at a very fast rate and it might eventually degenerate to ventricular
fibrillation. Since this patient with WPW pattern on EKG had syncope, suggestive of
symptomatic tachyarrhythmia, catheter ablation is recommended.
Catheter ablation in asymptomatic patients with WPW pattern on EKG is controversial.
(Choice A) The 24-hour Holter monitoring (also known as ambulatory monitoring) method
documents continuous recordings of two or three leads on EKG for a given period of time.
Current guidelines recommend ambulatory monitoring for unexplained syncope, near-syncope,
episodic dizziness, and recurrent palpitations.
(Choice C) Coronary angiography is an invasive procedure indicated for management of
acute coronary syndrome (ACS). This patient does not have findings to suggest ACS and
would be better managed with catheter-ablation therapy.
(Choice D) Nuclear stresstesting can be used for diagnosis of coronary artery disease but
has no role in managing patients with WPW syndrome.
(Choice E) Tilt-table testing can be used in the evaluation of recurrent syncope (particularly
vasovagal syncope) and is reserved for patients who have been ruled out for heart disease
(structural or conductive). This patient has conductive evidence of WPW and would not benefit
from a tilt-table test.

3/19/2011 Item 22 of 38
usmleworld.com/Step3//qstShow.as 1/2
Educational objective:
Catheter-ablation therapy is the preferred treatment of Wolff-Parkinson-White syndrome in
patients with symptomatic arrhythmias.
37% of people answered this question correctly;
3/19/2011 Item 22 of 38
usmleworld.com/Step3//qstShow.as 2/2

A 54-year-old heavyset Caucasian man presents to the emergency department complaining of
chest pain that began approximately thirty minutes ago while he was mowing the lawn. He is
panting and sweating heavily. He describes the chest pain as intense pressure, "like a pickup
truck is right on top of me." The pain radiates to his neck, jaw, and left arm. He has never
experienced an episode like this before. He has been diagnosed with diabetes mellitus,
hypertension, hypercholesterolemia, peptic ulcer disease, and onychomycosis. He smokes
one pack of cigarettes per day and consumes 1-3 alcoholic drinks per night. He denies usage
of recreational drugs. Vital signs include temperature of 37.7C (99.9F), blood pressure of
142/90 mm Hg, pulse of 114/min, and respirations of 21/min. There is diffuse mild chest
tenderness to palpation. No edema of the extremities is evident and the remainder of the
examination is unremarkable. He was given oxygen and aspirin while en route to the hospital.
Nitroglycerin is now administered, which provides moderate pain relief. Which of the following
findings on electrocardiogram most strongly indicates the need for thrombolytic therapy?
A. ST depression >1mm in the lateral leads
B. ST elevation >1mm in the lateral leads
C. T wave inversion in the inferior leads
D. Poor R wave progression
E. QT prolongation

Explanation:
Patients stricken with chest pain suggestive of an acute myocardial infarction are eligible for
thrombolytic therapy if they present within 12 (and sometimes 24) hours of symptom onset and
if an electrocardiogram demonstrates ST elevations >1mm in two contiguous leads (Choice
B). It is important to administer nitroglycerin in these patients before performing the
electrocardiogram to rule out coronary vasospasm. Additional candidates for thrombolytic
therapy include individuals with persistent symptoms of new left bundle branch block. There is
no indication that thrombolytic therapy is of benefit in patients with non-ST elevation acute
coronary syndrome. Contraindications for thrombolytic therapy include evidence of active
bleeding, adverse intracerebral history (history of hemorrhage, ischemic stroke, or neoplasm),
systolic blood pressure greater than 180 mm Hg, recent trauma, or drug allergy.
ST depression >1mm (Choice A) is commonly caused by ischemia, "strain," digitalis effect,
and hypokalemia or hypomagnesemia. ST segment depression is not an indication for
thrombolysis unless it is evidence of a posterior or dorsal MI.
T wave inversion (Choice C) commonly occurs in myocardial infarction, angina, and
ventricular hypertrophy. However, T wave inversion rarely indicates ischemia if the QRS is also
negative in those leads.
Poor R wave progression (Choice D) occurs when the R wave in leads V1 through V4
remains the same size or increases very little. Common pathologic causes of poor R wave
progression include LVH, RVH, COPD, anterior infarction, conduction defects, and

3/19/2011 Item 23 of 38
usmleworld.com/Step3//qstShow.as 1/2
cardiomyopathy.
The QT interval is considered prolonged (Choice E) if it measures more than half the R-R
interval, though measurement of the interval is of little clinical significance if the patient is
tachycardic. Common causes of QT prolongation include drugs (eg, antiarrhythmic agents and
tricyclic antidepressants), electrolyte imbalances (eg, hypokalemia), and CNS catastrophes
(eg, stroke, seizure).
Educational Objective:
Patients with chest pain suggestive of an acute myocardial infarction are eligible for
thrombolytic therapy if they present within 12 hours of symptom onset and if an
electrocardiogram demonstrates ST elevations >1mm in two contiguous leads.
82% of people answered this question correctly;
3/19/2011 Item 23 of 38
usmleworld.com/Step3//qstShow.as 2/2

A 45-year-old Caucasian male is brought to the emergency department after losing
consciousness in a shopping mall. He regains consciousness in 5 minutes, and eyewitnesses
say that he had no seizure-like activity during the episode. He does not understand what
happened, and describes no preliminary symptoms such as nausea, vomiting and
lightheadedness. He has never had such an episode before. He is alert and oriented. His past
medical history is insignificant. His blood pressure is 130/90 mmHg while supine and 128/87
mmHg while sitting. The physical examination findings are within normal limits. What is the
best next step in the management of this patient?
A. Electroencephalogram
B. CT scan of the head
C. Electrocardiogram
D. Echocardiogram
E. Holter monitoring

Explanation:
The cause of syncope can be often established by careful history taking, physical examination,
simple lab tests and ECG. Examples of significant diagnostic clues in the history are
preliminary nausea or lightheadedness (neurocardiogenic syncope), convulsions, chest pain,
dyspnea, palpitations, etc. Physical examination findings that also provide important clues are
orthostatic blood pressure changes, abnormal cardiac examination, response to carotid
massage, etc. ECG helps to reveal brady- or tachyarrhythmias and conduction abnormalities
that can explain the symptoms, as well as provide other clues to the presence of organic heart
disease. It has been estimated that this simple evaluation (i.e., history, physical exam and
ECG) establishes the diagnosis in up to 70% of patients. More complex diagnostic studies
are indicated if specific clues are present; otherwise, these are not cost-effective.
(Choices D and E) When suggested by the history, physical examination or ECG findings, an
organic heart disease can be further evaluated by echocardiography, Holter monitoring,
exercise testing and electrophysiological tests. Routine use of these tests in all patients with
syncope is not reasonable.
(Choices A and B) Neurologic tests, including electroencephalogram, brain CT scan, brain
magnetic resonance imaging, and carotid Doppler ultrasound, are frequently obtained in
patients with syncope; however, these are usually of limited value unless specifically
suggested by the history or physical examination.
Educational Objective:
The cause of syncope can be established by careful history taking, physical examination,
simple lab tests and ECG in 70% of cases.
63% of people answered this question correctly;

3/19/2011 Item 24 of 38
usmleworld.com/Step3//qstShow.as 1/1

A 65-year-old male comes to the emergency department with a two week history of
progressive shortness of breath. He has been sleeping in a chair for the last three days
because he cannot lie flat in bed. His past medical history is significant for hypertension,
hyperlipidemia, coronary artery disease, and type 2 diabetes mellitus. He experienced a
myocardial infarction two years ago but refused cardiac catheterization. His current
medications are metoprolol, metformin, losartan, pioglitazone, atorvastatin, glyburide, and low-
dose aspirin. He is a lifelong non-smoker. His blood pressure is 151/82 mmHg and his heart
rate is 72/min, regular in rhythm. His oxygen saturation is 92% on room air. An S3 heart sound
is present on cardiac auscultation. Bilateral crackles are heard on chest examination. There is
2+ symmetric bilateral pitting edema. EKG shows normal sinus rhythm with no acute ST
segment or T wave changes. Cardiac enzymes are negative. Chest X-ray reveals increased
interstitial markings with Kerley B lines and small bilateral pleural effusions. Which of the
following medications most likely contributed to this patient's current condition?
A. Atorvastatin
B. Glyburide
C. Losartan
D. Metformin
E. Pioglitazone

Explanation:
This patient's orthopnea, hypoxia, bilateral crackles on exam, and chest radiograph findings
are all consistent with pulmonary edema. Of the medications listed, the most likely to result in
pulmonary edema is the PPAR- agonist pioglitazone, a member of the thiazolidinedione
class. PPAR- receptors are present in the collecting tubule of the nephron, and stimulation by
pioglitazone results in increased sodium reabsorption. This is the same channel via which
aldosterone mediates its effects on sodium retention. Fluid retention can occur in 4-6% of
patients on thiazolidinediones, but most of these patients likely have underlying heart failure.
This patient's history of a myocardial infarct that was not followed by catheterization, as well as
the presence of an S3 on physical exam, are highly suggestive of underlying heart failure.
Given that the aldosterone channel is involved in mediating the sodium retention, treatment
with an aldosterone inhibitor such as spironolactone may be particularly effective in removing
the excess fluid.
(Choice A) Fluid overload is not associated with statin medications such as atorvastatin. The
most common side effects of statins include hepatotoxicity and muscle injury.
(Choice B) Glyburide is a sulfonylurea medication and as such its most common side effect is
hypoglycemia.
(Choice C) ACE inhibitors or ARBs such as losartan are a key component of heart failure
therapy and would not be expected to induce pulmonary edema in patients with underlying
heart failure.

3/19/2011 Item 25 of 38
usmleworld.com/Step3//qstShow.as 1/2
(Choice D) There is no increased risk of fluid retention with metformin. However, metformin
should be used with caution in patients with heart failure since lactic acidosis can result should
the patient become hemodynamically unstable.
Educational objective:
The thiazolidinedione medications can result in fluid retention via PPAR- agonist effects on
the renal collecting tubule resulting in sodium reabsorption. This effect is usually seen in
patients with underlying heart failure.
60% of people answered this question correctly;
3/19/2011 Item 25 of 38
usmleworld.com/Step3//qstShow.as 2/2

A 70-year-old Caucasian man is brought to the emergency room by his wife after an episode
of syncope while working in his garden. His wife says that he has always been physically
active, but complained of increased fatigability lately. His past medical history is significant for
diabetes mellitus type-2 and gout. His only current medication is glyburide. His blood pressure
is 100/80 mmHg, and heart rate is 90/min. Physical examination is significant for a harsh
crescendo-decrescendo type murmur that is heard at the base of the heart and radiates to the
carotids. ECG reveals left ventricular hypertrophy and secondary ST segment and T wave
changes. Echocardiography shows concentric left ventricular hypertrophy with severe
calcification of the aortic valve. The aortic valve area is 0.78 cm2, and the trans-valvular
gradient is 50 mmHg. Which of the following is the best statement about this patient's
condition?
A. The onset of symptoms minimally, if at all, affects the prognosis.
B. Conservative treatment should be tried before offering an intervention.
C. Balloon valvulotomy is preferred due to low procedural morbidity.
D. Balloon valvulotomy is preferred due to better long-term prognosis.
E. Aortic valve replacement is associated with marked reduction in mortality.

Explanation:
This patient presents with symptomatic aortic stenosis. An aortic valve area of less than 1.0
cm2 is considered severe stenosis. The onset of symptoms markedly affects the prognosis in
patients with aortic stenosis (Choice A); therefore, prompt intervention is recommended in
patients who have cardinal symptoms of aortic stenosis (dyspnea of heart failure, anginal pain,
and syncope). Aortic valve replacement is associated with marked reduction in symptoms and
mortality in patients with symptomatic aortic stenosis, and is considered the treatment of
choice.
(Choices C and D) Balloon valvulotomy is associated with high procedural morbidity and
transient efficacy. It is considered only in selected clinical settings (e.g. patients with
hemodynamic instability or significant comorbidities).
(Choice B) Conservative treatment has little role in the management of patients with aortic
stenosis, because most medical interventions can destabilize the patients (e.g. diuretics or
vasodilators).
Educational Objective:
Aortic valve replacement is associated with marked reduction in symptoms and mortality in
patients with symptomatic aortic stenosis, and is considered the treatment of choice.
64% of people answered this question correctly;

3/19/2011 Item 26 of 38
usmleworld.com/Step3//qstShow.as 1/1

A 72-year-old Caucasian man is brought to the emergency department (ED) after having an
episode of syncope during a family meeting. His granddaughter states that in the last couple of
months he has been having frequent episodes of dizziness, which are sometimes
accompanied by confusion, and are not related to physical activity or changes in position. The
patient lives at home, has a history of coronary artery disease and hypertension, and does not
use tobacco or drink alcohol. His medications include aspirin, enalapril, metoprolol, and
isosorbide mononitrate. While in the ED, his symptoms are progressively getting better. The
physical examination reveals a heart rate of 45/min, a blood pressure of 90/50 mmHg, and
respiratory rate of 14/min. No other important clinical findings are noticed. He is fully alert and
oriented, and there are no focal neurologic findings. An EKG shows a constant PR interval of
0.18 ms and a normal QT interval, but the QRS complexes are lost on every third beat. Which
of the following is the most appropriate course of action?
A. Administer intravenous atropine.
B. Observe and admit.
C. Place an external pacemaker.
D. Place a transvenous pacemaker.
E. Place the patient on telemetry or Holter monitor for 24-48 hours.

Explanation:
This patient has Mobitz II, second-degree atrioventricular (AV) Block. In comparison to Mobitz
I, second-degree AV Block (Wenckebach's), the PR does not prolong progressively, but QRS
complexes are suddenly lost. Mobitz II AV Block can cause dizziness, episodes of syncope, or
transient altered mental status, thus explaining the symptoms of the patient. Because Mobitz II
can progress to third-degree AV Block, it needs to be managed with a permanent pacemaker.
Even if the patient is asymptomatic, the AHA/ACC (American Heart Association/American
College of Cardiology) consensus advises the use of a permanent pacemaker inserted
through a venous access.
(Choice B) It would be unsafe to manage a patient with this condition without a pacemaker.
Observation is not an option.
(Choice A) Atropine will increase the heart rate, worsening the heart block. Atropine is usually
indicated for sinus bradycardia; in that case, QRS complexes will not be lost.
(Choice C) An external pacemaker will only be a temporary solution and may require sedation
of the patient. It is generally used for acute rate control, not for this chronic condition.
(Choice E) A Holter monitor or telemetry is not needed at this point to evaluate the cause of
the syncope if Mobitz II is treated appropriately.
Educational Objective:
Mobitz II second-degree AV Block is diagnosed when the EKG shows bradycardia with
preservation of the PR interval, accompanied by periodic loss of QRS complexes. This

3/19/2011 Item 27 of 38
usmleworld.com/Step3//qstShow.as 1/2
condition is an indication for permanent cardiac pacemaker insertion.
41% of people answered this question correctly;
3/19/2011 Item 27 of 38
usmleworld.com/Step3//qstShow.as 2/2

The following vignette applies to the next 2 items
A 27-year-old woman is brought to the emergency department because of a 4-hour history of
substernal chest pain. She has not had nausea, vomiting or shortness of breath. She smoked
crack cocaine shortly prior to her episode of chest pain. She has a history of intravenous drug
use. She has no family history of premature coronary artery disease. She does not take any
medications and has no known drug allergies. Her temperature is 37.8 C (100 F), blood
pressure is 204/102 mm Hg, pulse is 102/min, and respirations are 18/min. She appears thin,
anxious, and agitated. Cardiac examination shows normal heart sounds. The lungs are clear
to auscultation. An ECG shows a sinus tachycardia but is otherwise unremarkable. An x-ray
film of the chest shows no abnormalities, and the initial set of cardiac enzymes is normal.
Item 1 of 2
Which of the following is the most appropriate next step in management?
A. Anticoagulation with low-molecular-weight heparin
B. Cardiac catheterization
C. High-dose ibuprofen
D. Intravenous lorazepam
E. Intravenous metoprolol

Explanation:
Chest pain in patients who have recently taken cocaine is common and caused by several
factors, including increased myocardial oxygen demand, vasoconstriction, and a
prothrombotic state (secondary to platelet activation). Patients who have recently taken
cocaine have an increased rate of myocardial infarction and aortic dissection, and the risk of
these complications is highest in the first hours after drug use.
For the most part, management of patient with cocaine related chest pain is similar to chest
pain in the general population. One key difference is that cocaine-related chest pain patients
should be treated early with benzodiazepines. Benzodiazepines (e.g. lorazepam) can
decrease anxiety and agitation associated with cocaine use. Because the cardiovascular
manifestations of cocaine use are intimately associated with its neuropsychiatric effects,
benzodiazepines may indirectly decrease myocardial ischemia by resolving associated
hypertension and tachycardia.
(Choice A) Anticoagulation with heparin should only be performed if there is evidence of
myocardial infarction in the form of positive cardiac biomarkers or abnormal ECG findings.
(Choice B) Emergent cardiac catheterization should be performed if the patient has evidence
of ST elevation on ECG.
(Choice C) Non-steroidal anti-inflammatory drugs (NSAIDs) such as ibuprofen should only be

3/19/2011 Item 28 of 38
usmleworld.com/Step3//qstShow.as 1/2
used in cases of chest pain with a suspected musculoskeletal etiology. Musculoskeletal chest
pain is typically a diagnosis of exclusion.
(Choice E) Although beta-blockers are used in the management of myocardial infarction in
the general population, they should be avoided in patients with recent cocaine use to prevent
coronary vasospasm caused by unopposed alpha stimulation.
Educational objective:
Management of cocaine-related chest pain is similar to management of chest pain in the
general population with a few key exceptions. Benzodiazepines should be given early to these
patients to decrease neuropsychiatric manifestations of cocaine use and thereby improve
hypertension and tachycardia. In addition, beta-blockers should be avoided in patients with
cocaine-related chest pain to avoid causing coronary vasospasm.
55% of people answered this question correctly;
3/19/2011 Item 28 of 38
usmleworld.com/Step3//qstShow.as 2/2

Item 2 of 2
In spite of initial treatment measures, the patient continues to have chest pain. Her blood
pressure is 216/112 mm Hg and pulse is 103/min. Which of the following is the most
appropriate pharmacotherapy for this patient's hypertension?
A. Intravenous loop diuretics
B. Intravenous metoprolol
C. Intravenous morphine
D. Intravenous phentolamine
E. Oral angiotensin converting enzyme inhibitors

Explanation:
Benzodiazepines often improve tachycardia and hypertension in patients with cocaine-related
chest pain to the degree that myocardial ischemia is lessened. However, this patient is
persistently hypertensive and more aggressive blood pressure treatment is therefore
indicated. Phentolamine is an alpha-antagonist which is a frequently used option for
management of persistent hypertension after cocaine use. Phentolamine also decreases
coronary vasospasm, which may be a precipitating factor for myocardial ischemia in patients
with recent cocaine use. Phentolamine may be helpful for management of hypertensive
emergency in patients with any cause of increased sympathetic activity, which would also
include pheochromocytoma. Alternative options for blood pressure management in this case
would include nitroglycerine and nitroprusside.
(Choice A) Loop diuretics are typically used more as a treatment for volume overload as
opposed to hypertension, and there is no evidence that this patient is fluid overloaded.
(Choice B) Beta-blockers such as metoprolol should be avoided in patients with recent
cocaine use because they can result in unopposed alpha effects with subsequent coronary
vasospasm.
(Choice C) Benzodiazepinesnot opioidsare typically the treatment of choice for
managing the neuropsychiatric effects of cocaine use.
(Choice E) ACE inhibitors may be indicated before discharge from the hospital if the patient
has had a myocardial infarction, but they are not used to acutely lower blood pressure during
initial evaluation.
Educational objective:
In patients with cocaine-related chest pain, more aggressive management of hypertension is
indicated when blood pressure does not improve with benzodiazepines. Phentolamine is
particularly effective and additionally decreases coronary artery vasospasm, although
nitroprusside and nitroglycerine are also reasonable options. Beta-blockers should be
avoided.

3/19/2011 Item 29 of 38
usmleworld.com/Step3//qstShow.as 1/2
56% of people answered this question correctly;
3/19/2011 Item 29 of 38
usmleworld.com/Step3//qstShow.as 2/2

A 62-year-old Caucasian man comes to the emergency department (ED) and complains of
crushing substernal chest pain. The pain started one hour ago while he was shoveling snow on
his driveway. At the onset of his pain, he took nitroglycerin sublingually, but the pain did not get
any better and he was taken to the ED by his son. He has a history of coronary artery disease,
hypertension, hyperlipidemia, and osteoarthritis. He is on daily aspirin, metoprolol, lisinopril,
atorvastatin, and naproxen as needed. In the ED, his electrocardiogram reveals a normal
sinus rhythm with no acute ST-T wave changes. He is given morphine for his pain, started on
nitroglycerin drip, and admitted to the telemetry floor. The next day, an adenosine thallium
stress test reveals a moderate size reversible perfusion defect in the lateral wall of the left
ventricle. Which of the following coronary vessels is most likely to have a significant occlusion?
A. Left circumflex coronary artery
B. Left anterior descending coronary artery
C. Left main coronary artery
D. Right coronary artery

Explanation:
In all patients with known or suspected coronary artery disease, exercise or pharmacological
stress testing with radionuclide myocardial perfusion imaging (thallium or technetium-99m) is
useful for confirming the diagnoses and for risk stratification. The pharmacologic stress agents
(adenosine and dipyridamole) act by producing coronary vasodilatation and increasing the
coronary flow rate and velocity. In normal coronary vessels, the resulting vasodilation increases
the blood flow; however, in areas with severe stenosis, there is already a compensatory
microvascular dilatation at rest to maintain normal blood flow, so no further increase in the flow
occurs. The resulting heterogenous blood flow due to the stenotic or occluded region is
detected by radionuclide imaging studies as a perfusion defect. After establishing the
presence of a defect, the next step is to localize the affected myocardial region in order to
identify the culprit vessel.
All coronary arteries supply a specific region of the ventricle. In this case, the patient's
adenosine thallium stress test revealed a perfusion defect in the lateral wall of the left ventricle.
The culprit vessel is most probably the left circumflex artery, which runs laterally in the left
atrioventricular groove to supply the lateral and posterolateral parts of the left ventricle.
(Choices B and C) The left main coronary artery arises from the left coronary ostium and
branches off into the left anterior descending (LAD) artery and left circumflex artery. The LAD
artery runs along the anterior interventricular groove and supplies the anterior wall of the left
ventricle. Diagonal branches from the LAD artery also run downward to supply the
anterolateral wall of the left ventricle.
(Choice D) The right coronary artery runs in the right atrioventricular groove to supply the right
ventricle and inferoposterior walls of the left ventricle.
Educational Objective:

3/19/2011 Item 30 of 38
usmleworld.com/Step3//qstShow.as 1/2
The left circumflex coronary artery runs in the left atrioventricular groove to supply the lateral
wall of the left ventricle.
63% of people answered this question correctly;
3/19/2011 Item 30 of 38
usmleworld.com/Step3//qstShow.as 2/2

A 32-year-old man comes to the emergency department (ED) and complains of severe chest
discomfort for the past two hours. Approximately two hours after dinner, he noticed a mild
substernal chest discomfort, which has progressed in severity. The pain does not radiate or
change in intensity with respirations or any change in position. He has never had such chest
discomfort in the past. He does not have any significant past medical history. His father died of
a "heart attack" at the age of 59 years, and his mother died of congestive heart failure at the
age of 75 years. His temperature is 37.2C(99F), blood pressure is 130/85 mmHg, heart rate
is 110/min, and respirations are 16/min. Physical examination reveals a young, well-built man
in acute distress due to the chest discomfort. The lungs are clear to auscultation, and there are
no murmurs or adventitious heart sounds on cardiac auscultation. His abdominal examination
is unremarkable. A stat electrocardiogram in the ED reveals sinus tachycardia with no other
acute abnormalities. The patient is given sublingual nitroglycerin, which relieves his discomfort
completely. What is the next step in the management of this patient?
A. Intravenous fluids
B. Barium swallow
C. Cardiac enzymes
D. Echocardiogram

Explanation:
The patient in the above vignette is most likely suffering from an acute coronary syndrome. The
presence of persistent substernal chest discomfort in a male patient that gets relieved by
sublingual nitroglycerin is highly suspicious for the presence of an acute coronary syndrome.
All patients with classic symptoms and presentation should be presumed to have acute
coronary syndrome until proven otherwise, even if the patient does not have any significant risk
factors for the development of coronary artery disease.
The diagnosis of acute coronary syndrome is usually made by a combination of chest pain,
EKG abnormalities and elevation in the serum markers of myocardial injury. The chest pain or
discomfort can often be atypical at presentation, and EKG abnormalities may be nonspecific
or even absent in patients with acute coronary syndrome; therefore, the diagnosis of acute
myocardial infarction in patients with classical clinical presentation and normal
electrocardiographic findings is usually made by measuring the levels of serum markers of
acute myocardial injury. This involves serial measurements of serum troponin and creatine
kinase with its isoforms. A typical rise and fall in the levels of serum biomarkers in the
presence of symptoms of chest discomfort is highly suggestive of an acute or recent
myocardial infraction.
(Choice A) There is no indication for the use of intravenous fluids in this patient.
(Choices B and D) A barium swallow or an echocardiogram is not necessary for this patient.
Educational Objective:
Serial evaluation of serum markers of myocardial injury, especially troponin and creatine

3/19/2011 Item 31 of 38
usmleworld.com/Step3//qstShow.as 1/2
kinase, should be performed in all patients presenting with signs and symptoms of cardiac
ischemia.
67% of people answered this question correctly;
3/19/2011 Item 31 of 38
usmleworld.com/Step3//qstShow.as 2/2

A 64-year-old male comes to the emergency department with face swelling. He has never had
such symptoms before. He has a past medical history of type 2 diabetes mellitus,
hypertension, and coronary artery disease. He had a myocardial infarction seven years ago.
He was recently diagnosed with congestive heart failure with low left ventricular ejection
fraction. His current medications are ramipril, metoprolol, furosemide, spironolactone, aspirin,
glyburide, and simvastatin. Physical examination reveals prominent lip and tongue swelling
without stridor. The patient's symptoms subside within 24 hours after he is admitted and
appropriate treatment instituted. Which of the following is the best statement concerning the
management of this patient?
A. Lisinopril can be substituted for ramipril because there is little cross-reactivity
between different ACE inhibitors.
B. Losartan can be substituted for ramipril because angiotensin receptor blockers do
not affect the kinin system and they are not associated with angioedema.
C. Losartan can be substituted for ramipril because there is little cross-reactivity
between ACE inhibitors and angiotensin receptor blockers.
D. Ramipril can be continued because the risk of recurrent angioedema is very low and
the benefits outweigh the risk.

Explanation:
Given that this patient was just diagnosed with congestive heart failure, it is probable that his
medication regimen has recently been adjusted. The facial swelling with prominent lip and
tongue swelling described in this vignette is consistent with angioedema. Angioedema is
usually self-limited lasting 24-48 hours, however if the larynx becomes involved it can be fatal.
The most likely etiology of this patient's angioedema is ramipril, which is an ACE inhibitor that
has a 0.3% incidence of angioedema. Half of all cases of ACE inhibitor-related angioedema
occur in the first few weeks of therapy, although it can occur even in patients who have been on
the culprit medication for several years. Overall, as many as 20% of people are unable to
tolerate ACE inhibitors secondary to side effects, although in most cases the reason is cough,
not angioedema.
The risk of recurrent angioedema is significant and the medication should probably not be
used in the future. Angiotensin receptor blockers (ARBs) are generally used in this situation as
there is a low rate of cross reaction between the medication classes and patients switched to
ARBs usually tolerate them well. A randomized controlled clinical trial specifically addressed
the effectiveness of ARBs in the group of patients unable to tolerate ACE inhibitors and found
that there was a modest reduction in cardiovascular death, myocardial infarction, and stroke.
(Choice A) The tendency of ACE inhibitors to cause angioedema is inherent to their
mechanism of action via increased bradykinin levels. All ACE inhibitors are equally likely to
result in angioedema, and therefore switching to another medication in the same class is not
recommended.
(Choice B) While it is true that ARBs do not affect the kinin system, there is still an

3/19/2011 Item 32 of 38
usmleworld.com/Step3//qstShow.as 1/2
association with angioedema. A small number of patients who develop angioedema from
ACE inhibitors may also have a reactions to ARBs although the cross reactivity rate is rather
low.
(Choice D) An ACE inhibitor should not be continued after angioedema has occurred.
Educational objective:
Angioedema is a possible side effect of ACE inhibitors that occurs in 0.3% of patients.
Patients with this reaction should be switched to an angiotensin receptor blocker (ARB).
Though the risk of angioedema with ARBs is not zero, their cross reactivity rate with ACE
inhibitors is low.
31% of people answered this question correctly;
3/19/2011 Item 32 of 38
usmleworld.com/Step3//qstShow.as 2/2

A 29-year-old Caucasian male presents to the emergency department with sudden onset
chest pain. He says, 'Doc, I have this squeezing in my chest that does not go away.' He has
never had such symptoms before. His past medical history is insignificant. He smokes two
packs of cigarettes daily, and consumes alcohol occasionally. He uses cocaine and had 'one
shot' several hours ago. His family history is significant for a stroke in his father and a heart
attack in his mother. His blood pressure is 140/90 mmHg, pulse is 112/min, temperature is
36.7C (98F) and respirations are 24/min. ECG shows 2 mm ST segment elevation in leads I,
aVL, V4-V6. The patient receives oxygen per mask, nitroglycerin, aspirin and diazepam.
Twenty minutes later, he still complains of chest pain, and his ECG is unchanged. What is the
best next step in the management of this patient?
A. Administer streptokinase
B. Administer phentolamine and observe
C. Administer metoprolol
D. Coronary angiography
E. Reassurance and observation

Explanation:
Myocardial ischemia is a well-known complication associated with cocaine use. The main
factor that leads to myocardial ischemia is vasospasm, although other mechanisms (e.g.,
increased oxygen demand) also play a role. Cocaine-associated vasospasm may lead to
coronary artery thrombosis, which cannot be reversed by vasodilator agents. In patients with
transmural ischemia (ST elevation), the initial treatment regimen consists of nitrates (or
calcium channel blockers), aspirin and benzodiazepines. If the patient does not promptly
improve with these drugs, immediate coronary angiography should be performed to check for
any coronary thrombus.
(Choice A) If a coronary thrombus is detected, the next step in management is coronary
angioplasty.
(Choice B) Phentolamine is a reasonable choice; however, in this case with transmural
ischemia, precious time may be lost if phentolamine is given instead of performing coronary
angiography, especially if an obstructing thrombus is present.
(Choice C) Agents with beta-blocking activity are not recommended because these can
exacerbate the condition and may even be associated with worse outcomes.
(Choice E) Cocaine-associated myocardial infarction leads to the same complications as
'traditional' myocardial infarction; therefore, a quick and appropriate response to save the
jeopardized myocardium is important.
Educational Objective:
Immediate coronary angiography is performed in patients with cocaine-induced transmural
ischemia to check for any coronary thrombus.

3/19/2011 Item 33 of 38
usmleworld.com/Step3//qstShow.as 1/2
35% of people answered this question correctly;
3/19/2011 Item 33 of 38
usmleworld.com/Step3//qstShow.as 2/2

A 52-year-old man comes to the emergency department (ED) after an episode of severe
substernal chest pressure lasting 30 minutes, which occurred one hour ago. His symptoms
started when he was helping his sister move into her new house. He describes associated
mild dizziness and shortness of breath. He has had similar episodes with exertion in the past,
but in those instances the discomfort lasted only a few minutes at a time. With the present
episode, the patient took 3 nitroglycerin tablets sublingually and his pain eventually subsided.
He is currently pain free. His daily medications include metoprolol, lisinopril, atorvastatin, and
aspirin. On physical examination, his blood pressure is 152/93 mmHg and his heart rate is
105/min. His lungs are clear to auscultation bilaterally. Cardiac examination reveals a
prominent point of maximal impulse that is displaced downwards and to the left side.
Auscultation reveals regular, normal heart sounds with an audible S4. There are no audible
murmurs. The rest of the physical examination is within normal limits. An electrocardiogram in
the ED reveals sinus tachycardia and voltage criteria for left ventricular hypertrophy. The
patient's initial troponin I level is normal. What is the most appropriate next step in his
management?
A. Admit the patient and measure serial cardiac enzymes
B. Offer immediate cardiac catheterization
C. Order a radionuclide perfusion scan
D. Order a stress echocardiogram
E. Reassure and discharge with close follow-up

Explanation:
This patient's description of chest pain is concerning for acute coronary syndrome. He
describes a history of intermittent chest pain with exertion in the past that is suggestive of
angina. The present acute episode of chest pain lasting 30 minutes with associated shortness
of breath and dizziness, which resolved with nitroglycerin, is also suspicious for a cardiac
etiology for his pain, possibly myocardial infarction. His initial EKG does not demonstrate
findings suspicious for cardiac ischemia such as ST-segment changes, and his initial cardiac
troponin level is normal. However, it may take up to 6 hours for cardiac troponin levels to
become elevated. Given that this patient's episode of chest pain was only one hour ago, a
single normal troponin level cannot be used to exclude myocardial infarction. He falls into the
intermediate to high risk category for myocardial infarction and should be admitted to the
hospital for measurement of serial cardiac enzymes.
(Choice B) Immediate cardiac catheterization with revascularization is only indicated in
patients having an ST-elevation myocardial infarct (STEMI). While there is no evidence of ST-
elevation on this patient's EKG, repeat EKG should be considered if he has recurrent chest
pain after being admitted.
(Choice C) This patient may eventually need stress testing to exclude the presence of
underlying coronary artery disease, but serial troponins are needed first to exclude the
presence of a myocardial infarction.

3/19/2011 Item 34 of 38
usmleworld.com/Step3//qstShow.as 1/2
(Choice D) This patient will likely need stress testing with either a stress echocardiogram or
radionuclide perfusion study, but this should be done after serial troponins are negative.
(Choice E) Outpatient management can be considered if there is an alternative explanation
for the patient's chest pain or if the patient is young with a very low risk of coronary artery
disease. However, this patient has a number of cardiac risk factors and no clear alternative
etiology for his pain.
Educational objective:
At least two negative troponins spaced 6 hours apart are need to exclude myocardial
infarction. Outpatient management of chest discomfort should only be considered in patients
where there is a clear alternative explanation for the chest pain or in very young patients where
suspicion for coronary artery disease is low. Stress testing should only be done after
myocardial infarction has been excluded with serial troponins.
59% of people answered this question correctly;
3/19/2011 Item 34 of 38
usmleworld.com/Step3//qstShow.as 2/2

A 70-year-old white male with a past medical history of coronary artery disease, myocardial
infarction, congestive heart failure, hypertension, and type II diabetes comes to the emergency
department after passing out early this morning. He was sitting in his chair, watching TV, when
he had a brief episode of loss of consciousness, followed by complete recovery. He denies
any headaches, nausea, pallor, or diaphoresis. His friend who was with him at that time said
that he did not observe any limb movements during the episode. There was no soiling of his
clothes by urine or feces. His medications include furosemide, potassium, metformin,
glipizide, atenolol, enalapril, simvastatin and aspirin. He lives alone in his house, and is a non-
smoker and non-alcoholic. His temperature is 36.7C(98F), blood pressure is 130/86 mm Hg,
pulse is 70/min, and respirations are 16/min. Neurological exam is non-focal. There is no
postural drop of blood pressure. Chest x-ray, EKG, CBC, serum chemistries and set of
cardiac enzymes are within normal limits. Which of the following is the most appropriate next
step in the management of this patient?
A. Admit the patient and do continuous EKG monitoring for 24 hours.
B. Admit the patient and do coronary angiography.
C. Send the patient home on Holter monitor.
D. Do CT scan of head without contrast.
E. Admit the patient and do EEG.

Explanation:
Based on the patient's past medical history and presentation, cardiac arrhythmia is the most
likely cause of his syncope. Inpatient continuous cardiac monitoring is the most appropriate
next step in the management of this patient. An acute coronary event should be ruled out with
serial cardiac enzymes, and an echocardiogram is needed to evaluate left ventricular function.
Coronary revascularization is needed if evidence of coronary ischemia exists.
(Choice B) This patient has a normal EKG, normal cardiac enzymes, and no chest pain.
Immediate coronary angiography is not needed.
(Choice C) The patient lives by himself. Sending him home without identifying the cause of his
syncope is inappropriate because he will then be at high risk of falling and injuring himself.
(Choice D) The patient does not give any history of headache or focal neurological signs;
therefore, a CT scan of the head is also inappropriate.
(Choice E) The patient does not have any abnormal movements, urinary or fecal incontinence
or postictal state; therefore, the possibility that seizures could have caused the syncope is
highly unlikely.
Educational Objective:
Patients with multiple risk factors for developing cardiac arrhythmias need proper evaluation
for possible implantable cardiac defibrillator placement to prevent sudden cardiac death.

3/19/2011 Item 35 of 38
usmleworld.com/Step3//qstShow.as 1/2
53% of people answered this question correctly;
3/19/2011 Item 35 of 38
usmleworld.com/Step3//qstShow.as 2/2

A 42-year-old male is brought to the emergency department with a four-hour history of chest
discomfort. He describes tightness in his chest and shortness of breath. He has never had
similar symptoms before. He experienced a mild respiratory infection soon after returning from
a trip to Paris one week ago. He takes no medications and does not use illicit drugs. He has
smoked one pack of cigarettes per day for 20 years. His blood pressure is 152/99 mmHg and
his pulse is 80/min. His BMI is 32 kg/m2. His initial EKG is shown on the slide below.
Which of the following is the most appropriate next step in management?
A. Cardiac enzymes, serial EKGs and observation
B. Coronary angiography
C. CT angiography of the chest
D. D-dimer followed by CT angiography, if positive
E. Non-steroidal anti-inflammatory agents and reassurance

Explanation:
This patient's EKG shows classic anterior wall ST elevation myocardial infarction (STEMI).
STEMI indicates complete transmural occlusion of a major coronary artery and is a medical
emergency. The initial treatment includes relief of ischemic pain with nitroglycerin or morphine,
assessing and stabilizing hemodynamic status, beta blockers, antithrombotic therapy, and
reperfusing the artery with either thrombolytics or percutaneous coronary intervention (PCI).
Coronary angiography is the preferred treatment if the treating facility is well equipped and can
perform the procedure in the first 90 minutes of patient contact or within 12 hours from the
symptom onset. Prompt reperfusion restores blood flow, salvages the myocardium, and
decreases mortality. Thrombolytic therapy is inferior to PCI, but is appropriate for centers that

3/19/2011 Item 36 of 38
usmleworld.com/Step3//qstShow.as 1/2
cannot get the patient to a coronary catheterization lab in time. It should be administered within
6 hours of symptom onset.
(Choice A) Cardiac enzymes may take up to two to three hours to be elevated after the onset
of symptoms and might be positive in this patient. However, doing serial EKGs and observing
the patient without coronary intervention is not the standard of care. This strategy along with
medical therapy can be employed in certain patients with NSTEMI.
(Choice C) CT angiography is a noninvasive method to visualize coronary arteries and is a
reasonable strategy in symptomatic patients with intermediate risk for coronary disease (e.g.,
after equivocal stress test results). Its role in acute coronary syndrome and acute MI has not
been established.
(Choice D) Elevated D-dimer results from recent or ongoing intravascular blood coagulation
and can be elevated in a variety of conditions. It is neither specific nor sensitive for a STEMI.
(Choice E) Non-steroidal anti-inflammatory agents (NSAIDs) are indicated in patients with
acute pericarditis. These agents (other than aspirin) can interfere with the beneficial effects of
anti-platelet therapy, increase cardiovascular events, and worsen or cause heart failure. They
are currently not indicated in patients with STEMI.
Educational objective:
STEMI is a medical emergency that requires prompt reperfusion therapy with either
thrombolytic therapy or PCI. The goals of treatment are to salvage the myocardium and
decrease mortality.
57% of people answered this question correctly;
3/19/2011 Item 36 of 38
usmleworld.com/Step3//qstShow.as 2/2

A 33-year-old female presents to the emergency department after she passed out at a local
bank. It was very busy in the bank; she was standing in a queue for a long time and was
waiting for her turn, when she lost consciousness for a few seconds. She did not sustain any
injuries. She was pale, diaphoretic and nauseous prior to and after the episode. Her friend,
who witnessed the episode, said that she did not have any abnormal jerky movements of her
extremities. She did not have any bowel or bladder incontinence during the episode. She
denies any palpitations prior to the syncope. She does not have any significant past medical
history, except for migraines, for which she takes ibuprofen. Physical examination is normal.
Her blood pressure is 110/70 mm/Hg while supine, and 108/70 mmHg while standing. EKG
shows a normal sinus rhythm. Which of the following is the most likely cause of her syncope?
A. Neurocardiogenic
B. Cardiovascular
C. Autonomic neuropathy
D. Neurological
E. Seizures

Explanation:
The above patient is most likely suffering from neurocardiogenic or vasovagal syncope. This is
also called 'common faint'. Vasovagal syncope is associated with prodromal symptoms of
nausea, lightheadedness, pallor and diaphoresis. It usually occurs when the patient is erect,
and recovery occurs in the supine position. Patients are usually young and otherwise healthy.
Precipitating events include prolonged standing, exertion, venipuncture, or a painful stimulus.
(Choice B) Cardiovascular syncope is due to arrhythmia or obstructive lesions such as aortic
stenosis or HOCM. Syncope may be associated with a prodrome of palpitations. Arrhythmias
usually occur without warning, with the patient in a supine or resting position, and lasts for a
few seconds. Patients will have risk factors such as left ventricular dysfunction, prior
myocardial infarction or conduction system disease. Physical examination may show the
murmur of aortic stenosis or HOCM. EKG/Holter monitoring may detect an arrhythmia.
(Choice C) Patients with autonomic neuropathy have postural hypotension. They develop
syncope when they change their posture from supine to erect.
(Choice D) Neurological syncope is due to atherosclerotic disease of the cerebral circulation,
and is associated with a history of focal neurological deficits.
(Choice E) A seizure may mimic syncope; however, patients usually have a history of tonic-
clonic movements, urinary or fecal incontinence, and a postictal state. J erky movements
similar to tonic-clonic movements can occur in any patient with syncope, but the postictal
confusion is very unlikely with other conditions except seizures.
Educational Objective:
Vasovagal syncope is associated with prodromal symptoms of nausea, lightheadedness,

3/19/2011 Item 37 of 38
usmleworld.com/Step3//qstShow.as 1/2
pallor and diaphoresis. Precipitating events include prolonged standing, exertion,
venipuncture, or a painful stimulus.
64% of people answered this question correctly;
3/19/2011 Item 37 of 38
usmleworld.com/Step3//qstShow.as 2/2

A 50-year-old Caucasian man is brought to the emergency department by the paramedics
after an episode of chest pain of sudden onset. He was clearing snow on a cold winter day,
when he suddenly felt severe chest pressure over the left side. The pain was constant,
radiating to the left jaw, and was not relieved by rest or nitroglycerine spray in the ambulance.
He also complains of dizziness and fatigue. He has a history of controlled hypertension. He
has no other significant past medical or family history. He smokes occasionally, especially
over the weekends. He denies any alcohol intake. His temperature is 36.2C (97F), blood
pressure is 83/52 mmHg, heart rate is 34/min, and respiratory rate is 16/min. His stat
electrocardiogram reveals a sinus bradycardia with a 3 mm ST segment elevation in leads II, III
and AVF. Which of the following is the most appropriate initial step in the management of this
patient?
A. IV normal saline
B. Intravenous Dobutamine
C. Intravenous atropine sulfate
D. Temporary pacemaker insertion
E. Synchronized cardioversion

Explanation:
The patient has developed sinus bradycardia as a complication of acute inferior wall MI. Sinus
bradycardia is seen in approximately 15 to 25% of patients with an acute inferior wall MI. The
heart rate is primarily controlled by the intrinsic pacemaker of the heart, and is located at the
SA node. Blood is supplied to the SA node by the right coronary artery in 60 to 70% of the
population; hence, its interruption in patients with an inferior wall MI can cause sinus
bradycardia. Aside from this, sinus bradycardia can also be caused by an increased vagal
tone during and after an acute MI, and by a reperfusion arrhythmia after thrombolysis.
Sinus bradycardia after an acute inferior wall MI is usually transient, and typically resolves
within the next 24 hours. Patients are usually asymptomatic and require no further treatment;
however, severe bradycardia can result in a low cardiac output, thereby causing hypotension
and subsequent hemodynamic compromise, as well as worsening of ongoing ischemia in the
setting of acute myocardial infarction with reduced ventricular function. If treatment becomes
necessary, intravenous atropine sulfate (0.6 to 1.0 mg) can be given to reverse the
bradycardia and improve the cardiac output and hypotension.
(Choice A) Intravenous fluids are usually required in patients with right ventricular infarction,
which causes reduced left ventricular filling pressures and hypotension. The primary cause of
hypotension in this patient is severe bradycardia resulting from an acute inferior wall MI.
(Choice B) Dobutamine is a beta-1 agonist that can be used as an inotropic agent in patients
with cardiogenic shock; however, since it can produce vasodilation and further hypotension, it
should not be given to hypotensive patients.
(Choice D) Sinus bradycardia after an acute MI is usually transient and resolves within the

3/19/2011 Item 38 of 38
usmleworld.com/Step3//qstShow.as 1/2
next 24 hours. Temporary cardiac pacing is required if the bradycardia is persistent and
symptomatic in spite of IV atropine use.
(Choice E) Synchronized cardioversion has no role in the management of patients with
severe bradycardia.
Educational objective:
Although symptomatic severe bradycardia after an acute inferior wall MI is usually transient, it
should be treated with intravenous atropine sulfate.
35% of people answered this question correctly;
3/19/2011 Item 38 of 38
usmleworld.com/Step3//qstShow.as 2/2

An imaging study detects a 3.5 cm abdominal aortic aneurysm in a 60-year-old male. He has
no associated symptoms. His past medical history is significant for hypertension, type 2
diabetes, hypercholesterolemia and hypothyroidism. His medications include aspirin,
metformin, glipizide, hydrochlorothiazide, lisinopril, low dose atorvastatin and levothyroxine. He
has smoked 1-2 packs of cigarettes daily for the past 40 years, and he drinks 1-2 glasses of
wine daily. On physical examination, his blood pressure is 160/90 mmHg and his pulse is
80/min. Heart sounds are normal and his lungs are clear. Laboratory studies show:
Serum creatinine 1.7 mg/dL
Low density lipoprotein (LDL) 150 mg/dL
HbA1C 8.5%
Serum TSH 3.0
Which of the following interventions would most reduce the likelihood of his aortic aneurysm
enlarging?
A. Better control of hypertension
B. Better control of hypercholesterolemia
C. Better control of diabetes
D. Smoking cessation
E. Alcohol cessation

Explanation:
For abdominal aortic aneurysms (AAAs) greater than about 5 cm in diameter, surgery is
typically recommended. This cutoff was chosen because the risk of rupture increases
dramatically at this diameter. For patients with smaller AAAs, periodic imaging should be
done to assess for aneurysm growth. Rapid growth itself can be an indication for surgical
repair. Smoking is a major risk factor for aneurysm formation, enlargement, and rupture.
Continued smoking increases the rate of aneurysm growth by about 25%. In most patients,
atherosclerosis is the underlying precipitant for AAA formation, but altering other
atherosclerotic risk factors has not been demonstrated to have as much impact as smoking
cessation. Thus, smoking cessation is the medical intervention with the greatest potential
impact for this patient.
(Choice A) Hypertension is a risk factor for atherosclerosis and aneurysm formation.
Hypertension in patients with AAA should be treated with beta-blockers.
(Choice B) Hypercholesterolemia is a risk factor for AAA development and should be treated,
but will not have as much impact on AAA progression as smoking cessation.
(Choice C) Improved diabetes control will improve the patients overall survival, but will not
have much impact on progression of AAA size as smoking cessation.
(Choice E) Alcohol cessation will undoubtedly improve the patients overall health, but will

3/19/2011 Item 1 of 44
usmleworld.com/Step3//qstShow.as 1/2
not slow progression of AAA size.
Educational objective:
Indications for surgery in patients with AAA include: diameter greater than about 5 cm,
presence of symptoms, or rapid rate of growth. For patients being medically managed,
smoking cessation is the intervention with the greatest likelihood of slowing AAA progression.
24% of people answered this question correctly;
3/19/2011 Item 1 of 44
usmleworld.com/Step3//qstShow.as 2/2

A 62-year-old Caucasian male returns to clinic for re-evaluation of his hypertension. For the
past six months, his blood pressure has become increasingly difficult to control despite the
addition of several antihypertensive medications. Aside from occasional headaches, he does
not have any complaints. Past medical history is also significant for type 2 diabetes mellitus,
peripheral arterial disease, hyperlipidemia, and coronary artery disease. Currently, he is on
metoprolol, amlodipine, hydrochlorothiazide, lisinopril, and hydralazine for his blood pressure.
Other medications include glipizide, clopidogrel, aspirin, and simvastatin. He is not allergic to
any medications. He smokes cigars on a daily basis and drinks alcohol socially. He denies
illicit drug use. Vital signs include temperature 36C (96.8F), pulse 76/min, respiratory rate
16/min, and blood pressure 152/95 mm Hg. On examination, there are no carotid bruits and
no jugular venous distension. His lungs are clear to auscultation. His PMI is palpated lateral to
the midclavicular line on the left. On auscultation, heart sounds are normal and there is no
murmur. The abdomen is soft and nontender. A bruit is heard on auscultation to the left of the
umbilicus. There is trace pedal edema. The distal pulses of the lower extremities are
diminished bilaterally. Laboratory studies show:
Leukocyte count
4,500/mm
3
Hemoglobin 11.5 g/dL
MCHC 33%
MCV 89 fl
Platelets
205,000/mm
3
Sodium 140 mEq/L
Potassium 3.4 mEq/L
Chloride 101 mEq/L
Bicarbonate 22 mEq/L
Blood urea nitrogen (BUN) 32 mg/dL
Serum creatinine 2.8 mg/dL
Calcium 9.1 mg/dL
Blood glucose 169 mg/dL
Urinalysis
Specific gravity 1.022
Protein 2+
Blood Negative
Glucose Negative
Ketones Negative
Esterase Negative
Nitrites Negative
Bacteria None
WBC 2-5/hpf
RBC 1-2/hpf
Which of the following is the most appropriate initial diagnostic test?
A. Magnetic resonance angiography
B. CT abdominal angiography

3/19/2011 Item 2 of 44
usmleworld.com/Step3//qstShow.as 1/2
C. Intravenous pyelogram
D. Angiography of the renal arteries
E. Duplex Doppler ultrasonography

Explanation:
In patients with hypertension difficult to control despite the use of several medications, causes
of secondary hypertension should be investigated. Renal artery stenosis is an important,
potentially reversible cause of secondary hypertension that tends to develop in patients with
other risk factors for renovascular disease, including diffuse atherosclerosis, resistant or
refractory hypertension, worsening renal insufficiency with ACE inhibitor use, or asymmetry in
kidney size in the presence of uncontrolled hypertension. This patient has several clinical
findings consistent with renal artery stenosis, including diffuse atherosclerosis, renal
insufficiency, and a bruit in the region of the renal artery. As a result, diagnostic testing with
duplex Doppler ultrasound of the renal arteries is indicated. This test is preferred over the
other options because of the patient's renal insufficiency, as any contrast-enhanced study can
cause contrast-induced nephropathy in patients with decreased renal function. In addition, MR
angiography uses a gadolinium-based contrast, which has been linked to nephrogenic
systemic fibrosis (NSF) in patients with poor renal function.
(Choice A) MR angiography of the renal arteries is an excellent diagnostic test for renal artery
stenosis in patients with normal renal function. However, it should be used with caution in
patients with renal insufficiency because of the risk of NSF.
(Choice B) If necessary, CT angiography can be performed in patients with renal insufficiency
to assess for renal artery stenosis, with certain protective measures undertaken prior to the
scan. However, duplex Doppler ultrasonography is the better initial test, as it has no adverse
effects on renal function.
(Choice C) An intravenous pyelogram may reveal an asymmetry in the size of the kidneys in
patients with renal artery stenosis. However, it is no longer used as a diagnostic test in
patients with this condition.
(Choice D) The iodinated contrast used in angiography should be avoided in this patient
because of the risk of contrast-induced nephropathy.
Educational objective:
In patients with renal insufficiency who require evaluation for renal artery stenosis, the most
appropriate initial screening test is a duplex Doppler ultrasound of the renal arteries. Contrast-
enhanced studies should be used with extreme caution because of the risk of contrast-induced
nephropathy.
48% of people answered this question correctly;
3/19/2011 Item 2 of 44
usmleworld.com/Step3//qstShow.as 2/2

A 76-year-old diabetic, African-American man is admitted to the hospital with a three-day
history of nausea, vomiting, and generalized abdominal distention. His physical examination
and diagnostic workup reveal the presence of a complete small bowel obstruction. He
undergoes exploratory laparotomy, and is extubated postoperatively in the surgical intensive
care unit without any complications. The day after extubation, he complains of a sudden onset
of difficulty in breathing, nausea and marked diaphoresis. On physical examination, his
temperature is 36.1C (97F), heart rate is 32/min, respiratory rate is 26/min, and blood
pressure is 84/50 mmHg. On lung auscultation, there are bilateral basilar rales present.
Cardiovascular examination reveals normal heart sounds with no murmurs or rubs. Chest x-ray
is consistent with pulmonary edema. An EKG reveals the presence of marked sinus
bradycardia, with 3 mm ST segment elevations in leads II, III, aVF, aVL, V5, and V6, consistent
with a diagnosis of acute inferolateral wall myocardial infarction. Which of the following is the
most appropriate next step in the management of this patient?
A. Start the patient on full-dose anticoagulation with heparin.
B. Administer thrombolytic therapy with reteplase.
C. Urgent transvenous cardiac pacing.
D. Start the patient on low-dose dopamine infusion.
E. Administer IV norepinephrine.

Explanation:
Sinus bradycardia is commonly seen as a complication of an inferior wall myocardial
infarction. It is usually present transiently, immediately after or within the first six hours, and
resolves within 24 to 48 hours of acute myocardial infarction. Most of the episodes are
asymptomatic, and resolve without any therapy or intervention.
Active intervention is required in patients with sinus bradycardia after myocardial infarction, if
the patient exhibits signs and symptoms of hemodynamic compromise. The definite treatment
is by reperfusion of the infarcted myocardium, either with thrombolysis or percutaneous
transluminal coronary angioplasty (PTCA). This patient is exhibiting the signs of hemodynamic
instability with new onset congestive heart failure secondary to sinus bradycardia after an
inferolateral wall MI. He should receive a temporary transvenous cardiac pacer as a bridge to
the PTCA.
(Choices A and B) Anticoagulation with full-dose heparin or thrombolysis with reteplase is
contraindicated in patients with recent (less than two weeks) intraabdominal surgery.
(Choice D) Low-dose dopamine acts predominantly on dopaminergic receptors in the
mesenteric, renal and coronary vessels, and causes selective vasodilation. Moderate doses
of dopamine stimulate beta 1 adrenergic receptors and increase cardiac output; however, it
can worsen the myocardial oxygen demand, and further worsen the hemodynamic status.
(Choice E) Norepinephrine acts on both alpha 1 and beta 1 receptors, causing
vasoconstriction and an increase in cardiac output. An increase in the blood pressure due to

3/19/2011 Item 3 of 44
usmleworld.com/Step3//qstShow.as 1/2
the above effects can cause a reflex bradycardia, thereby causing further decrease in the
heart rate. It can also increase the myocardial oxygen demand, and should therefore not be
used in patients after an acute myocardial infarction.
Educational Objective:
Transient sinus bradycardia can occur after an inferior wall myocardial infarction, and is usually
asymptomatic. Persistent bradycardia causing hemodynamic instability should be treated with
intravenous atropine or temporary transvenous cardiac pacing.
41% of people answered this question correctly;
3/19/2011 Item 3 of 44
usmleworld.com/Step3//qstShow.as 2/2

A 34-year-old woman is admitted with a diagnosis of acute lymphoblastic leukemia (ALL).
She is scheduled to receive induction chemotherapy with vincristine, doxorubicin, and
prednisone. She has no other past medical history. She has never smoked or used illegal
drugs. Which of the following tests is the most appropriate for monitoring this patient's
condition while on chemotherapy?
A. Ambulatory 24-hour EKG monitoring
B. Coronary angiography
C. Measurement of cardiac troponins
D. Radionuclide ventriculography
E. Stress myocardial perfusion imaging

Explanation:
Cardiotoxicity is one of the primary concerns when administering anthracycline chemotherapy
agents, such as doxorubicin and daunorubicin. The risk of cardiotoxicity with these agents is
related to the cumulative dose, and is increased in patients with preexisting cardiac disease
with a low ejection fraction. In the general population, resting echocardiography is often used
to evaluate ejection fraction but there is potential for significant variability in the results.
Radionuclide ventriculography, also known as a MUGA (multigated acquisition) scan, is
typically used to monitor patients receiving cardiotoxic chemotherapy as it is a highly accurate
and reproducible test for quantitating left ventricular ejection fraction. A radionuclide
ventriculogram is generally performed at baseline before chemotherapy is initiated, and
before each subsequent dose of chemotherapy. The therapeutic regimen is dependent on the
baseline cardiac function, with anthracycline chemotherapy contraindicated in patients with
baseline ejection fractions less than 30%, and modified dosing required for patients with
baseline ejection fractions of less than 50%. A decrease in the ejection fraction by ten or more
percentage points may warrant discontinuation of therapy.
(Choice A) Ambulatory 24-hour EKG monitoring is most useful when there is suspicion for an
intermittent arrhythmia, for example, in patients being evaluated for syncope.
(Choice B) Anthracycline chemotherapy agents may worsen the ejection fraction by a direct
toxic effect on the myocardium. Patients with coronary artery disease may have an increased
risk of cardiotoxicity if they have a low baseline ejection fraction, but this is secondary to the
preexisting myocardial damage as opposed to the coronary artery disease itself.
(Choice C) Cardiac troponins may be elevated in patients with myocardial damage from
doxorubicin. This may one day be used as a monitoring test, but at the present time there is
insufficient evidence to support its clinical use in the above described context.
(Choice E) Stress myocardial perfusion imaging is used to stratify patients at risk of having
coronary artery disease. However, accurate measurement of the ejection fraction is the most
important and classically used test for evaluating and following patients receiving cardiotoxic
chemotherapy.

3/19/2011 Item 4 of 44
usmleworld.com/Step3//qstShow.as 1/2
Educational objective:
Radionuclide ventriculography has high accuracy and reproducibility for measuring ejection
fractions. Its most common use is in the initial evaluation and subsequent follow-up of patients
receiving cardiotoxic chemotherapy agents such as doxorubicin.
37% of people answered this question correctly;
3/19/2011 Item 4 of 44
usmleworld.com/Step3//qstShow.as 2/2

The following vignette applies to the next 2 items
A 72-year-old Caucasian man comes to the physician's office because of palpitations. He has
had these symptoms for the past few hours. He also complains of dyspnea on exertion and
malaise for the past few months. His other medical problems include hypertension and
degenerative joint disease. He has smoked one pack of cigarettes daily for 55 years. His
medications include hydrochlorothiazide and fosinopril. His blood pressure is 140/90 mm Hg,
pulse is 148/min, and respirations are 20/min. Physical examination reveals scattered rhonchi
and wheezes in both lungs. The heart sounds and the rest of the examination are normal. The
electrocardiogram (EKG) demonstrates tachycardia with narrow QRS complexes, different P
wave morphologies, and variable PR segments and R-R intervals. A diagnosis of multifocal
atrial tachycardia is made.
Item 1 of 2
Which of the following is the most appropriate next step in management?
A. Administer an intravenous bolus of digoxin
B. Obtain a chest x-ray
C. Administer intravenous amiodarone
D. Check the arterial oxygen saturation
E. Administer an intravenous dose of atenolol

Explanation:
Multifocal or multiform atrial tachycardia (MAT) is characterized by the presence of 3 or more
P waves of different morphologies. The QRS complexes are narrow, while the PR segments
and the R-R intervals are variable. The heart rate can reach up to 200 beats per minute.
MAT is usually secondary to the following conditions:
1) hypoxia
2) chronic obstructive pulmonary disease (COPD)
3) hypokalemia
4) hypomagnesemia
5) coronary/ hypertensive/ valvular disease
6) medications (i.e., theophylline, aminophylline, isoproterenol)
Hypoxia and COPD are the most common etiologies that affect mainly elderly patients. In this
patient, hypoxia must first be ruled out since its rapid correction may eliminate the arrhythmia.
(Choices A and C) Both digoxin and amiodarone are not effective therapeutic drugs for MAT.
(Choice B) A chest x-ray will be useful in ruling out pneumonia or another lung pathology which
may explain the patient's dyspnea, malaise and tachypnea; however, identifying hypoxemia via
pulse oximetry allows for the immediate correction of the underlying cause of the arrhythmia.

3/19/2011 Item 5 of 44
usmleworld.com/Step3//qstShow.as 1/2
(Choice E) Although atenolol can be used in the management of MAT, initial therapy is
directed towards immediate correction of any identified etiology of this tachyarrhythmia.
Moreover, this patient's significant smoking history and physical findings (i.e., bilateral
wheezing and rhonchi) are suggestive of COPD, which is another cause of MAT; such a
condition can be worsened by the use of beta-blockers.
Educational Objective:
Multifocal atrial tachycardia (MAT) is a tachyarrhythmia that arises in patients with hypoxia,
COPD, hypomagnesemia, hypokalemia, heart disease or individuals who are taking
aminophylline, theophylline or isoproterenol. It is characterized by the presence of narrow QRS
complexes, P waves of 3 or more different morphologies, and variable PR segments and R-R
intervals. Therapy is aimed at correcting the underlying cause.
34% of people answered this question correctly;
3/19/2011 Item 5 of 44
usmleworld.com/Step3//qstShow.as 2/2

Item 2 of 2
The patient is admitted to the hospital. His dyspnea has improved, but he continues to have
palpitations. His heart rate is 142/min. His blood pressure and respiratory rate remain
unchanged. The EKG shows no new changes. His labs reveal the following:
CBC
Ht 50%
Platelet count 320,000/cmm
Leukocyte count 7,000/cmm
Serum chemistry
Serum Na 125 mEq/L
Serum K 2.9 mEq/L
Chloride 80 mEq/L
Bicarbonate 33 mEq/L
BUN 25 mg/dL
Serum creatinine 1.2 mg/dL
Calcium 10.1 mg/dL
Blood glucose 84 mg/dL
Serum magnesium 2.0 mEq/L
Which of the following is the most appropriate pharmacotherapy?
A. Start intravenous verapamil
B. Start intravenous atenolol
C. Start hypertonic saline infusion
D. Start potassium replacement
E. Prepare for synchronized cardioversion

Explanation:
The mainstay of therapy of multifocal atrial tachycardia is to treat the underlying cause of the
arrhythmia. Hypoxia should be quickly identified and corrected. If the patient fails to improve,
electrolyte abnormalities (e.g., hypokalemia or hypomagnesemia) should be sought.
This patient has hypokalemia, which could be due to his use of diuretics. He should be treated
with potassium replacement. If this measure is not effective, drug therapy is necessary.
(Choice A) Verapamil is the preferred calcium-channel blocker for patients with MAT. It should
be started only if there is no other correctable cause of MAT. It is effective for rate control,
although it converts MATs into sinus rhythms in only 43% of the patients.
(Choice B) In the absence of contraindications, beta-blockers such as metoprolol have

3/19/2011 Item 6 of 44
usmleworld.com/Step3//qstShow.as 1/2
proven to be effective for the conversion of MATs into sinus rhythms in 80% of the cases;
however, verapamil is prefered in patients with bronchospasm.
(Choice C) Hyponatremia does not cause MAT; therefore, sodium replacement will not lead
to rate control or MAT elimination. His hyponatremia is most likely the result of
hydrochlorothiazide.
(Choice E) Synchronized cardioversion is not effective in converting MAT into sinus rhythm.
Educational objective:
Treatment of multifocal atrial tachycardia (MAT) is directed towards the correction of the
underlying etiology (e.g., hypoxia, hypokalemia, hypomagnesemia). If therapy is not effective
and there are no contraindications, beta-blockers can be used successfully. In patients with
asthma or COPD, verapamil is the drug of choice.
78% of people answered this question correctly;
3/19/2011 Item 6 of 44
usmleworld.com/Step3//qstShow.as 2/2

A 38-year-old Caucasian female presents to your office for a routine check-up. She has no
present complaints. Her past medical history is significant for hypertension and diabetes
mellitus, type II, both of which are controlled with low-dose hydrochlorothiazide and glyburide,
respectively. She smokes one pack of cigarettes daily, and consumes alcohol occasionally.
Her blood pressure is 130/85 mmHg and heart rate is 85/min. Physical examination reveals
moderate obesity (BMI =30 kg/m2). Her fasting blood glucose level is 120 mg/dL and HbA1C
level is 6.7%. Which of the following factors is the single most important predictor of future
cardiovascular events for this woman?
A. Obesity
B. Age
C. Hypertension
D. Smoking
E. Diabetes mellitus
F. Gender
G. Alcohol consumption

Explanation:
Diabetes mellitus is the single most important predictor of adverse cardiovascular outcomes.
Due to the very high prevalence of atherosclerotic vascular disease and increased risk of
future cardiovascular events in diabetics, diabetes mellitus is considered as an equivalent of
coronary heart disease (CHD). Diabetes has even more prognostic significance for women.
Moreover, any additional risk factor in patients with diabetes mellitus (e.g., hypertension,
smoking) has a synergistic effect, greatly increasing the risk. In this patient, glycemic status
seems to be well controlled, but diabetes mellitus remains as the most important risk factor.
The role of glycemic control in the development of macrovascular disease in patients with
type-2 diabetes is not firmly established.
(Choices F and B) Female gender traditionally is considered protective until menopause.
(Choice C) Hypertension is a well-established risk factor. The blood pressure should be kept
below 130/85 mmHg in diabetics, unlike non-diabetic individuals whose target blood pressure
is below 140/90 mmHg.
(Choice D) Smoking is an important modifiable risk factor. The risk of cardiovascular events
declines rapidly after smoking cessation, and approaches that of non-smokers in several
years.
(Choice A) Obesity is an independent risk factor for cardiovascular disease, but it is also
associated with an unfavorable metabolic profile.
(Choice G) Moderate alcohol consumption is associated with protection against adverse
cardiovascular outcomes.

3/19/2011 Item 7 of 44
usmleworld.com/Step3//qstShow.as 1/2
Educational Objective:
Diabetes mellitus is the single most important predictor of adverse cardiovascular outcomes,
especially in women. It is considered to be an equivalent of coronary heart disease (CHD).
The role of glycemic control in the development of macrovascular disease in patients with
type-2 diabetes is not firmly established.
35% of people answered this question correctly;
3/19/2011 Item 7 of 44
usmleworld.com/Step3//qstShow.as 2/2

The following vignette applies to the next 2 items
A 70-year-old Caucasian man is admitted to the hospital because of a change in mental status,
fever, productive cough and shortness of breath. He has diabetes mellitus type 2, hypertension,
atrial fibrillation, peripheral vascular disease and chronic renal failure. His current medications
are warfarin, sotalol, insulin, and lisinopril. His chest x-ray shows an alveolar infiltrate in the left
base. He is admitted and given azithromycin, ceftriaxone and oxygen therapy. He becomes
agitated during the night, and he receives a dose of parenteral haloperidol. Two hours after the
administration of haloperidol, he suddenly becomes unresponsive.
Item 1 of 2
Which of the following is the most appropriate next step in management, based on the following
rhythm strips shown by the EKG montior?
A. Intravenous magnesium sulfate
B. Defibrillation
C. Synchronized cardioversion
D. Temporal transvenous overdrive pacing
E. Intravenous amiodarone

Explanation:
The patient's EKG tracing demonstrates prolonged QT intervals, which may be caused by
chronic renal failure, liver disease, electrolyte abnormalities (i.e., hypomagnesemia,
hypokalemia), and the use of drugs such as sotalol, antibiotics (macrolides, pentamidine,
trimethoprim-sulfamethoxazole), psychotropic drugs (risperidone, phenothiazines, tricyclic
antidepressants) or antihistamines (terfenadine, astemizole). Acquired long QT syndrome may
then lead to torsade de pointes.

3/19/2011 Item 8 of 44
usmleworld.com/Step3//qstShow.as 1/2
Torsade de pointes is characterized by the following EKG findings:
1. Ventricular rate greater than 100 beats/min (often 160-250/min)
2. Frequent variations in the QRS morphology
3. Peaks of the QRS complexes appear to "twist" around the isoelectric line (torsade de
pointes =twisting of the points)
The above EKG tracing typically illustrates a progressive, sinusoidal, cyclic alteration of the
QRS axis. Another important clue is the presence of the markedly prolonged QT interval in the
last sinus beat preceding the onset of the arrhythmia.
The treatment of torsade de pointes or polymorphic ventricular tachycardia with hemodynamic
compromise is immediate nonsynchronized electric defibrillation.
(Choice A) Magnesium sulfate is the drug of choice in stable patients.
(Choice D) Temporary overdrive pacing is indicated in patients who are refractory to
intravenous magnesium administration.
(Choice C) Synchronized cardioversion is indicated for patients with stable ventricular
tachycardia, atrial fibrillation, atrial flutter or supra ventricular tachycardia. It is not useful for
patients with torsade de pointes.
(Choice E) Amiodarone is one of the drugs that can potentially cause torsade de pointes. Its
use is contraindicated in this case; however, certain antiarrhythmics such as lidocaine or
isoproterenol can be used.
Educational Objective:
Torsade de pointes is a life-threatening polymorphic ventricular arrhythmia. It is associated with
QT prolongation, which can be acquired or congenital. Acquired QT prolongation can be due to
hypomagnesemia, hypokalemia, or the use of certain medications (macrolides, antihistamines,
psychotropic drugs, etc). If the patient is unstable, immediate defibrillation is indicated.
44% of people answered this question correctly;
3/19/2011 Item 8 of 44
usmleworld.com/Step3//qstShow.as 2/2

Item 2 of 2
The patient regains consciousness after receiving the appropriate therapy. He is now awake
and alert, but oriented only to person. His blood pressure is 120/70 mm Hg, pulse is 82/min,
and respirations are 14/min. The results of the lab tests that were ordered thirty minutes prior
to this event reveal the following:
CBC
Hct 31%
MCV 89fl
Platelet count 220,000/cmm
Leukocyte count 14,000/cmm
Segmented neutrophils 75%
Bands 6%
Lymphocytes 16%
Monocytes 3%
Serum chemistry
Serum Na 144 mEq/L
Serum K 4.5 mEq/L
Chloride 108 mEq/L
Bicarbonate 20 mEq/L
BUN 40 mg/dL
Serum creatinine 2.4 mg/dL
Calcium 9.6 mg/dL
Blood glucose 240 mg/dL
Magnesium 1.8 mEq/L
Phosphorus 3.5 mg/dL
Which of the following is the most appropriate next step in management?
A. Intravenous magnesium sulfate
B. Non-synchronized cardioversion
C. Synchronized cardioversion
D. Temporary transvenous overdrive pacing
E. Intravenous lidocaine

Explanation:
The patient should receive intravenous magnesium sulfate, which is the mainstay of therapy for
torsade de pointes. Normal serum magnesium levels cannot necessarily rule out the possibility
of hypomagnesemia, because such findings may reflect only normal total magnesium levels in
the serum, even if the ionized fraction of magnesium is low. Unfortunately, measurement of
ionized magnesium is a complicated procedure and is not feasible in most hospitals; when

3/19/2011 Item 9 of 44
usmleworld.com/Step3//qstShow.as 1/2
feasible, the results are usually reported with much delay. For these reasons, IV magnesium
sulfate is administered to all patients with torsade de pointes with little regard for the serum
magnesium levels.
Magnesium sulfate plays a role in both the treatment of torsade de pointes and prevention of
recurrence of arrhythmia. Benefits are seen even if there is no readily observed improvement
or changes in the EKG findings (shortening of the QT interval) and serum magnesium levels.
(Choice B) Non-synchronized cardioversion or defibrillation is indicated only when the patient
is having torsade de pointes with hemodynamic instability or loss of consciousness.
(Choice C) Synchronized cardioversion is not indicated in the management of torsade de
pointes.
(Choice D) Temporary transvenous overdrive pacing is indicated only in those cases in which
torsade de pointes is refractory to intravenous magnesium sulfate administration.
(Choice E) Lidocaine is a class 1B antiarrhythmic drug that shortens the duration of the action
potential. It has been effective in some cases of torsade de pointes, but it is not as effective as
intravenous magnesium sulfate. It is indicated only if there is no response to intravenous
magnesium and there is no temporary overdrive pacing available.
Educational Objective:
Intravenous magnesium sulfate is indicated for the treatment and prevention of recurrent
episodes of torsade de pointes. This should be administered regardless of the patient's
serum magnesium levels. If therapy with magnesium fails, placement of a temporary
transvenous overdrive pacemaker is indicated.
64% of people answered this question correctly;
3/19/2011 Item 9 of 44
usmleworld.com/Step3//qstShow.as 2/2

The following vignette applies to the next 2 items
A 38-year-old healthy Caucasian man is brought to the emergency department because of
sudden onset of shortness of breath and diaphoresis. He denies fever, chills, cough or
abdominal pain. He has no other medical problems. He had surgery for bilateral inguinal
hernia when he was 16. He does not use tobacco, alcohol or illicit drugs. He takes no
medication and has no known drug allergies. His blood pressure is 110/60 mm Hg, pulse is
116/min and respirations are 28/min. He is in marked respiratory distress. Pallor and
diaphoresis are noted. His skin is velvety and has multiple scars. On auscultation of the heart,
an early, decrescendo, systolic murmur at the cardiac apex is heard; the murmur decreases
with Valsalva maneuver, and increases with the grip maneuver, radiating to the axilla. The first
sound is barely audible; the second heart sound is normal. A fourth heart sound is also
present. There are bilateral crackles in both lungs. J ugular venous distention and hepatojugular
reflux are present. The abdomen is soft, non-tender and non-distended. The neurologic
examination reveals no abnormalities. The initial EKG shows sinus tachycardia with
occasional premature ventricular complexes. The chest x-ray reveals no cardiomegaly, but
bilateral alveolar infiltrates and hilar prominence are present.
Item 1 of 2
Which of the following is the most likely cause of his condition?
A. Acute myocardial infarction
B. Rupture of chordae tendineae
C. Pulmonary embolism
D. Infective endocarditis
E. Papillary muscle rupture

Explanation:
The patient presents with signs and symptoms of acute heart failure. His EKG findings reveal
occasional premature ventricular complexes (PVC); there are no signs of ischemia or
ventricular hypertrophy. His pathologic murmur (systolic murmur that is heard in the apex,
radiates to the axilla, increases with the grip maneuver, and decreases with Valsalva) is
characteristic of mitral regurgitation, which may lead to acute heart failure. Acute mitral
regurgitation is usually characterized by a soft, decrescendo systolic murmur (can be early,
midsystolic or holosystolic), a decreased first heart sound, and the presence of a fourth heart
sound. Due to the acute nature of this condition, there is no evidence of left ventricular
hypertrophy in the electrocardiogram or the chest x-ray.
The four common causes of acute heart failure are papillary muscle rupture, infective
endocarditis, rupture of chordae tendineae, and chest wall trauma with compromise of the
valvular apparatus. The most common cause of isolated, severe acute mitral regurgitation in
adults is rupture of chordae tendineae with or without associated myxomatous disease. The
diagnosis is confirmed by echocardiography.

3/19/2011 Item 10 of 44
usmleworld.com/Step3//qstShow.as 1/2
(Choice A) Myocardial infarction can be complicated by acute mitral regurgitation when there
is rupture of the papillary muscle; however, this patient does not have evidence of ischemia in
the EKG.
(Choice C) Patients with pulmonary embolism can develop acute cor pulmonale; however, in
such a setting, the EKG will show right axis deviation, right bundle branch block or both. The
above patient clearly has acute pulmonary edema.
(Choice D) The patient does not have fever or risk factors for bacterial endocarditis.
(Choice E) Spontaneous papillary muscle rupture usually presents in elderly people who have
acute chest pain or as a complication of myocardial infarction. None of these are present here.
Educational Objective:
Rupture of chordae tendineae should be suspected in healthy individuals who develop flash
pulmonary edema (heart failure) associated with an acute mitral regurgitation. The differential
diagnosis of this condition includes infective endocarditis, papillary muscle rupture secondary
to ischemia, and mitral valve rupture secondary to trauma.
52% of people answered this question correctly;
3/19/2011 Item 10 of 44
usmleworld.com/Step3//qstShow.as 2/2

Item 2 of 2
The patient is feeling better after receiving the appropriate therapy. A more detailed physical
examination reveals the presence of scoliosis and pes planus. The cardiac enzymes were
negative. Blood gas analysis reveals respiratory alkalosis. Complete blood cell count and
serum chemistry results are unremarkable. The patient is scheduled for echocardiography.
Which of the following is the most probable diagnosis?
A. Coronary artery disease
B. Rheumatic fever
C. Marfan syndrome
D. Ehlers-Danlos syndrome
E. Thyrotoxicosis

Explanation:
The patient has features compatible with Ehlers-Danlos syndrome. This condition can cause a
myxomatous degeneration of the mitral valve, leading to acute rupture of chordae tendineae.
Pes planus and scoliosis are frequent, though not specific findings. The skin can be velvety or
thin, and is usually covered with multiple characteristic "cigarette-paper" scars, due to its frailty
and easy bruisability. A past history of hernias and mitral valve prolapse is not uncommon.
J oint hypermobility and skin hyperextensibility ("rubber man syndrome") can be dramatic in
severe cases.
(Choice A) There is no evidence of ischemic heart disease. Troponins and CK-MB are
normal. The EKG shows no ischemic changes.
(Choice E) Thyrotoxicosis can lead to acute, high-output, heart failure; however, this will not
explain the mitral regurgitation. Skin changes of thyrotoxicosis can be confounded with those
found in Ehlers-Danlos, but other characteristic signs and symptoms are lacking.
(Choice B) Rheumatic fever is a common cause of mitral regurgitation in young individuals;
however, the course of the disease is chronic, and the patients usually have evidence of left
ventricular and atrial hypertrophy in the EKG.
(Choice C) Marfan syndrome can affect the valvular apparatus, causing degeneration of the
mitral and aortic valves. The presentation is usually chronic, progressive mitral regurgitation,
and rarely, acute regurgitation due to ruptured chordae tendineae. This patient does not have
arachnodactyly, loose joints or increased arm span, which are characteristic of Marfan
syndrome.
Educational Objective:
Marfan or Ehlers-Danlos syndrome must be suspected in patients with connective tissue
abnormalities and an acute mitral regurgitation secondary to chordae tendineae rupture,
although a primary, pre-existing mitral valve prolapse (MVP) is the most common cause.

3/19/2011 Item 11 of 44
usmleworld.com/Step3//qstShow.as 1/2
Some cases may be idiopathic (individuals who experience rupture of the chordae tendineae
without previous MVP or connective tissue disease), but an etiology can be found in most
occasions.
54% of people answered this question correctly;
3/19/2011 Item 11 of 44
usmleworld.com/Step3//qstShow.as 2/2

A 55-year-old Caucasian female presents to your office for a routine check-up. She has no
present complaints. She experienced an acute myocardial infarction two years ago. Her
current medications are metoprolol and aspirin. She does not smoke or consume alcohol. Her
blood pressure is 125/80 mmHg and heart rate is 65/min. The physical examination is
insignificant. Her fasting blood glucose level is 100 mg/dL. Her lipid profile is:
Total cholesterol: 240 mg/dL
HDL cholesterol: 40 mg/dL
LDL cholesterol: 140mg/dL
Triglycerides: 400 mg/dL
Which of the following medications should be considered first in the treatment of this patient's
dyslipidemia?
A. Atorvastatin
B. Gemfibrozil
C. Niacin
D. Cholestyramine
E. Estrogen replacement therapy

Explanation:
This patient's lipid profile indicates hypercholesterolemia and hypertriglyceridemia (>200
mg/dL). Her history of a previous myocardial infarction places her in a high-risk group;
therefore, aggressive measures to control her lipid profile should be undertaken. The elevated
LDL cholesterol in this patient is the first target of therapy. The non-HDL cholesterol (total
cholesterol - HDL cholesterol) is the second target of therapy after controlling the LDL
cholesterol levels in patients with hypertriglyceridemia . According to the current
recommendations, non-HDL cholesterol should be kept below 130 mg/dL in patients with
established coronary heart disease (CHD). Atorvastatin is effective in lowering non-HDL
cholesterol and triglycerides; therefore, it is the best initial choice for this patient.
(Choices B and C) Gemfibrozil or niacin should be added if atorvastatin fails to control the
hypertriglyceridemia.
(Choice D) Cholestyramine increases the triglyceride concentration in the blood; therefore, it
is contraindicated in this patient.
(Choice E) Estrogen replacement therapy is not a good choice due to the potential adverse
cardiovascular outcomes.
Educational Objective:
In patients with hypercholesterolemia and hypertriglyceridemia, a statin is the best initial drug
of choice. The primary target of therapy is normalizing LDL-cholesterol; the secondary target is
normalizing non-HDL cholesterol. If the statin fails to control hypertriglyceridemia, gemfibrozil

3/19/2011 Item 12 of 44
usmleworld.com/Step3//qstShow.as 1/2
or niacin can be added.
63% of people answered this question correctly;
3/19/2011 Item 12 of 44
usmleworld.com/Step3//qstShow.as 2/2

A 68-year-old woman comes to the office for follow-up of her anticoagulation therapy. Her
medical problems include chronic atrial fibrillation, hypertension, congestive heart failure
(CHF) NYHA II, and coronary artery disease. Her medications include daily warfarin 6 mg,
lisinopril, furosemide, and carvedilol. She started taking an over-the-counter supplement 2
weeks ago to help increase her energy level but cannot recall the name. She denies bleeding
episodes. Echocardiogram performed 2 months ago shows a dilated left atrium and
moderately depressed left ventricular ejection fraction. Laboratory evaluation done early in the
morning shows an international normalized ratio (INR) of 4.8 with a prothrombin time of 52 sec.
Which of the following is the most appropriate next step in management?
A. Administer low-dose (12.5 mg) oral vitamin K
B. Administer low-dose (12.5 mg) subcutaneous vitamin K
C. Transfuse 1 or 2 units of fresh frozen plasma (FFP)
D. Hold warfarin for 1 or 2 days and monitor INR
E. Administer high-dose (510 mg) intravenous vitamin K

Explanation:
The algorithm for treatment of a patient with supratherapeutic INR depends on the INR level
and presence of clinically serious bleeding. Studies have shown that oral vitamin K is as
effective as intravenous vitamin K in lowering the INR in 24 hours. Since intravenous vitamin K
has a risk of anaphylaxis, oral vitamin K is the preferred route.
Patients INR <5 and no clinical evidence of serious bleeding should have their warfarin held for
1 or 2 days and then restarted when the INR is in the therapeutic range. Patients with INR 59
and no serious bleeding should have their warfarin held and be given a low dose (12.5 mg)
of oral vitamin K if there is an increased risk of future bleeding (Choice A). Patients with INR
>9 and no serious bleeding should have their warfarin held and be given a higher dose
(2.55 mg) of oral vitamin K. Patients with serious bleeding, regardless of the INR, should

3/19/2011 Item 13 of 44
usmleworld.com/Step3//qstShow.as 1/2
have their warfarin held and receive intravenous vitamin K, FFP, recombinant factor VIIa, or
prothrombin complex concentrate (Choices C and E).
This patient's supratherapeutic INR is likely due to a drug interaction between the supplement
and the warfarin. She has no evidence of bleeding, has INR <5, and can be managed by
holding her warfarin for 1 or 2 days and monitoring her INR. Giving this patient higher doses of
vitamin K or FFP would completely reverse anticoagulation and potentially increase the risk of
future thrombotic events given her risk factors of CHF, atrial fibrillation, and dilated atrium.
(Choice B) Subcutaneous vitamin K is not as effective as either oral or intravenous vitamin K
and only slightly better than placebo in lowering the INR within 24 hours. As a result, it is not
recommended to treat supratherapeutic INR.
Educational objective:
Patients with supratherapeutic international normalized ratio (INR) <5 on warfarin and no
serious bleeding should have their warfarin held and restarted when the INR is therapeutic. All
patients with supratherapeutic INR 5 and no serious bleeding should receive oral vitamin K in
addition to holding their warfarin. Oral vitamin K is the preferred route at any supratherapeutic
INR unless there is serious bleeding.
77% of people answered this question correctly;
3/19/2011 Item 13 of 44
usmleworld.com/Step3//qstShow.as 2/2

A 55-year-old Caucasian man comes to the emergency department because of palpitations.
He denies any chest pain or shortness of breath. He has hypertension, asthma and coronary
artery disease. He does not use tobacco, alcohol or drugs. His medications include lisinopril,
aspirin and albuterol metered dose inhaler. His blood pressure is 140/80 mm Hg, heart rate is
130 to150/min, pulse is 120/min and respirations are 19/min. Examination reveals an anxious
man with markedly irregular carotid and radial pulses. Auscultation of the heart reveals a
systolic murmur in the apex, which radiates to the axilla. The lungs are clear. The abdomen is
soft, non-tender and non-distended. Neurologic examination shows no abnormalities. The
patient is placed on an EKG monitor, and a 12-lead EKG is shown below.
He receives one bolus of intravenous digoxin by an emergency room physician. After five
minutes, reevaluation reveals a heart rate of 136/min. Which is the most appropriate next step
in management?
A. Repeat half-dose of intravenous digoxin
B. Reassure the patient and reevaluate him in five minutes
C. Start intravenous diltiazem
D. Start intravenous propranolol
E. Repeat full dose of intravenous digoxin

Explanation:
The above EKG reading shows an irregularly irregular rhythm with absent P waves; this is
consistent with atrial fibrillation with rapid ventricular response. Calcium channel blockers are
the first line drugs for rate control in patients with paroxysmal atrial fibrillation. These are
indicated in most cases, unless the patient has significant congestive heart failure (CHF) or

3/19/2011 Item 14 of 44
usmleworld.com/Step3//qstShow.as 1/2
cardiac conduction system disease. (Their negative chronotropic and inotropic effects can
worsen CHF or heart blocks.) Diltiazem is effective in more than 90% of the cases. The
median time to maximal ventricular rate control is between 4 to 5 min of administration.
(Choices A, B and E) Intravenous digoxin has been traditionally used for acute rate control in
patients with atrial fibrillation and high ventricular rate; however, its use is not recommended in
this case due to its very slow onset of action (two hours), and time to cause a significant
reduction in the ventricular rate (approximately 5.5 hours). On the other hand, patients who
receive diltiazem for rate control can be reevaluated after five minutes, when maximal rate
control is achieved. For these reasons, diltiazem is the preferred antiarrhythmic agent over
digoxin.
(Choice D) Propranolol is a good alternative to diltiazem for acute rate control of atrial
fibrillation with high ventricular rate; however, it is less effective and is not recommended for
patients with asthma.
Educational Objective:
Atrial fibrillation with a high ventricular rate must be managed with calcium-channel blockers
(i.e., diltiazem), or beta-blockers (i.e., esmolol, metoprolol, propranolol). Rate control is
achieved with diltiazem, metoprolol or esmolol five minutes after their administration. On the
contrary, it takes approximately 5.5 hours for digoxin to have a significant effect in rate control.
54% of people answered this question correctly;
3/19/2011 Item 14 of 44
usmleworld.com/Step3//qstShow.as 2/2

A 50-year-old man comes to the office for a routine physical examination. He complains of
several episodes of left-sided, non-radiating chest pain of recent onset. He describes this pain
as a slowly-developing, pressure-like sensation which has no aggravating or relieving factors.
One week ago, he had an episode of unbearable chest pain, for which he was rushed to the
emergency department (ED) with his wife. At that time, he was anxious, diaphoretic, and had
shortness of breath. The EKG showed ST changes in the anterior chest leads; however, his
cardiac enzymes were normal at that time, so he was discharged from the ED with advice to
follow-up with his family physician. He is a construction worker and claims that his job is very
hard. He does not exercise. He has had type 2 diabetes mellitus for the past 12 years. His
medications include glipizide and multivitamins. Physical examination reveals a temperature
of 37C (99F), pulse rate of 80/min, blood pressure of 152/85 mmHg, and respirations of
12/min. His HbA1c is 7% (normal is 4 to 6%). His fasting lipid profile reveals a total cholesterol
level of 279 mg/dL and LDL 200 mg/dL. Which of the following is the most important next step
in the management of this patient?
A. Continue current therapy
B. Start him on nitrates
C. Coronary angiography
D. Start gemfibrozil
E. Start thiazide to control blood pressure

Explanation:
This patient has unstable angina, and he needs coronary angiography as soon as possible to
diagnose the degree and location of blockage in the coronary circulation. He has several
major risk factors for coronary artery disease, including diabetes mellitus, hypertension, and
hyperlipidemia. Coronary angiography will be extremely useful in deciding the final
management of this patient. The patient with a high risk of an acute coronary event should be
treated aggressively by performing coronary angiography, followed by percutaneous coronary
intervention (PCI) or CABG. In patients with diabetes, the coronary artery disease is usually
severe and progresses rapidly.
(Choice A) This patient is not doing well with his current therapy; therefore, continuing with his
therapy could be disastrous.
(Choice B) Nitrates would be helpful in alleviating his chest pain, but will not affect the ultimate
survival of this patient. The long-term use of nitrate in preventing acute coronary events or
coronary deaths is unclear.
(Choice D) Gemfibrozil mainly reduces the triglyceride levels, and is the first line agent in
patients with primary hypertriglyceridemia.
(Choice E) Thiazide is not a good option for this patient with unstable angina. Other
antihypertensive drugs which have been shown to improve the survival of patients with CAD
(i.e., beta-blockers, ACEI) can be more beneficial.

3/19/2011 Item 15 of 44
usmleworld.com/Step3//qstShow.as 1/2
Educational Objective:
Patients with unstable angina and with high risk for an acute coronary event should be
managed with early coronary angiography, followed by PCI or CABG. Patients with diabetes
usually have extensive coronary artery disease, which progresses rapidly.
55% of people answered this question correctly;
3/19/2011 Item 15 of 44
usmleworld.com/Step3//qstShow.as 2/2

A 57-year-old Caucasian woman is hospitalized because of dyspnea on exertion and loss of
consciousness. Her other medical problems include hypertension and gastroesophageal
reflux disease. Her social history and family history are not significant. Her medications include
hydrochlorothiazide and famotidine. Work-up showed aortic stenosis, and the patient
subsequently underwent aortic valve replacement (AVR) with bileaflet mechanical valve. There
is no evidence of left atrial enlargement on the echocardiogram. Her ejection fraction is 55-
60%. The preoperative angiogram did not show coronary artery disease. During the entire
hospitalization, she is in sinus rhythm. Which of the following is the most appropriate course of
action for the long-term management of her condition?
A. Aspirin 325 mg once daily
B. Warfarin with a goal INR of 2.0-3.0
C. Warfarin with a goal INR of 2.5-3.5
D. Warfarin with a goal INR of 3.0-4.0
E. No need of anticoagulation

Explanation:
The following are the anticoagulation recommendations are appropriate for a mechanical
valves (ACC/AHA):
A goal INR of 2.5 (range 2 to 3) is recommended for patients with AVR with bileaflet
mechanical valve, provided that the patient is in sinus rhythm, left atrium is of normal size, and
the left ventricular ejection fraction is normal, as in this patient.
A goal INR of 3 (range 2.5 to 3.5) is recommended for patients with MVR with bileaflet
mechanical valve or for patients with a bileaflet mechanical aortic valve who have atrial
fibrillation.
A goal INR of 3 (range 2.5 to 3.5) in combination with low dose aspirin is recommended for
patients with mechanical prosthetic valves who suffer a systemic embolism despite adequate
anticoagulant therapy.
Educational objective:
Know the INR goals for mechanical prosthetic valves.
40% of people answered this question correctly;

3/19/2011 Item 16 of 44
usmleworld.com/Step3//qstShow.as 1/1

A 68-year-old Caucasian man comes to the physician because of recurrent episodes of
dizziness. His medical problems include hypertension, coronary artery disease (CAD),
asthma, diabetes mellitus type-2, and hypercholesterolemia. He is retired and lives alone. He
does not use tobacco, alcohol, or drugs. His family history is not significant. His medications
include aspirin, enalapril, isosorbide mononitrate, simvastatin, glyburide, and metformin. His
temperature is 36.7 C (98 F), blood pressure is 125/80 in decubitus and 100/70 mmHg in the
standing position, pulse is 86/min. in decubitus and 92/min. while standing, and respirations
are 14/min. Examination shows no abnormalities. Which of the following is the most effective
strategy to manage this patient's symptoms?
A. Stop enalapril.
B. Stop isosorbide.
C. Stop simvastatin.
D. Order a CT scan of the head .
E. Start meclizine.

Explanation:
Polypharmacy is defined as the use of five or more drugs in a single patient for different
medical conditions. It is very common in the elderly. Knowledge of the various drug
interactions, as well as their adverse effects, is very essential to the primary care provider. For
instance, although the usual treatment of coronary artery disease (CAD) involves beta-
blockers, this was not given to the patient due to his other medical problems (diabetes and
asthma).
This patient has multiple medical conditions, for which he has the following medications:
aspirin, enalapril, isosorbide mononitrate, simvastatin, glyburide, and metformin. He currently
complains of dizziness and has orthostatic hypotension, which is associated with the use of
diuretics, alpha-blockers, or nitrates. Isosorbide dinitrate use has been associated to
dizziness (in 4% of patients) and to orthostatic hypotension (in 1% of patients). Isosorbide use
must therefore be stopped in this patient.
(Choice A) Enalapril is an antihypertensive drug which has not been associated with
orthostatic hypotension as frequently as the abovementioned drugs.
(Choice C) Simvastatin use can cause dizziness as frequently as a placebo; furthermore, its
use has not been associated with orthostatic hypotension.
(Choice D) The diagnosis has been clearly established. There is no need for further studies,
such as a CT scan or MRI of the head, and a neurology referral.
(Choice E) Meclizine can have some symptomatic benefit in patients with vestibular disease.
It will not be helpful in cases of dizziness associated with orthostatic hypotension.
Educational Objective:

3/19/2011 Item 17 of 44
usmleworld.com/Step3//qstShow.as 1/2
Polypharmacy is frequent in the elderly. Orthostatic hypotension is not uncommon in this
setting and is usually related to the use of diuretics, alpha-blockers, or nitrates.
81% of people answered this question correctly;
3/19/2011 Item 17 of 44
usmleworld.com/Step3//qstShow.as 2/2

A 63-year-old Caucasian man comes to the office for a routine follow-up visit. He notes
bilateral knee osteoarthritis that is well controlled with acetaminophen. He takes no other
medications. He does not use tobacco, alcohol, or drugs. His blood pressure is 132/78 mmHg
and his heart rate is 82/min. Examination reveals a 2/6 ejection-type systolic murmur at the
right second intercostal space. His lungs are clear and there is no peripheral edema. He
denies chest pain, shortness of breath, palpitations, or syncope. The patient is referred for an
echocardiogram, which reveals mild aortic sclerosis and left ventricular dilatation with an
ejection fraction of 35%. There are no regional wall motion abnormalities. Which of the
following is most appropriate for this patient at this point?
A. Digoxin
B. Enalapril
C. Isosorbide mononitrate
D. Spironolactone
E. Strict salt restriction

Explanation:
Left ventricular dysfunction is a common problem. Many affected patients have signs and
symptoms of heart failure, but a considerable number of them do not despite a low ejection
fraction. Population studies have demonstrated that asymptomatic left ventricular dysfunction
is at least as prevalent as symptomatic heart failure. Asymptomatic left ventricular dysfunction
may result from a myocardial infarction, but in patients without a clear precipitating cause,
further evaluation is necessary as many will progress to symptomatic heart failure. In this
particular patient, the absence of a regional wall motion abnormality on echocardiogram
makes a prior myocardial infarction unlikely.
The most important medication for patients with asymptomatic left ventricular dysfunction is an
ACE inhibitor, which can delay the onset of symptomatic heart failure and potentially prolong
survival. Initiation of a beta-blocker is also reasonable, although the evidence for beta-blockers
is not quite as strong as it is for ACE inhibitors in this setting.
(Choice A) Digoxin can be useful in the treatment of symptomatic heart failure, although it has
not been shown to delay onset of heart failure in patients with asymptomatic left ventricular
dysfunction.
(Choice C) Isosorbide mononitrate is often used for angina, but this patient has no chest pain.
Nitrates are sometimes used along with hydralazine to treat symptomatic heart failure, but they
are not used to treat asymptomatic left ventricular dysfunction.
(Choice D) Spironolactone is not recommended for the treatment of asymptomatic left
ventricular dysfunction.
(Choice E) This patient has no symptoms of heart failure, making it unlikely that he is fluid-
overloaded and therefore rendering salt restriction unnecessary.

3/19/2011 Item 18 of 44
usmleworld.com/Step3//qstShow.as 1/2
Educational objective:
While left ventricular dysfunction commonly presents with signs and symptoms of heart failure,
a significant percentage of people with low ejection fractions are asymptomatic. ACE
inhibitors, and possibly beta-blockers, should be given to these patients to delay progression
to symptomatic heart failure.
48% of people answered this question correctly;
3/19/2011 Item 18 of 44
usmleworld.com/Step3//qstShow.as 2/2

A 68-year-old white male with a history of paroxysmal atrial fibrillation, hypertension, and
hypercholesterolemia is brought to the physician's office by his daughter because of nausea,
anorexia, and confusion. His daughter states that he was admitted at an outside hospital two
weeks ago because of uncontrolled hypertension and palpitations. He was started on a new
medication. Prior to the recent hospitalization, he was taking digoxin, warfarin, simvastatin,
and hydrochlorothiazide. His physical examination shows an irregular heart rate, but is
otherwise unremarkable. His blood pressure is 130/80 mmHg, pulse rate is 50/minute and
irregular, temperature 37.2C (99F), and respirations are 14/minute. Which of the following is
the most likely medication that is contributing to his symptoms?
A. Atenolol
B. Enalapril
C. Clonidine
D. Valsartan
E. Verapamil

Explanation:
This patient has developed classic early features of digoxin toxicity. Digoxin toxicity is
characterized by nausea, vomiting, anorexia, fatigue, confusion, visual disturbances, and
cardiac abnormalities. The most likely explanation of his digoxin toxicity is the recent addition
of verapamil. Verapamil inhibits the renal tubular secretion of digoxin, resulting in almost 70-
100% increase in serum digoxin levels. The other medications, which can cause digoxin
toxicity, are quinidine, amiodarone, and spironolactone.
(Choice B) Even though captopril can cause a slight elevation of digoxin levels, the other ACE
inhibitors and ARBs usually do not cause digoxin toxicity.
(Choice A) Beta-blockers may cause bradycardia when used with digoxin because of the
added negative chronotropic action; however, this usually does not result in digoxin toxicity.
(Choices C and D) The other medications in the other choices do not cause significant
digoxin toxicity.
Educational Objective:
Verapamil, quinidine, and amiodarone can potentially cause digoxin toxicity.
28% of people answered this question correctly;

3/19/2011 Item 19 of 44
usmleworld.com/Step3//qstShow.as 1/1

A 62-year-old Caucasian man comes to the office for the evaluation of progressive shortness
of breath for the last two months. He used to walk two miles a day without any difficulty. He has
recently noticed that he gets "out of breath" and tires easily after walking a few blocks or even
after walking upstairs. He denies any history of fever, chills, cough, weight loss, or night
sweats. Which of the following is the best initial method to diagnose congestive heart failure?
A. History and physical examination
B. Electrocardiogram
C. Chest radiograph
D. Plasma B-type natriuretic peptide
E. 2D-echocardiogram

Explanation:
Congestive heart failure is a clinical syndrome that results from impaired or inadequate
ventricular emptying (systolic dysfunction) or impaired ventricular relaxation (diastolic
dysfunction). It is characterized by specific symptoms that are either due to reduced cardiac
output (fatigue, weakness) or excessive fluid retention (peripheral or pedal edema, dyspnea).
All symptoms are exacerbated by exertion, and the degree of these symptoms helps in
assessing the severity of heart failure.
Heart failure is primarily a clinical diagnosis and is based on a detailed history and physical
examination. A detailed history and physical examination can also provide clues to the
specific cause of heart failure. The major criteria include the presence of paroxysmal
nocturnal dyspnea, orthopnea, raised jugular venous pressure, pulmonary rales, presence of
third heart sound, increased cardiac silhouette, and pulmonary vascular congestion on chest x-
ray. The minor criteria include the presence of bilateral lower extremity edema, nocturnal
cough, dyspnea on exertion, tachycardia, presence of pleural effusion, and hepatomegaly. The
diagnosis of heart failure is based on the presence of two major criteria or one major and two
minor criteria, provided the minor criteria cannot be explained by the presence of any
concurrent medical illness.
(Choice B) An electrocardiogram may provide clues to the specific cause of heart failure, but
is not helpful for the initial diagnosis of congestive heart failure.
(Choice C) Chest radiograph findings suggestive of congestive heart failure include
cardiomegaly, pulmonary vascular congestion (especially in the upper lobes), Kerley B lines
and bilateral pleural effusions. The presence or absence of such findings will help to
differentiate heart failure from dyspnea secondary to a primary pulmonary pathology.
(Choice D) Plasma B type natriuretic peptide levels are useful at times to differentiate
between cardiogenic versus non-cardiogenic causes of dyspnea; however, the history and
physical examination are primarily used to diagnose congestive heart failure initially.
(Choice E) 2D-echocardiogram is very helpful in confirming the diagnosis, and also provides

3/19/2011 Item 20 of 44
usmleworld.com/Step3//qstShow.as 1/2
clues to the cause and severity of heart failure; however, a history and physical examination
should be initially performed in all patients prior to obtaining the 2D-echocardiogram.
Educational Objective:
Obtaining a detailed history and physical examination is the initial method of choice to
diagnose congestive heart failure prior to obtaining any confirmatory diagnostic tests.
73% of people answered this question correctly;
3/19/2011 Item 20 of 44
usmleworld.com/Step3//qstShow.as 2/2

A 24-year-old male presents to your office with a one-week history of night sweats and
progressive fatigue. He also complains of palpitations and shortness of breath on moderate
exertion. His past medical history is insignificant. His blood pressure is 120/80 mmHg, pulse
is 98/min, temperature is 38.9C (102F) and respirations are 24/min. Lung auscultation reveals
scattered bilateral rales. A grade II/VI blowing systolic murmur that increases with inspiration is
heard over the lower part of the left sternal border. The chest x-ray demonstrates several
circumscribed round pulmonary infiltrates on both sides. Transesophageal echocardiography
reveals small vegetations confined to the tricuspid valve. What is the most likely cause of this
patient's problem?
A. Rheumatic fever
B. Rheumatoid arthritis
C. Illicit drug use
D. SLE
E. Congenital valve defect

Explanation:
The clinical scenario is highly suggestive of infective endocarditis involving the right cardiac
chambers. Two very important clues to the diagnosis are: evidence of right-sided cardiac
involvement (systolic murmur with inspiratory accentuation and tricuspid vegetation on
echocardiography) and evidence of septic pulmonary emboli (present in 75% of patients with
right-sided endocarditis). Peripheral manifestations and heart failure are uncommon in these
patients, and a vast majority are IV drug users. The endocarditis seen in drug users is different
from non-drug users in that the tricuspid valve is involved in 30 to 70% of endocarditis cases in
injection drug users. Overall, right-sided endocarditis (predominantly involving the tricuspid
valve) is far less common than left-sided disease.
(Choices A and D) Rheumatic fever and SLE predominantly involve left cardiac structures.
Isolated lesions of right cardiac valves are quite uncommon with these diseases.
Educational Objective:
A vast majority of patients with right-sided endocarditis are IV drug users.
86% of people answered this question correctly;

3/19/2011 Item 21 of 44
usmleworld.com/Step3//qstShow.as 1/1

A 68-year-old male comes to your office complaining of chest pain for the past three months
that is worse with exercise, radiates down his left arm, and causes shortness of breath. His
activity has been limited secondary to the pain. He has a history of type 2 diabetes mellitus,
hypertension, hyperlipidemia, chronic renal insufficiency, morbid obesity, and is a current
smoker. You determine that the patient is suffering from angina and you schedule him for a
cardiac catheterization. It is determined that he has a 90% blockage of his left main coronary
artery and should probably undergo coronary artery bypass. He does not pay for any of his
own healthcare because he is a Medicare recipient. Given his comorbid conditions his risk of
a poor outcome is reasonably increased. What should you tell him about coronary artery
bypass surgery?
A. "You are not a candidate for coronary artery bypass because you are on Medicare."
B. "Medicare will not pay for your cardiac bypass because you are a current smoker."
C. "You should not choose to have cardiac bypass surgery because you are at high risk
for complications."
D. "You have an increased risk of morbidity and mortality with coronary artery bypass
because of your comorbid medical conditions."
E. "It is not an efficient use of medical resources to offer you coronary artery bypass."

Explanation:
Physicians currently practice in an economically constrained environment with constant
pressure to conserve medical resources. However, you must act in the best interest of the
patient and provide this patient with all possible options. In several countries with nationalized
healthcare systems, older patients and patients with certain comorbidities may not be eligible
for coronary artery bypass. In the United States, however, a Medicare patient is most likely
eligible for the procedure and you should explain the risks and benefits of such a procedure
taking into account the patients comorbidities.
(Choice A) Medicare is a form of government healthcare offered to United States citizens
over 65 years of age. Medicare will pay for coronary artery bypass surgery when indicated.
(Choice B) Currently, Medicare does not withhold benefits from people because of smoking,
morbid obesity, or other potentially reversible causes of chronic illness.
(Choice C) The physician is responsible to fully inform this patient about the options available
to him, including the benefits and risks of each possible choice. The physician, however,
should not make the decision for the patient.
(Choice E) Whether or not coronary bypass surgery for this patient is an efficient use of
medical resources is not of immediate concern in this case. The physician must first act in the
best interest of the patient.
Educational objective:
Physicians must first be advocates for their patients, acting in the patients best interest to

3/19/2011 Item 22 of 44
usmleworld.com/Step3//qstShow.as 1/2
provide appropriate medical care.
89% of people answered this question correctly;
3/19/2011 Item 22 of 44
usmleworld.com/Step3//qstShow.as 2/2

A 44-year-old man comes to the physician with a two week history of palpitations. He denies
chest pain, orthopnea, dizziness, syncope, or shortness of breath. He has a history of
migraines and kidney stones. He does not use tobacco, alcohol, or illicit drugs. His family
history is significant for liver cancer and renal failure. His heart rate is 132/min and his blood
pressure is 124/82 mmHg. His BMI is 31 kg/m
2
. Physical examination reveals no murmurs.
His lungs are clear and there is no peripheral edema. Electrocardiogram (ECG) shows atrial
fibrillation with a rapid ventricular response. His CBC, basic metabolic panel, TSH, and chest
x-ray are normal. A transthoracic echocardiogram shows no valvular abnormalities, normal left
ventricular size and function, and mild enlargement of the left atrium. The patient is started on
metoprolol and reports improvement of his symptoms after one week. Repeat EKG shows
atrial fibrillation with a ventricular rate of 77/min at rest. Which of the following is the most
appropriate therapy at this point?
A. Amiodarone
B. Aspirin
C. Digoxin
D. Diltiazem
E. Warfarin

Explanation:
This patient has received a fairly comprehensive workup for atrial fibrillation (AF). Treatment of
AF is generally begun at the time of initial evaluation. Patients who are hemodynamically
unstable should receive urgent cardioversion. Stable patients can be treated medically to
control the ventricular rate, with agents like beta-blockers, verapamil, or digoxin. Patients will
often spontaneously convert back to sinus rhythm after treatment with beta-blockers or other
similar medications.
Anticoagulation is the other key component of AF management, assuming the patient does
not quickly, spontaneously convert back to normal sinus rhythm. A scoring system known as
the CHADS2 system can be used to help determine a patient's risk of stroke and whether
anticoagulation requires warfarin or if aspirin is sufficient. One point is given for a history of
congestive heart failure, hypertension, diabetes mellitus, or age over 75 years and two points
are given if the patient has had a prior ischemic stroke or transient ischemic attack. Patients
with a CHADS2 score of 0 can be treated with aspirin and those with a score of 3 or higher
should be treated with warfarin. A score of 1 or 2 is considered a gray area where either may
be appropriate. This relatively young, relatively healthy patient has a score of 0, meaning that
aspirin therapy is sufficient.
(Choice A) Amiodarone is sometimes used as a component of AF management to
pharmacologically convert the patient's rhythm back to sinus. However, several large clinical
trials have found that both rate and rhythm control are acceptable treatment strategies for AF.
Amiodarone is associated with numerous side effects and thus should be avoided since this
patient's heart rate is already well-controlled.

3/19/2011 Item 23 of 44
usmleworld.com/Step3//qstShow.as 1/2
(Choice C) Digoxin is usually only used for rate control in patients with heart failure.
Otherwise, beta-blockers and non-dihydropyridine calcium channel blockers are generally
preferred.
(Choice D) This patient could have initially been started on diltiazem as opposed to
metoprolol. However, his rate is now controlled with metoprolol and there is no need to add a
second rate control medication.
(Choice E) Warfarin would be considered for this patient if he had a history of heart failure,
hypertension, diabetes, age greater than 75, or prior ischemic stroke or TIA.
Educational objective:
The initial step in managing atrial fibrillation in a hemodynamically stable patient is rate control
with a beta-blocker or non-dihydropyridine calcium channel blocker. Anticoagulation is also
needed in most patients. Aspirin alone is sufficient for patients deemed to be at low risk for
stroke based on the CHAD2 scoring system, whereas warfarin is needed for patients with
higher scores.
26% of people answered this question correctly;
3/19/2011 Item 23 of 44
usmleworld.com/Step3//qstShow.as 2/2

A 62-year-old man comes to the physician's office with complaints of intermittent squeezing
chest pain and shortness of breath for the past two months. His symptoms are worsened by
exertion and relieved by rest. He denies orthopnea, cough, or syncope. His other medical
problems include hypertension, type 2 diabetes mellitus, and gastroesophageal reflux
disease. His medications are hydrochlorothiazide, metformin, sitagliptin, enalapril, aspirin,
and ranitidine (as needed for reflux symptoms). He has smoked one pack of cigarettes daily
for 35 years and drinks 1-2 bottles of beer daily. His blood pressure is 152/81 mmHg and his
heart rate is 64/min, regular. His BMI is 41 kg/m
2
. His physical examination is remarkable for
1+bilateral pitting edema. An electrocardiogram in the office shows prominent non-specific
ST segment and T wave changes. Echocardiogram reveals anteroseptal wall hypokinesis,
and mild left ventricular dilation with a left ventricular ejection fraction of 20-25%. What is the
most appropriate next step in the management of this patient?
A. Amlodipine and loop diuretics
B. Cardiac catheterization
C. Clopidogrel and enoxaparin
D. EKG stress test
E. Stress echocardiography

Explanation:
This patient's history of chest pain that is worsened by exertion and relieved by rest is highly
suggestive of stable angina, particularly in light of the cardiac disease detected on his
electrocardiogram and echocardiogram. The American College of Cardiology/American
Heart Association (ACC/AHA) guidelines for management of chronic stable angina state that
coronary angiography should be used to risk stratify patients with evidence of heart failure, a
high likelihood of severe coronary artery disease, or disabling anginal symptoms. This
patient's markedly decreased ejection fraction and high likelihood of coronary artery disease
would indicate coronary angiography at this time, with revascularization performed depending
on the degree and distribution of disease.
(Choice A) Amlodipine is a second-line medical treatment of stable angina in patients where
beta-blockers are contraindicated. Loop diuretics could be used for this patient's heart failure,
but would be unlikely to help his angina.
(Choice C) Clopidogrel could be considered for this patient if aspirin were contraindicated.
Enoxaparin would be unnecessary.
(Choice D) An EKG stress test would not be beneficial for this patient given his high risk of
underlying coronary artery disease and the presence of resting EKG abnormalities.
(Choice E) Stress echocardiography is more beneficial in the risk stratification of patients
deemed to be at intermediate risk of coronary artery disease.

3/19/2011 Item 24 of 44
usmleworld.com/Step3//qstShow.as 1/2
Educational objective:
A history of chest pain that is worsened by exertion and relieved by rest is highly suggestive of
stable angina. Stress testing is most beneficial to risk stratify patients deemed at intermediate
risk of coronary artery disease, whereas patients with a high risk or underlying heart failure
should proceed directly to coronary angiography.
63% of people answered this question correctly;
3/19/2011 Item 24 of 44
usmleworld.com/Step3//qstShow.as 2/2

A 25-year-old, Caucasian, female athlete comes to see you in the office with complaints of
episodes of "a racing heart". The episodes occur almost every day. These last only for a few
seconds and occur both during rest and exercise. There is no history of associated dizziness
or loss of consciousness with these episodes. She admits to smoking one pack of cigarettes
daily, but denies any significant alcohol or illicit drug use. She does not have any other past
medical history. Her physical examination reveals the presence of a midsystolic click without
any murmurs. An EKG done in the office does not reveal any abnormalities. Holter monitoring
reveals the presence of five to seven episodes of nonsustained monomorphic ventricular
tachycardia in a day. Which of the following is the most likely cause of the above findings?
A. Electrolyte abnormality
B. Congenital long QT syndrome
C. Cocaine abuse
D. Structural heart disease

Explanation:
Nonsustained ventricular tachycardia is defined as the presence of three or more consecutive
ventricular beats with a heart rate greater than 120 beats/min, and with the episode lasting for
less than 30 seconds. It is seen most commonly in patients with some form of structural heart
disease, possibly because structural abnormalities lead to ventricular arrhythmia. Some
structural abnormalities that can lead to nonsustained ventricular tachycardia include scarred
myocardium from prior myocardial infarctions (coronary artery disease), ventricular
hypertrophy (LVH or hypertrophic obstructive cardiomyopathy), dilated left ventricle (dilated
cardiomyopathy), and valvular abnormalities such as mitral valve prolapse. Although
nonsustained ventricular tachycardia can also be seen in patients without any evidence of
structural heart disease, it is important to recognize that structural heart disease is the most
common cause, and a 2D-echocardiogram and stress test should be obtained to rule out
ischemia.
Based on the Holter monitor findings, the patient's palpitations is due to the presence of
nonsustained monomorphic ventricular tachycardia. This is most likely secondary to the
presence of structural heart disease, which is most likely due to mitral valve prolapse
(midsystolic click).
(Choice B) A prolonged QT interval, either due to acquired or congenital QT syndrome, can
cause nonsustained polymorphic ventricular tachycardia (also known as torsade de
pointes); however, the baseline EKG in the patient did not reveal any evidence of a prolonged
QT interval.
(Choice A) Electrolyte abnormalities (hypokalemia or hypomagnesemia) can also cause
polymorphic ventricular tachycardia; however, structural heart disease still remains as the
most common cause of nonsustained ventricular tachycardia in patients across all age groups.
(Choice C) There is no history of cocaine abuse in the patient.

3/19/2011 Item 25 of 44
usmleworld.com/Step3//qstShow.as 1/2
Educational Objective:
Structural heart disease is the most common cause of nonsustained ventricular tachycardia
and should be ruled out before searching for other causes.
68% of people answered this question correctly;
3/19/2011 Item 25 of 44
usmleworld.com/Step3//qstShow.as 2/2

A 62-year-old postmenopausal African-American woman comes to see you in the office. She
tells you that she gets a sensation of tightness over her anterior chest after about 15-20
minutes of brisk walking. The tightness comes on gradually, and then resolves over the next 5-
10 minutes with rest. These episodes are sometimes associated with some difficulty in
breathing. The symptoms have been present for the last three months. The patient has a
history of hypertension, hyperlipidemia, and a 50-pack-year history of smoking. Her
medications include aspirin, metoprolol, hydrochlorothiazide, and simvastatin. She denies any
symptoms while she is in the office. Her physical examination, including her vital signs, is
unremarkable. The echocardiogram done in the office is also normal. Which of the following is
the most appropriate next step in the management of this patient?
A. Reassure the patient that she is on adequate treatment for her condition.
B. Prescribe nitroglycerin and schedule her for an exercise stress test.
C. Admit the patient to the hospital for further diagnostic workup.
D. Refer her to a cardiologist for coronary angiography.

Explanation:
The patient in the above vignette has classic symptoms of stable angina pectoris. She has
multiple risk factors for coronary artery disease, which are: age greater than 55 years,
hypertension, hyperlipidemia, and extensive smoking history. This, along with the typical
history of angina pectoris, is highly predictive for the presence of coronary artery disease.
All patients with coronary artery disease and stable angina should be referred for an exercise
stress test for further risk stratification (PLEASE do not confuse with a diagnostic stress test
used in intermediate probability patients!). Patients with a low risk treadmill score (able to
exercise more with a normal EKG) have less than 1% annual mortality rate. On the other hand,
patients who have a high-risk treadmill score (presence of chest pain or EKG changes with
minimal exercise) have greater than 3% annual mortality rate and require more aggressive
management. This patient should therefore be referred for an exercise stress test to obtain
further prognostic information.
(Choice C) An exercise stress test for prognostic information can be obtained on an
outpatient basis. There is no need to admit the patient to the hospital, unless she is having
active chest pain or unstable angina.
(Choice D) Coronary angiography is not routinely done in patients with chronic stable angina.
The two main indications for coronary angiography are patients with high-risk criteria on
exercise stress testing, and patients who continue to have symptoms despite maximal
medical therapy.
Educational Objective:
All patients with chronic stable angina should undergo an exercise stress test for prognostic
and risk stratification.

3/19/2011 Item 26 of 44
usmleworld.com/Step3//qstShow.as 1/2
82% of people answered this question correctly;
3/19/2011 Item 26 of 44
usmleworld.com/Step3//qstShow.as 2/2

A 32-year-old African-American woman comes to the office with a complaint of progressive
exertional dyspnea. She has a history of seizure disorder, hypertension, idiopathic dilated
cardiomyopathy, and angioedema with angiotensin converting enzyme inhibitor use. Her
ejection fraction on a recent 2 D-echocardiogram was estimated to be 15 - 20%. Her
medications include phenytoin (Dilantin), furosemide, carvedilol, spironolactone, and digoxin.
Her blood pressure is 140/90 mmHg, pulse is 65/min, and respirations are 18/min. Her pulse
oximetry is 98% at room air. Physical examination reveals normal jugular venous pressure,
clear lung fields, normal first and second heart sounds, and a distinct third heart sound. She is
started on a hydralazine and isosorbide dinitrate combination for congestive heart failure.
Which of the following statements is true regarding the initiation of this combination in this
patient?
A. It may lead to significant weight loss in the first few months
B. The patient should report any new onset of flu-like symptoms
C. It should be stopped for 3 months prior to planning a pregnancy
D. The patient should get her liver function test done at baseline and then every 6
months while on the drug
E. Her dilantin dose should be increased as hydralazine can lower seizure threshold

Explanation:
The hydralazine and isosorbide combination is frequently used in the management of
congestive heart failure in patients intolerant of ACE inhibitors. Patients should be informed
regarding the possible adverse reactions that may result from the use of these drugs, and
instructed to stop the medication if they develop any of the these reactions.
The development of drug-induced lupus-like syndrome has been associated with the use of
hydralazine. Other drugs that have been associated with lupus include procainamide,
penicillamine, isoniazid, minocycline, diltiazem, methyldopa, chlorpromazine, and interferon-
alfa. Patients with drug-induced lupus can develop a variety of flu-like symptoms such as fever,
malaise, arthralgias, myalgias, and maculopapular facial rash. They can also develop
lymphadenopathy, splenomegaly, pleurisy, and pericarditis. An important immunological
marker of drug-induced lupus is the presence of antihistone antibodies.
(Choice A) Hydralazine use does not lead to weight loss. On the contrary, its use can cause
weight gain by causing fluid and sodium retention. The dose of diuretics may have to be
increased to offset the fluid retention caused by hydralazine use.
(Choice C) Hydralazine is one of the few antihypertensive medications that can be used
safely in a pregnant patient. Some of the other drugs that are safe in pregnancy are
methyldopa, labetalol, and long-acting calcium channel blockers.
(Choice D) Hydralazine has no hepatic toxicity; therefore, monitoring of liver function tests is
not required.

3/19/2011 Item 27 of 44
usmleworld.com/Step3//qstShow.as 1/2
(Choice E) Hydralazine does not lower the seizure threshold, and can be safely used in
patients with a history of seizure disorder.
Educational objective:
It is important to recognize the syndrome of drug-induced lupus caused by hydralazine.
Stopping the medication is usually sufficient to resolve the symptoms.
24% of people answered this question correctly;
3/19/2011 Item 27 of 44
usmleworld.com/Step3//qstShow.as 2/2

A 76-year-old African-American man comes to see you in the office for a follow-up visit. He
was recently admitted to a local hospital six weeks ago due to congestive heart failure (CHF)
exacerbation, for which he was treated with intravenous diuretics for five days. On this visit, he
says that he has been doing well after his discharge. He denies any new symptoms. He has a
past medical history of hypertension, diabetes mellitus, coronary artery disease, coronary
artery bypass grafting five years ago, ischemic cardiomyopathy with an ejection fraction of 20-
25%, benign prostate hyperplasia, and osteoarthritis. His medications include aspirin,
metoprolol, lisinopril, digoxin, simvastatin, NPH insulin, spironolactone, furosemide, and
acetaminophen as needed for his symptoms. His temperature is 36.7C, blood pressure is
106/72 mmHg, respiratory rate is 16/min, and heart rate is 72/min. His lung examination
reveals minimal basilar crackles at both lung bases. Cardiovascular examination reveals
regular heart sounds and an audible S3 over the cardiac apex. The rest of the physical
examination is unremarkable. His laboratory examination done today reveals a plasma
sodium concentration of 115 mEq/L. His plasma sodium concentration six weeks ago was
123 mEq/L. Which of the following is the most appropriate way to treat his hyponatremia?
A. Water restriction
B. Intravenous diuretics
C. Hypertonic saline
D. Isotonic saline
E. Oral salt tablets

Explanation:
Excess free water normally causes a fall in plasma osmolality, which suppresses the release
of the antidiuretic hormone (ADH). This leads to the formation of dilute urine, and prevents the
development of hyponatremia. When this physiologic mechanism fails, hyponatremia is said to
occur, wherein there is a relative excess of free water in the body in relation to sodium
concentration. Strictly speaking, hyponatremia is defined as a plasma sodium concentration of
less than 135 mEq/L. This can develop in patients due to a marked increase in water intake
which overwhelms the excretory capacity of the kidneys. Other frequent causes are: impaired
water excretion by the kidneys due to impaired renal function, as well as persistent and
inappropriate release of ADH.
Congestive heart failure (CHF) accompanied by hyponatremia is a very interesting clinical
situation. In such a case, the decreased cardiac output and systemic blood pressure
decreases the perfusion pressure at the carotid sinus baroreceptors and afferent renal
arterioles, despite the existing volume overload. This decreased perfusion pressure leads to
the stimulation of the renin-angiotensin system and the secretion of ADH, which causes further
retention of water and gradual development of the hyponatremia. Affected patients are usually
asymptomatic. Treatment involves gradual correction of the electrolyte abnormality in order to
avoid the development of irreversible neurological damage. Water restriction to lessen the
patient's total output is the mainstay of therapy.

3/19/2011 Item 28 of 44
usmleworld.com/Step3//qstShow.as 1/2
(Choice B) The patient appears to have compensated CHF. At this point, intravenous
diuretics are not indicated and may even worsen the patient's hyponatremia.
(Choice C) The use of hypertonic saline to increase the plasma sodium concentration is
usually indicated only for patients with symptomatic hyponatremia.
(Choice D, E) Isotonic saline or oral salt tablets are used in patients with true volume
depletion. Patients with CHF have volume excess/overload; therefore, giving isotonic saline or
an increase in dietary salt is not indicated.
Educational Objective:
Water restriction is the mainstay of therapy for hyponatremia in patients with advanced CHF.
57% of people answered this question correctly;
3/19/2011 Item 28 of 44
usmleworld.com/Step3//qstShow.as 2/2

A 58-year-old Caucasian male presents to the emergency department complaining that he is
coughing up blood and has trouble breathing, especially when he is engaged in activity. His
medical history is significant for diverticulosis and a surgically repaired inguinal hernia. He is
married with two children and does not smoke cigarettes, drink alcohol, or use recreational
drugs. Vital signs include temperature of 37.0C (98.6F), blood pressure of 126/82 mm Hg,
pulse of 76/min, and respirations of 14/min. His head, eye, ear, nose, and throat examination
are unremarkable. Chest auscultation demonstrates crackles in the middle and upper lobes of
both lungs. What will cardiac auscultation reveal if there is prolapse of his mitral valve?
A. Holosystolic murmur
B. Systolic crescendo-decrescendo ejection murmur
C. Mid systolic click followed by late systolic murmur
D. Loud S1 and low-pitched diastolic murmur loudest at apex
E. Muffled heart sounds and low-pitched diastolic murmur loudest at apex

Explanation:
Mitral valve prolapse (MVP) is found in 2-5% of adults and is the most common valvular
abnormality among the residents of industrialized nations. The condition is typically caused by
redundant valve tissue at the valve ring (Barlow's syndrome) and may be associated with mild
mitral regurgitation, though left ventricular function remains normal. MVP causes a mid-to-late
systolic click that may be followed by a late systolic murmur most easily detected over the left
ventricle (Choice C).
An apical holosystolic murmur (Choice A) may be due to mitral regurgitation, which is most
often secondary to myocardial infarction, mitral valve prolapse, rheumatic heart disease, or
coronary artery disease.
A systolic crescendo-decrescendo ejection murmur in the left upper sternal border (Choice B)
characterizes pulmonic stenosis, which is most often secondary to congenital malformation,
rheumatic heart disease, or carcinoid.
Mitral stenosis produces a low-pitched, rumbling diastolic murmur best heard over the apex
when the patient is lying in a left lateral decubitus position (Choice D).
Muffled heart sounds (Choice E) are a classic indication of cardiac tamponade, and are not
commonly associated with heart murmurs.
Educational Objective:
Mitral valve prolapse causes a mid-to-late systolic click that may be followed by a late systolic
murmur most easily detected over the left ventricle.
80% of people answered this question correctly;

3/19/2011 Item 29 of 44
usmleworld.com/Step3//qstShow.as 1/1

A 17-year-old African American male comes to the walk-in clinic complaining of frequent
headaches. He plays basketball on weekends but over the last 3 months he has noticed
decreased exercise tolerance due to shortness of breath. He also describes lower extremity
swelling. His blood pressure is 210/120 mmHg and his heart rate is 85/min. Which of the
following is the most likely cause of this patient's condition?
A. Adrenal cortical tumor
B. Adrenal medullary tumor
C. Congenital heart abnormality
D. Renal artery stenosis
E. Renal parenchymal disease

Explanation:
Hypertension is a common finding in outpatient clinics, with essential hypertension being the
most common variety. However, this new-onset severe hypertension in a young patient is
suggestive of a secondary cause of hypertension. The diagnosis is important to make
because many cases of secondary hypertension have an underlying cause that is easily
treatable. Prolonged untreated hypertension in young patients can lead to premature
atherosclerosis and cardiovascular disease. The most common cause of secondary
hypertension in young patients is renal parenchymal disease. This patient's shortness of
breath and peripheral edema may be secondary to volume overload from renal failure. The
diagnosis can be easily confirmed with a chemistry panel, although further evaluation to
determine the exact cause of the renal dysfunction would likely be required. Endocrine causes
including pheochromocytoma, Cushing's syndrome, and mineralocorticoid excess are less
likely possibilities. After renal parenchymal disease and endocrine etiologies, renovascular
disease is the third leading cause of secondary hypertension.
(Choice A) Adrenal cortical tumors may secrete excess corticosteroids or mineralocorticoids
that may lead to secondary hypertension.
(Choice B) Adrenal medullary tumors such as pheochromocytoma typically result in excess
catecholamine secretion that may result in secondary hypertension.
(Choice C) Most forms of congenital heart disease do not cause hypertension. Coarctation of
the aorta is an exception, with the typical finding being hypertension in the upper extremities
with low blood pressure in the lower extremities.
(Choice D) Renal artery stenosis results in decreased renal blood flow and subsequent
increase in renin, which leads to hypertension. Renovascular hypertension is much less
common than renal parenchymal disease as a secondary cause of hypertension.
Educational objective:
A secondary cause of hypertension should be suspected in a very young or very old patient
presenting with new-onset severe hypertension. Renal parenchymal disease is the most

3/19/2011 Item 30 of 44
usmleworld.com/Step3//qstShow.as 1/2
common cause of secondary hypertension in young patients.
43% of people answered this question correctly;

3/19/2011 Item 30 of 44
usmleworld.com/Step3//qstShow.as 2/2

A 60-year-old male with hypertension and hypercholesterolemia comes for a follow-up visit.
His blood pressure in the office is 160/100 mmHg, and his readings are consistently elevated
when he checks his blood pressure at home. An antihypertensive agent is added to his current
regimen. He takes a brief vacation in Florida, and calls after a few days because he has
developed an erythematous rash on the exposed parts of his body. Which of the following
agents is most likely responsible for his rash?
A. Furosemide
B. Hydrochlorothiazide
C. Lisinopril
D. Amlodipine
E. Metoprolol

Explanation:
The patient is likely to have a photosensitivity reaction, which is most commonly associated
with the use of hydrochlorothiazide. Thiazides are sulfonamides; therefore, these can cause
photosensitivity or generalized dermatitis. Treatment of the rash includes discontinuation of
hydrochlorothiazide, use of sunscreen, and avoidance of sun exposure.
(Choice A) Furosemide can cause a Stevens-J ohnson syndrome type of reaction that is seen
with the use of sulfonamide antibiotics. Urticarial rash can also occur; however, furosemide is
not usually recommended for the management of hypertension alone.
(Choice C) Lisinopril is an ACE inhibitor and can be associated with angioedema and
urticaria. Photosensitivity rash is uncommon. Occasionally, lisinopril can lead to aggravation of
psoriatic rash.
(Choice D) Amlodipine therapy can be associated with significant fluid retention and urticarial
rash.
(Choice E) Metoprolol can cause urticaria in upto 5% of patients. Photosensitivity rash can
occur, but it is much more uncommon compared to hydrochlorothiazide.
Educational objective:
Thiazides are sulfonamides; therefore, hydrochlorothiazide use can cause photosensitivity
rash, and treatment of this rash includes discontinuation of the thiazide, use of sunscreen, and
avoiding sun exposure.
41% of people answered this question correctly;

3/19/2011 Item 31 of 44
usmleworld.com/Step3//qstShow.as 1/1

A 55-year-old Caucasian woman comes to the office for a routine follow-up examination. Her
only complaints at this time regard some aches and pains in various muscles. She has been
diagnosed with irritable bowel syndrome, fibromyalgia, depression, and rheumatic heart
disease. Current medications include fluoxetine and ibuprofen. Vital signs include temperature
of 37.0C (98.6F), blood pressure of 128/86 mm Hg, pulse of 80/min, and respirations of
14/min. Heart auscultation reveals a low-pitched, rumbling diastolic murmur that is preceded
by an opening snap. The affected cardiac valve is best auscultated at which of the following
regions?
A. Between the second and third intercostal spaces at the right sternal border
B. Between the second and third intercostal spaces at the left sternal border
C. Between the fifth and sixth intercostal spaces at the left sternal border
D. Between the fifth and sixth intercostal spaces at the right sternal border
E. Between the fifth and sixth intercostal spaces in the left mid-clavicular line

Explanation:
Mitral stenosis (MS) arises from a thickening of the mitral valve leaflets that obstructs blood
flow from the left atrium to the left ventricle. This obstruction results in increased pressure
within the left atrium, the pulmonary vasculature, and the right ventricle and atrium. The most
common cause of MS is rheumatic fever, as in this patient; involvement of the mitral valve is
thought to occur in up to 90% of individuals with rheumatic heart disease. Mitral stenosis is
best heard with the bell of the stethoscope at the cardiac apex, which is located between the
fifth and sixth intercostal spaces in the left mid-clavicular line (Choice E). It may also prove
helpful if the patient lies in the left lateral decubitus position.
The aortic valve is best auscultated between the second and third intercostal spaces at the
right sternal border (Choice A).
The pulmonic valve is best auscultated between the second and third intercostal spaces at the
left sternal border (Choice B).
The tricuspid valve is best auscultated between the fifth and sixth intercostal spaces at the left
sternal border (Choice C).
No valve is best auscultated between the fifth and sixth intercostal spaces at the right sternal
border (Choice D).
Educational Objective:
Mitral stenosis is best heard with the bell of the stethoscope at the cardiac apex, which is
located between the fifth and sixth intercostal spaces in the left mid-clavicular line. It may also
prove helpful if the patient lies in the left lateral decubitus position.
66% of people answered this question correctly;

3/19/2011 Item 32 of 44
usmleworld.com/Step3//qstShow.as 1/1

A 62-year-old Caucasian woman presents to the office with complaints of chest discomfort
and mild shortness of breath while walking. She started having these symptoms after starting
an exercise program two weeks ago. She describes the pain as a dull pressure, which starts
within a few minutes after the beginning of exercise, and resolves with rest. Her past medical
problems include hypertension, hyperlipidemia, and a history of panic attacks. She takes
aspirin, atorvastatin, and lisinopril daily. She denies any history of smoking or alcohol intake.
Her physical examination and resting electrocardiogram is unremarkable. Which of the
following is the most appropriate for the initial assessment of this patient?
A. Close observation and follow up
B. Exercise electrocardiography
C. Exercise echocardiography
D. Exercise technetium-99m myocardial perfusion imaging
E. Adenosine thallium perfusion imaging

Explanation:
The patient in the vignette has the classic symptoms of coronary artery disease. In a patient
with known or suspected coronary artery disease (CAD), the diagnosis and risk stratification
should be performed initially by noninvasive tests for myocardial ischemia. According to the
American College of Cardiology/American Heart Association (ACC/AHA) guidelines,
exercise electrocardiography should be performed as the initial test in patients with
intermediate pretest probability of CAD who have a normal resting EKG and are able to
exercise. Such patients have a normal LV function and generally have an excellent prognosis;
therefore, a stepwise approach is recommended.
(Choice A) Observation alone is not recommended in patients with suspected CAD. These
patients need to be risk stratified by using noninvasive tests.
(Choice C) Stress/exercise echocardiography can be used for the diagnosis of CAD. It also
provides additional information regarding left ventricular and valvular functions. However, it is
not recommended as the initial test for the diagnosis of suspected CAD.
(Choices D and E) Technetium-99m and adenosine thallium perfusion imaging are different
types of radionuclide perfusion imaging modalities used frequently for the assessment of
known or suspected CAD. They should be used in patients with complete left bundle branch
block (LBBB), electronically-paced ventricular rhythm, preexcitation (WPW) syndromes, >1
mm ST segment depression at rest, LVH with repolarization changes, inability to exercise,
and a prior history of revascularization.
Educational Objective:
Exercise electrocardiography should be the initial test of choice for evaluation of suspected
CAD in patients with a normal resting EKG and the ability to exercise.
65% of people answered this question correctly;

3/19/2011 Item 33 of 44
usmleworld.com/Step3//qstShow.as 1/2
3/19/2011 Item 33 of 44
usmleworld.com/Step3//qstShow.as 2/2

A 56-year-old Caucasian man is brought to the emergency department with a four hour history
of crushing, substernal chest pain. His electrocardiogram (EKG) shows a 3 mm ST segment
elevation in leads V2 - V4. He is taken for cardiac catheterization and he has primary
angioplasty and stenting of the occluded left anterior descending (LAD) artery. There is 50%
stenosis of the right coronary artery (RCA) which is not intervened upon. After the procedure,
he is admitted to the coronary care unit. Echocardiogram reveals mild anterior wall
hypokinesis and a normal left ventricular ejection fraction. Throughout his three day hospital
stay he remains asymptomatic. On the day of discharge, he asks you whether it would be safe
for him to resume sexual activity. Which of the following is the most appropriate response?
A. It is safe to resume sexual activity after discharge because it carries no risk
B. It is safe to resume sexual activity within a week if he has no chest pain
C. It is safe to resume sexual activity after 4-6 weeks
D. It is safe to resume sexual activity after 4-6 months

Explanation:
Patients with cardiac disease are often anxious about their ability to engage in sexual activity.
Sexual activity results in a moderate increase in heart rate and blood pressure. Sexual activity
has been shown to be a possible precipitant of myocardial infarction within the first two hours,
with a relative risk of approximately 2.5 during that time period, although the attributable risk is
rather small. The Second Princeton Consensus Panel on sexual activity and cardiac risk
published a set of guidelines in 2005. Patients can be categorized as low, intermediate, or
high risk. Patients deemed to be at intermediate risk should have stress testing prior to
engaging in sexual activity and high risk patients should be optimized from a cardiac
perspective before engaging in sexual activity. Patients who had an uncomplicated myocardial
infarction greater than 6 weeks ago are categorized as low risk as long as they do not have
exercise-induced ischemia or have undergone coronary revascularization. Had this patient not
undergone revascularization, he would be classified as intermediate risk even after 6 weeks.
All patients between 2 and 6 weeks following MI are categorized as intermediate risk and all
patients less than two weeks out are categorized as high risk.
(Choice A) Patients are considered at high risk of recurrent myocardial infarction if they
engage in sexual activity within the first 2 weeks.
(Choice B) While the absence of stress-induced angina is a component of risk stratification
for resumption of sexual activity post-myocardial infarction, this is only true of patients who are
at least 6 weeks out from their event. All patients less than 2 weeks post myocardial infarction
are considered high risk.
(Choice D) Waiting 4-6 months after myocardial infarction to resume sexual activity is an
unnecessarily long period of time.
Educational objective:
All patients should refrain from sexual activity for the first 2 weeks following myocardial

3/19/2011 Item 34 of 44
usmleworld.com/Step3//qstShow.as 1/2
infarction. Post-MI patients who have undergone coronary revascularization are considered
low-risk for repeat MI, and can resume sexual activity at 6 weeks.
48% of people answered this question correctly;
3/19/2011 Item 34 of 44
usmleworld.com/Step3//qstShow.as 2/2

A 62-year-old Caucasian female presents to your office complaining of lower extremity
edema. She has never had such edema before and seems scared. She denies dyspnea,
cough or palpitations. Her past medical history is significant for hypertension, diabetes
mellitus, type 2 and chronic bronchitis. She smokes one-and-half packs of cigarettes a day.
Her current medications are enalapril, diltiazem, hydrochlorothiazide and glyburide. She also
takes ipratropium for her chronic bronchitis and uses salbutamol occasionally to relieve acute
shortness of breath and wheezing. Her blood pressure is 140/90 mmHg and heart rate is
65/min. Physical examination reveals symmetric lower extremity edema, but is otherwise
insignificant. Which of the following is the most likely potential cause of this patient's
complaints?
A. Enalapril
B. Diltiazem
C. Hydrochlorothiazide
D. Glyburide
E. Ipratropium

Explanation:
Peripheral edema is a well-known side effect of calcium antagonist therapy. It is more
common with dihydropyridine agents (e.g. nifedipine, amlodipine), but also occurs in 2 to 15%
of patients taking diltiazem. The exact mechanism of calcium antagonist-associated edema is
not known; arteriolar dilatation seems to be responsible for increased interstitial fluid
accumulation in these patients.
(Choice A) ACE inhibitors can rarely cause angioneurotic edema, but it is not usually
dependent and symmetric.
(Choices C, D, and E) The other agents listed do not cause peripheral edema.
Educational Objective:
Peripheral edema is a well-known side effect of calcium antagonist therapy.
59% of people answered this question correctly;

3/19/2011 Item 35 of 44
usmleworld.com/Step3//qstShow.as 1/1

A 46-year-old male with type-2 diabetes mellitus is scheduled to undergo cardiac
catheterization in two days for an undiagnosed episodic chest pain. The patient has been
having retrosternal chest pain radiating to his arm for the past 15 days. The pain is gradually
becoming more frequent. He went to the emergency department about 7 days ago following
an episode of chest pain. He was admitted overnight, and he left against medical advice when
three sets of cardiac enzymes were negative. His EKG revealed nonspecific ST-T changes.
The patient has had diabetes for the past six years, and has been on metformin (1,000 mg
twice daily) for the past several years. He does not perform self-monitoring of his blood sugar.
His chemistry profile and CBC from his last emergency department visit were normal. He also
has a history of hypertension that is controlled with atenolol (50 mg once daily) and
hydrochlorothiazide (12.5 mg once daily). Baby aspirin (81 mg once daily) was started on his
last emergency room visit. He smokes one pack a day, and drinks alcohol socially. He has a
strong family history of premature coronary artery disease. On examination, he is 5'6" (165
cm) tall, and weighs 180 lbs. (82 kg). His blood pressure is 136/70 mmHg and pulse rate is
66/min. The rest of the clinical examination is normal. What is the next best step in this
patient's care?
A. Stop metformin
B. Stop hydrochlorothiazide
C. Increase atenolol
D. Discontinue aspirin
E. No change in therapy until cardiac catheterization

Explanation:
Metformin was approved in the United States for the treatment of type-2 diabetes after its use
in the rest of the world showed that the chances of lactic acidosis are minimal when used in
patients without any contraindications. Major contraindications include renal insufficiency
(creatinine more than 1.5 mg/dl in males, 1.4 mg/dl in females or creatinine clearance less
than 60 ml/min), hepatic dysfunction, alcoholics, sepsis, and congestive heart failure. Because
of its potential to cause renal dysfunction when a large amount of contrast is being infused,
metformin is typically held before or at the time of procedure. It should not be restarted unless
normal renal function is documented following the procedure.
(Choice B) Hydrochlorothiazide is usually not associated with any problems with cardiac
catheterization, especially when used in low doses.
(Choice C) There is no need to increase atenolol dosage, because the patient appears to be
adequately beta-blocked.
(Choice D) Aspirin can be continued safely during cardiac catheterization. In procedures
where an increased risk for bleeding is expected, aspirin should be discontinued at least
seven days before the procedure, because it may cause platelet dysfunction that can last for
more than a week.

3/19/2011 Item 36 of 44
usmleworld.com/Step3//qstShow.as 1/2
(Choice E) Metformin needs to be discontinued before cardiac catheterization.
Educational Objective:
Metformin use is contraindicated in patients with renal failure, sepsis, hepatic dysfunction, and
severe heart failure. It should also be stopped in patients who are at risk to develop renal
failure, such as those who will undergo angiography, a procedure that involves infusion of a
high load of contrast agents.
52% of people answered this question correctly;
3/19/2011 Item 36 of 44
usmleworld.com/Step3//qstShow.as 2/2

A 66-year-old African-American female comes to the office for a follow-up visit. She was in the
hospital two weeks ago with complaints of a sudden onset of dizziness and palpitations. Her
initial electrocardiogram in the hospital revealed atrial fibrillation with rapid ventricular
response. She was started on diltiazem drip for rate control. The next day, she spontaneously
converted back into sinus rhythm. She has a history of hypertension, diabetes mellitus,
coronary artery disease status post coronary artery bypass grafting x 2 five years ago,
ischemic cardiomyopathy with an ejection fraction of 35%, and osteoarthritis. Her medications
include daily aspirin, metoprolol, hydrochlorothiazide, lisinopril, atorvastatin, insulin, warfarin,
and ibuprofen as needed. In the office, she tells you that she has been feeling well in general.
Occasionally, she has a few episodes of palpitations with slight lightheadedness, which lasts
only for a few minutes. Her temperature is 36.7C (98F), blood pressure is 126/82 mmHg,
heart rate is 82/min, and respiratory rate is 14/min. Her lungs are clear on auscultation; there
are irregularly irregular heart sounds without any audible murmurs. An electrocardiogram done
in the office reveals atrial fibrillation. Which antiarrhythmic agent would you use to keep the
patient in sinus rhythm?
A. Amiodarone
B. Ibutilide
C. Diltiazem
D. Verapamil
E. Flecainide

Explanation:
The patient in the above vignette has a classic presentation of paroxysmal atrial fibrillation.
Patients with this condition may present with recurrent episodes of palpitations, which resolve
spontaneously over time; however, a great majority of patients may remain asymptomatic and
are unaware of the paroxysms of atrial fibrillation. This is especially true if they are on
medications to slow their heart rate (beta-blockers or calcium channel blockers).
Patients with paroxysmal atrial fibrillation are at a similar risk of significant thromboembolism
and stroke as patients with persistent atrial fibrillation. All patients therefore need to be
anticoagulated with warfarin to keep their INR between 2.0 to 3.0; however, there has been a
great deal of debate recently regarding the need to keep all such patients in sinus rhythm with
the use of antiarrhythmic drugs. In the past, many cardiologists preferred a rhythm control
strategy over a rate control one in most patients with atrial fibrillation; however, a number of
recent clinical studies have shown that both these approaches are acceptable, and rate
control with AV-nodal blocking agents (beta-blockers, calcium channel blockers, or digoxin)
was not inferior to maintenance of sinus rhythm in such patients.
Rhythm control strategy is still recommended in patients with marked and persistent symptoms
(for example, palpitations, dizziness, dyspnea) or with hemodynamic instability.
Recommendations for the use of antiarrhythmic drugs to maintain sinus rhythm are based on
the presence or absence of structural heart disease. Structural heart disease is defined by the

3/19/2011 Item 37 of 44
usmleworld.com/Step3//qstShow.as 1/2
presence of coronary artery disease, congestive heart failure due to ischemic or non-ischemic
cardiomyopathy, or hypertension with left ventricular hypertrophy. In all such patients,
amiodarone is the preferred antiarrhythmic drug for the maintenance of sinus rhythm. It has a
better efficacy than sotalol and most of the other class I antiarrhythmics and has very low
incidence of fatal arrhythmias.
(Choice B) Ibutilide is a class III antiarrhythmic drug and has been approved for the acute
termination of atrial fibrillation. It is only available in intravenous form and therefore is not useful
for long-term maintenance of sinus rhythm.
(Choices C and D) Calcium channel blockers (diltiazem or verapamil) have not been shown
to prevent recurrence of atrial fibrillation in the long term in patients with paroxysmal atrial
fibrillation.
(Choice E) Flecainide is a class Ic antiarrhythmic agent and is recommended to maintain
normal sinus rhythm in patients without any structural heart disease; however, it can lead to
fatal arrhythmias in patients with structural heart disease and can cause a significant increase
in mortality.
Educational objective:
Amiodarone is the recommended first line antiarrhythmic drug to maintain normal sinus rhythm
in patients with paroxysmal atrial fibrillation and structural heart disease.
44% of people answered this question correctly;
3/19/2011 Item 37 of 44
usmleworld.com/Step3//qstShow.as 2/2

A 65-year-old male is being evaluated for jaundice and elevated liver enzymes. He is
scheduled to undergo endoscopic retrograde cholangiopancreatography (ERCP) to explore
the possibility of common bile duct stricture or stones. His past medical history is significant
for coronary artery disease, ischemic cardiomyopathy, paroxysmal atrial fibrillation,
hypertension, hyperlipidemia, and diet-controlled diabetes mellitus. He has a history of
ischemic stroke with minimal residual right-sided weakness. He had a right coronary artery
stent placed five years ago. His last echocardiogram was three months ago and showed a
mildly dilated left ventricle with an ejection fraction of 40-45% and mild mitral regurgitation. He
can walk one block on level ground but has to stop frequently because of shortness of breath
and "leg tiredness." He has a 30 pack-year smoking history but quit 15 years ago. His blood
pressure is 134/78 mmHg and his heart rate is 85/min. His BMI is 33 kg/m
2
. Physical
examination reveals no murmurs or additional sounds. His chest is clear on auscultation.
There is no peripheral edema. His EKG shows sinus rhythm and pathologic Q waves in the
inferior leads. Which of the following is the best pre-operative/pre-procedural management for
this patient?
A. Offer coronary angiography
B. Proceed to ERCP without further testing
C. Repeat resting echocardiogram
D. Schedule EKG stress testing
E. Schedule myocardial perfusion imaging

Explanation:
The preoperative assessment of a patient's cardiac risk (per the 2007 American College of
Cardiology/American Heart Association, or ACC/AHA, guidelines) depends on the patient's
cardiac risk factors, the patient's functional capacity, and the risk of the proposed surgery. The
results of prior invasive and noninvasive cardiac imaging should also be taken into
consideration. This patient has a multitude of cardiac risk factors as well as known coronary
artery disease that required coronary artery stent placement in the past. His history of
frequently feeling short of breath while walking one block suggests that his functional status is
less than ideal. However, the patient is having an endoscopic procedure which is considered
a low risk procedure (risk of cardiac death or nonfatal myocardial infarction of less than 1%).
Most ambulatory or superficial surgeries are considered low risk. Vascular surgery is generally
considered high risk. Intrathoracic, intraperitoneal, or orthopedic surgery is generally
considered intermediate risk. For procedures that are categorized as low risk, further
preoperative cardiac testing is usually not required and therefore this patient can proceed with
the planned ERCP.
(Choice A) Even when some form of preoperative evaluation is needed, it is usually
noninvasive to start, with coronary angiography reserved for patients with concerning results
on a prior noninvasive test. When the ACC/AHA approach is used, few patients undergo
coronary angiography and less than 2% of total patients will require revascularization.

3/19/2011 Item 38 of 44
usmleworld.com/Step3//qstShow.as 1/2
(Choice C) Regardless of any abnormalities on this patient's resting echocardiogram, the
proposed ERCP is low risk and therefore no further preoperative evaluation is necessary.
(Choice D) If a stress test is performed, either a stress nuclear medicine perfusion study or
stress echocardiogram should be performed, rather than EKG stress testing. This is
particularly true for patients with resting EKG abnormalities, such as this patient's Q waves.
(Choice E) Most would recommend a noninvasive stress test in a patient with three or more
risk factors or a poor functional capacity who is undergoing high risk surgery, such as vascular
surgery. Noninvasive imaging would also be considered in similar patients undergoing
intermediate risk surgery or patients with one risk factor undergoing high risk surgery.
Educational objective:
The preoperative assessment of a patient's cardiac risk (per the 2007 ACC/AHA guidelines)
depends on the patient's cardiac risk factors, the patient's functional capacity, and the risk of
the proposed surgery. Low risk procedures do not need preoperative evaluation regardless of
the patient's risk factors.
53% of people answered this question correctly;
3/19/2011 Item 38 of 44
usmleworld.com/Step3//qstShow.as 2/2

A 55-year-old male with type 2 diabetes and hypercholesterolemia is brought to the
emergency department (ED) because of an acute, substernal chest pain. His chest pain
resolves after taking sublingual nitroglycerine and aspirin. The patient uses dietary means to
control his blood glucose levels, which are currently borderline. He takes 20 mg of simvastatin
every day. The initial electrocardiogram shows a new 1-mm ST segment depression in leads
II, III, aVF, V2-V5. After admission, his cardiac enzymes are found to be elevated. Cardiac
catheterization shows >90% blockage in the right coronary, left circumflex and left anterior
descending arteries. His HBA1c is 7.2 % (normal 4 - 6 %) and his current LDL is 120 mg/dl
(goal <100 mg/dl in diabetes). What is the most effective way of dealing with this patient's
coronary artery disease?
A. Start insulin therapy
B. Coronary artery bypass graft
C. Angioplasty with stent placement
D. Balloon angioplasty alone
E. Increase the dose of simvastatin

Explanation:
Studies have shown that compared to angioplasty, coronary artery bypass graft (CABG)
improves long-term survival in diabetic patients with multivessel disease and recent Q-wave
infarction. Good, perioperative control of the blood glucose levels is advocated during CABG.
An insulin drip achieves better blood glucose control during CABG, and decreases the
incidence of sternotomy wound infections.
(Choices C and D) Aside from the abovementioned reasons, trials have shown that patients
with diabetes have a higher incidence of restenosis after angioplasty.
(Choice A) Insulin treatment could be necessary in this patient during cardiac surgery.
Furthermore, improved glycemic control after an acute myocardial infarction has been shown
to improve long-term survival; however, giving insulin treatment without subjecting the patient to
CABG would be detrimental.
(Choice E) Similarly, lowering of LDL is essential, but if done alone, would not be enough for
opening up the significant blockage in the patient's coronary arteries. Improved glycemic
control and lower LDL levels are likely to be beneficial in the long run.
Educational Objective:
Coronary artery bypass graft is indicated in patient's with severe 3-vessel coronary artery
disease or left main disease.
79% of people answered this question correctly;

3/19/2011 Item 39 of 44
usmleworld.com/Step3//qstShow.as 1/1

A 22-month-old boy who recently immigrated from Eastern Europe with his family is brought to
your office by his mother who is worried that he seems to tire easily. She describes how he
becomes short of breath with even mild physical activity and cannot keep up with his peers
during play. The boy is at the 40
th
percentile for height and 20
th
percentile for weight. On
general examination, there is no clubbing, cyanosis, or peripheral edema, but there is a harsh
3/6 holosystolic murmur over the left lower sternal border and a rumbling diastolic murmur over
the cardiac apex. Which of the following is the most likely cause of his symptoms?
A. Tetralogy of Fallot
B. Atrial septal defect
C. Ventricular septal defect
D. Transposition of the great vessels
E. Mitral stenosis

Explanation:
This patient has a ventricular septal defect (VSD), the most common congenital heart
malformation. VSDs range from small and asymptomatic to large with significant left-to-right
shunting. There are several indications that this patient has a significant VSD with left-to-right
shunt. First, he is symptomatic. Second, he has the telltale murmur. (Affected patients have
pansystolic murmurs loudest at the left lower sternal border. They can also have diastolic
murmurs at the apex because of increased flow across the mitral valve.) This patient should
have an echocardiogram with a bubble study to evaluate his congenital heart defect.
(Choice A) Tetralogy of Fallot is the most common cyanotic heart malformation. It consists of
a VSD, pulmonic stenosis, overriding aorta, and right ventricular hypertrophy. The patient
above is not cyanotic.
(Choice B) Atrial septal defects are relatively common among children and may cause
symptoms like those described above. On exam, however, there is a widely split and fixed S2,
systolic ejection murmur in the left upper sternal border, and, if there is a large shunt, a
diastolic murmur at the left lower sternal border.
(Choice D) Transposition of the great vessels is a congenital cyanotic heart disease where
the aorta comes off the right ventricle and a pulmonary artery comes off the left ventricle. It is
incompatible with life unless there is also an intracardiac shunt. The patient in this vignette is
not cyanotic and it would be extraordinarily unusual for a patient with transposition to survive to
22 months without surgery.
(Choice E) Mitral stenosis also causes a diastolic rumble but does not typically cause a
pansystolic murmur at the left lower sternal border unless there is tricuspid regurgitation as
well. This patient also lacks other features to suggest rheumatic fever such as fever, arthritis,
or skin lesions.

3/19/2011 Item 40 of 44
usmleworld.com/Step3//qstShow.as 1/2
Educational objective:
Ventricular septal defects can cause failure to thrive, easy fatigability, and heart failure. On
exam they characteristically cause a pansystolic murmur loudest at the left lower sternal
border, plus diastolic flow murmurs at the apex.
55% of people answered this question correctly;
3/19/2011 Item 40 of 44
usmleworld.com/Step3//qstShow.as 2/2

You are beginning a cardiology fellowship that strongly encourages research. One of the
attending physicians is about to begin experimenting with a new type of drug eluting stent that
will be used in patients with ST-elevation myocardial infarctions undergoing percutaneous
coronary artery intervention. Unfortunately, a large portion of the hospitals patient population
is of higher socioeconomic status and unlikely to participate in the experiment without
significant compensation that would draw funding away from other experiments that the
attending is involved with. There is a large prisoner population that uses the hospital. Your
attending tells you that you should get them to participate for free. What is the best response to
your attendings request?
A. "That sounds like a great idea, and should not be a problem."
B. "They are required to accept the treatment we offer them since they are inmates in
the state prison."
C. "Prisoners cannot participate in research studies."
D. "I will ask them to participate, but we have to offer them the same compensation as
our other patients."
E. "I will not ask them to participate because they cannot accept compensation from the
hospital."

Explanation:
Experimentation with prisoners or other captives is an issue that dates back to research
performed by Nazi physicians in World War II and, more recently, the Tuskegee Syphilis Study.
Prisoners are at an obvious disadvantage if they are forced to participate in medical research.
It is generally appropriate, however, to allow prisoners to participate in clinical research if they
are fully informed, give their written consent, and are compensated in the same way non-
prisoner participants would be.
(Choice A) This would be a problem since the attending is not prepared to compensate the
prisoner population. Furthermore, focusing on such a narrow group of participants could result
in a significant population bias.
(Choice B) Competent patients are never required to accept treatment without their consent.
This includes prisoners.
(Choice C) Prisoners can participate in research as long as they do so of their own free will
and the benefits they are provided with by the study are equal to those provided to non-
prisoner participants.
(Choice E) Prisoners can accept compensation, and the compensation must be the same as
that offered to non-prisoners.
Educational objective:
Prisoners must be given equal rights to non-prisoners regarding participation in medical
research, and should not be taken advantage of because of their legal status.

3/19/2011 Item 41 of 44
usmleworld.com/Step3//qstShow.as 1/2
82% of people answered this question correctly;
3/19/2011 Item 41 of 44
usmleworld.com/Step3//qstShow.as 2/2

A 74-year-old Caucasian female is brought to the physician's office by her daughter because
she has been coughing a lot for the past four weeks. Her cough is present throughout the day,
and is most prominent at nighttime. She easily gets "out of breath" while walking around the
house. She denies fevers, rigors, or sputum production. She has a past medical history of
hypertension, atrial fibrillation, congestive heart failure with an ejection fraction of 35%, history
of ventricular tachycardia, hypothyroidism, and vascular dementia. Her daily medications
include aspirin, metoprolol, furosemide, levothyroxine, amiodarone, and multivitamins. She is
allergic to ACE inhibitors and penicillin. On examination, her temperature is 37.2C (99F),
blood pressure is 110/82 mmHg, heart rate is 82/min, and respiratory rate is 20/min. Her
mucous membranes are moist. There is no evidence of jugular venous distention. The lung
examination reveals equal and normal air entry on both sides, with fine end inspiratory
crackles heard at both the lung bases. Cardiovascular examination reveals a regular heart
rhythm with no evidence of S3 gallop or murmurs. The rest of her physical examination is
unremarkable. An initial chest x-ray reveals the presence of a normal cardiac silhouette. There
is no evidence of pleural effusions. There are diffuse interstitial opacities seen bilaterally in the
lower lung fields. Which of the following is the most appropriate next step in her management?
A. Stop amiodarone
B. Increase the dose of furosemide (Lasix)
C. Discontinue metoprolol
D. Increase the dose of metoprolol
E. Start digoxin

Explanation:
The patient has a clinical presentation consistent with the development of amiodarone-
induced pulmonary toxicity. Amiodarone is a commonly used antiarrhythmic drug for the
treatment of ventricular and supraventricular tachyarrhythmias. Prolonged treatment with
amiodarone is associated with a variety of adverse effects including photosensitivity, skin
discoloration, bone marrow suppression, thyroid dysfunction, abnormal liver function tests, and
pulmonary toxicity. Of all the above, pulmonary toxicity is the most serious adverse effect that
is responsible for most of the deaths associated with amiodarone therapy.
Pulmonary toxicity with amiodarone can be seen in the form of chronic interstitial pneumonitis,
organizing pneumonia, acute respiratory distress syndrome, and rarely with a solitary
pulmonary mass. Chronic interstitial pneumonitis is the most common presentation of
amiodarone-induced pulmonary toxicity. It is characterized by the presence of nonproductive
cough, fever, pleuritic chest pain, weight loss, dyspnea on exertion and a focal or diffuse
interstitial opacity on the chest radiograph. It is usually seen after months to years of
amiodarone therapy, especially in patients who are on higher maintenance doses (more than
400 mg/day). It is a cumulative dose effect, and the serum amiodarone levels are usually within
the normal range in these patients. The chest radiograph may reveal the presence of diffuse or
focal interstitial or alveolar opacities.

3/19/2011 Item 42 of 44
usmleworld.com/Step3//qstShow.as 1/2
Discontinuation of amiodarone is the mainstay of treatment for amiodarone-induced
pulmonary toxicity. Corticosteroids can be used in patients with severe or life-threatening
pulmonary disease. The prognosis is usually good in most cases, and majority of patients
either stabilize or improve after the complete withdrawal of the drug.
(Choices B and C) Based on the clinical presentation and chest radiographic findings, the
patient's symptoms are unlikely to be from congestive heart failure exacerbation (absence of
cardiomegaly, cephalization of pulmonary vessels, and pleural effusions on the chest
radiograph). Therefore, increasing the dose of Lasix or reducing the dose of beta-blocker is
not indicated at this point.
(Choice D) Increasing the dose of metoprolol would have no effect on the course of
amiodarone-induced pulmonary toxicity or chronic interstitial pneumonitis.
Educational Objective:
Prolonged therapy with amiodarone can cause a serious and potentially fatal pulmonary
disease. Its treatment primarily consists of complete withdrawal of amiodarone.
Corticosteroids can be used in patients with severe or life-threatening pulmonary disease.
57% of people answered this question correctly;
3/19/2011 Item 42 of 44
usmleworld.com/Step3//qstShow.as 2/2

A 53-year-old Caucasian male presents to your office for a follow-up visit. His previous visit
two weeks ago revealed a blood pressure of 145/95 mmHg. He has no present complaints.
His past medical history is significant for diabetes mellitus, type 2, which is being treated with
glyburide. He smokes two packs of cigarettes and consumes 1-2 bottles of beer daily. His BMI
is 29.5 kg/m2. His blood pressure is 150/95 mmHg and his heart rate is 80/min. The physical
examination, including ophthalmoscopy findings, is normal. You explain to the patient that
lifestyle changes may decrease his risk of future cardiovascular events. Which of the following
non-pharmacological measures would be the most effective in decreasing this patient's blood
pressure?
A. Weight reduction
B. Decreased sodium intake
C. Smoking cessation
D. Moderation of alcohol consumption
E. Moderate exercise

Explanation:
Almost all guidelines recommend non-pharmacologic measures that can be preventive,
adjunctive or sometimes definitive in the treatment of hypertension. Weight loss enhances the
effect of anti-hypertensive drugs. It is the single most effective non-pharmacologic measure to
decrease blood pressure in overweight individuals, and it is believed to reduce the overall
cardiovascular risk in such patients.
(Choice B) Although the effect of weight loss on hypertension is independent of sodium
intake, a decrease in sodium intake may enhance the blood pressure-lowering effect.
(Choice E) Increased moderate-intensity physical activity can lower blood pressure in both
hypertensive and normotensive individuals; however, this is less efficacious than weight
reduction.
(Choice D) Moderate alcohol consumption may help to control hypertension since it is also
believed to decrease the overall cardiovascular risk of patients.
(Choice C) Smoking cessation is an important component of lifestyle modification that
reduces the risk of future cardiovascular events.
Educational Objective:
Weight loss is the single most effective non-pharmacologic measure to decrease blood
pressure in overweight individuals.
45% of people answered this question correctly;

3/19/2011 Item 43 of 44
usmleworld.com/Step3//qstShow.as 1/1

A 63-year-old Caucasian woman is admitted to the hospital for an episode of paroxysmal
atrial fibrillation that caused palpitations and dizziness. She is started on metoprolol and
amiodarone. Her past medical history is also significant for deep venous thrombosis two
months ago, and she has been on chronic anticoagulation therapy with warfarin since then.
Upon discharge, her blood pressure is 131/72 and her heart rate is 72/min and regular.
Examination shows no abnormalities. Her INR is 2.6. Which of the following is the most
effective strategy to decrease this patient's risk of adverse drug reactions?
A. Continue current dose of warfarin and re-check INR in one week
B. Decrease warfarin dose by 25%
C. Double the dose of warfarin
D. Increase warfarin dose by 25%
E. Stop warfarin and re-check INR in one week

Explanation:
Administration of new medications should always be done cautiously in patients on chronic
anticoagulation with warfarin. Warfarin is metabolized by the liver, and new medications can
easily lead to either subtherapeutic or supratherapeutic INR levels that lead to complications.
Amiodarone is a common medication taken by patients who are also on chronic warfarin.
Amiodarone slows metabolism of warfarin by the liver and can lead to supratherapeutic INR
levels. The dose of warfarin should be decreased by 25% when amiodarone is introduced and
the INR should be closely followed until it has reached a new stable level. Other medications
that can increase the effect of warfarin include cephalosporins, ciprofloxacin, erythromycin,
and fluconazole.
(Choice A) Continuing the current dose of warfarin will lead to a supratherapeutic INR, and the
patient could easily suffer a bleeding complication over the course of the intervening week until
the INR is rechecked.
(Choice C) Doubling the warfarin dose could be considered if a new drug was administered
that enhanced warfarin metabolism, although adjustments to warfarin dose are generally made
in smaller increments.
(Choice D) Increasing the warfarin dose by 25% would be reasonable if the patient was
started on a drug that increases warfarin metabolism, such as rifampin or phenobarbital.
(Choice E) Stopping warfarin is inappropriate as this will leave the patient inadequately
anticoagulated.
Educational objective:
Warfarin interacts with a large number of medications that can easily lead to either under or
over anticoagulation. These changes need to be anticipated with appropriate adjustments
made in warfarin dose and with subsequent close INR monitoring until a new baseline is
established.

3/19/2011 Item 44 of 44
usmleworld.com/Step3//qstShow.as 1/2
26% of people answered this question correctly;
3/19/2011 Item 44 of 44
usmleworld.com/Step3//qstShow.as 2/2

Vous aimerez peut-être aussi